You are on page 1of 334

MANAGEMENT PROGRAMME

Term-End Examination
December.2O05

M S - l @ : MANAGEMENT FUNCTIONS
AND BEHAVIOUR
Time : 3 hours MaximumMorks: 100
(weishtage 70%)

(i) There are tuo SectiansA and B.


(ii) Attempt ony thtee questionslrcm SectionA, eoch
question carrging 20 morks.
(iii) Section B is compulsory ond carries40 marks. t

SECTION A

1. Id€ntifuvariousresponsibilitjes
of a professional manager.
Hightght any live impodant responsibilitjesol a manager
in an organizationalset,up and bring out ih€ir functional
consequences with suitableexamples.

MS-]@ P.T.O.
2 . Exptain with examples the various phases and types of
managerialdecisionFaklng.

3. Describe how and why conflict is generated in th€


organization and briefly discuss various mod€s of conflict

4. Having designed the organisation structure, io suit the


specific needs of the company and its eNnronment, on the
basis of most efficient grouping, it becomesdesirableto
ensure that thes€ diflerentiated groups are iniegrated
'
iowards th€ common organisational objectives

Elaborate and discuss the under\ing issues in this

5.. Writ€ short notes on any three oI the following :


(i) Sustainingleadershipelfectiveness
(ii) Manaserialvaluesand ethos

{iii} Matrix structure


d
{iv) Policies and design choices in conirol

M Challengas of change

MS-1o
SECTION B

6. Pleaseread the caseand an$rer the questionsgiven at the


end.

Mr- Banerjee is the Chjef Executive of a medium


siz€d pharmaceuticallirm in Calcutta.He holds a Ph.D.
in Pharmacy. However, h€ has not been involved in
research and developm€nt of new products for txo
decades. Though turnover is not a problem for th€
company, Mr. Banerjeeand his senior colleaguesnoticed
that ihe workers on hourly basis are not working upto
their full poteniial. It is a well known faci that they filled
their days wiih rinn€c€ssaryand unprcductive activities
and worked only for the sake of a pay cheque. In th€
recent past the situation has become quite alarming as
the organizationbegan to crumble under the weight of
uneconomicaleffort. The situationdemandedimmediate
managerialattention and prompt rectificationalmeasures.
Mr. Banerjee kn€w very well that the onlg way to
progress and prosper is to motivate work€rs to p€ak
perlomance through variousincentiveplans.

One fine moming, Mr. Banerj€e contacted ihe


PersonnelManager and enquired: "What is the problem
t,ith tbe urorkers on hourly basls ? The wage bill shows
that we pay ihem the highest in the industrg. Our
working conditions are fine. Our fringe benefits ar€
excellent.Still these workers are not motivated.What do

MS-l@ P.T,O
th€y require reallg ?" The personnel Manager gave rne
follo\ring reply : "l have alreadyinformed you a number
of iimes, that money, working conditionsand ben€fiisare
not enough- Other things are €qually ihportant. One of
the workers in that group rec€ntly gave me a clue as to
why more and more workers ara joining the bandwagon
of non-performers'. He felt bad thai hard work and
efficiency go unnoticed and unrewarded in our
organization.Our promotions and benefit plans are tied
to length oI service.Even the lazy workers, accordingly,
enjoy all the benefjtsin the organization.which, in fact,
according to the worker, should go io only those rvho
work hard." Mr Banerjee then wanted the personnel
Manager io look into the problem more closelyand find
out a solutionio the probl€msof workers on hourly basis.

Questions :
(a) What is the key problem in the case? What probably
may be ihe "other things" which the personnel
Manger is pointing out to Mr. Ban€rjee?
(bJ Discuss the problem of motiv?tjon in this case,
relatingto Herzberg'stheory.
(c) What would be your recommendations as personn€l
Managerto tacklethe problem in the organization?

MSto 1,000
lMctGn
MANAGEMENT PROGRAMME.
Term-End Examlnation
December.2OO5

MS-2@: Pre-Revised
: MANAGINGMEN
Revised: MANAGEMENTOF
HUMANRESOURCES

Time: 3 hours MoximumMorks: 100


Weishtose704
Note: There are two SectionsA and B. Sectlon A hos
' ttlo sets. set I is meont Jor students rrho haue
reglstercd prior to Jonuorg 2005 i.e. upto June,
2004. Set ll is meont for students who hoDe
registered Jrom Januar9,2005 ond onwards.
Attempt any three questlonsJrom Section A. All
' questions corry 20 morks each. Section B is
cotnputsory lor oll and carries 40 morks.

SECTION A
(Sel I)
(Pr€-R€vised)

l. Identily ihe sali€nt f€atures of personnel functions in the


organisational cont€xt keeping in mind strat€gy, structute
and systems. 20

N4s2o P.T.O.
DeIine career planning. Explain implementalion of ihe
process of career planning with the help of examples.

Discuss the salieni features of job analysis. Eiplain what


steps the HR manager ls required to take lor carrying out
the function of job description. 20

Identify the iactors leading to indiscipline. Explain the


judicialprocessof dealingwith indiscipline. 20

5. Writ€ short notes on any lhree of the following ,


(a) Principlesof learning

ft) Behaviourally Anchored Rating Scales


(c) Selecdoncrteria.
(d) Morale
(e) Consultaiivemachinery

MS2@
SECTIONA
(Serll)
(Rerdsed)

l- Define and discuss the objectives of Human Resource


Management. Discuss the chang:ng role of human
resourceEin view of changingsocialscenario.Explainwith
suitable examples. 20

2. Discuss the concept oi outsourcing and its cons€quences


from HR point of view. 20

3. What is workers' pariicipation in management ? Discuss


the lactors responsible for its success. 20

4. Explain lhe processof coaching.What are lhe condilions


necessaryfor effective coaching ? 20

Write short noteqon any trrre€ ol the following:

{a) Team development


(b) E\ecutivecompensation
(c) Separation

{d) Human resource res€arch


(e) Talent management

MS,2@ P.T.O.
SECTION B
Please read the following iwo cases and answer the
questions given at the end of each. 40
(A) Case Study
MANAGEMENT DEVELOPMENT
Any efforts and inputs to improve current ar future
mdnagement performance by imparting knowledge,
changing attitudes and enhancing skill is called
management development. It is an inhouse activity
and includes co-acting, given professional inputs to
increase knowledge and attempts lo change attitudes
and enhance managenal skills. The aim of sitch
programmes is to improv€ the perlormance of the
organisation itself. Th€ Management Development
process consists of (1) Assessins ihe organisational
needs (2) Appraising Manager's Performance
{3) Developing Manager's perlomance by susiained
elforts.
..DELTAPRODUCTS''
Defba is a comparry
Products underone
ma,.tulactufing
shednumberof productsrequiringfor Housingand
Building Consblciion Industry. The products include
hardware like door knobs, hinger, lock, tower bolts"
sanitary fittings and floor tiles. AII requirem€nts of
housing and building construction industry aie
manufactured in different departrnents at tts Bhopal
plant. The company wish€s io produce and rratket
top grade products. Mr- Chakravarty, the Director ol
Operations has been on foreign tour for over lhtee
months to see the manufacturing lacilities at different
plants world over Mr Chakavarty's goal is to give to

tvrs-2
@
the industry, wrirld classquality product at competitive
.rates and pric€s. His ambition is to make DELm the
class one producer in this activity in lndia._Mr
Chakravady is a highly qualitied technocrat wiih
excdtent backgrcunA al engneeins, genera) and
rnatketing management. He had however limited
exposur€ to Human Resource Managem€nt. You are
hired as Manager of Human Resources. The
supporirng team of highly qualified engineers.
purchase managers and marketing managers ar€
young and almost all of them are in the age group
oI 25 lo 28 and very enthusiastic.They are excellent
as individuals but lack the team spirit and work very
tard to excel in their respective specialised
knowledge. Manv problems have come up for lack of
communication, coordination, quality management
and delivery schedules.The plant main@nanceis
below average. The inventories are high and the
industrial relaiions poor as none of the managers had
an9 exposur€ io this subject.
You are required to make a comprehensive
Management Dev€lopment Programme lor
Managers and Assistant Managers.

Questions :
(a) How would you plan a programme for the
yormg managers ?
(b) How will you test the efsting leve! o{ their
skills, attitudes and team spirit ?

{c) What metiods of developmenial training would


you like to use and why ?

Ms-2@ P,T.O,
(B) Case Study

APPRAISING PERFONMANCE

AII progressive compani€s have some formal or


informal appfaising sysiems tof apptaising the
performanceof their employees.
Performance Appraisal is defined as any procedure
that involves assessing employees' performance
against sei standards and, providing feedback to the
employeesassessed.
The aim is to motivate employees (a) to improve
their performance (b) encourage for better
performance above par
The reasonslor perlormance appraisalare (1) For
$alary decisions, rewards, promotion. {2) To review
employee's work related behaviour with a view to
correcting any deficiencies. Appraisal should b€
central to career planning process.

..AJI,IBER PHARMACEUTICALS''

ln a pharma company manulacruringand markenng


drugs and medicines, ihe research staff has
developed a number of new products and
Iormulations which are effectiv€. But at the same
time it has to meet severe competiiion fuonl
stalwartswith loleign collaboration.Mr. Shah, the
Vice President Marketing has a very successful
Pharma Marketing background. He has been with

MS2@
the company lor the past 4 9ears. Mr Shah had
made ambitiousplans for capiuringsizeableshare of
market in. the Guja{at State. The company being
mediurn sized, Mr. Shah had k€pt his marketing
departhent and the marketing team lean and trim.
The fieid sales stafl was given aggressivetargets and
were viftuallg pushed to reach the respective targets.
The field staff worhed to their best abilities to
compieie their respective targeis. Mr Shah had
himself been working hard almost 11- 12 hours a
day. There was no formal apraisal and reward
system in ihe company. During last 5 years more
than 60 Medical Representativesand ihe Aren
Supervisors had left the compang due to
unsaiislactory incrernents and I,romotions. Those
who lefi the company were star worke6. But
Mr Shah did noi care for ihis high turnover He was
over confidentthat he would be able to hire {reshers
and also select candidates who were not happy
wiih their remuneration in their respeciive company.
Mr. Shah had never communicatedto the field sal€s
stalf about their perlormance or reasons tor oot
recognising thei{ outstanding performance in a lew
cases. There was on the whole a great
dissatisfaction
and good perlormerswere l€avingthe

Ms-2@ P.T.O.
Q estions I
(a) What ao you perceive is the basic problem in
.AMBER'
?
(b) What are the steps you nill take serially to
conect th€ situation ?
(c) In the eveni of your suggestinga Performance
Appraisal System
(i) How will you decide a suitable system of
appraisal?
(ii) Will your sysiem include merii, r€wards and
promotions ?

Ms2@ 1,000
MANAGEMENT PROGRAMME
Term-End Examination
Decernber, 2OO5

MS-3@ : ECONOMICAND SOCIAL


ENVIRONMENT

Time : 3 hours Maxinum Morks: 100


(Weishtose 70Vo)

Note , There are turo Se.tions A ond B. Attempt


a.g three questions Jrcm Section A, carrging
20 morks each. Section B is compulsory ond
cofties 40 matks.

SECTION A

1. Describethe criticalelementsoI politicolegalenvironment


ot business. Explain bn€fly the impact of this environment
on businessorgonisations, citing relevant examples. 20

2. While examining the institutional framework tor smatl scale


industries(SSI), comment on iheir contribuhonto Indian
economicdev€lopment. 20

MS-3o P.T.O.
3- Critically analyse the achievements and adverse effecls oI
re$ialory {€mework in ihe course o{ lndia's
industrialisation.

Analyse the trad€ policy re{orms implemented by lndia in


the 90s. What are its implicaijonsin enhancing Indias
competitiven€ss in the intematioDalmarkei ?

5 . Bring out the distinguishing features of Industrial Policy


1991 and its impact on the technologystatus of Indian
indusiry.

Ms-3@
SECTION B

6. Bne V des{:],betbe lndjan Jjnancial sysiem and analysethe


significance and need for linancial relorms.

7. Write short notes on any t..,o of the following : 20


(a) EnvironmentMovement
(b) lndustrial Grov"th Expedence

lr) Currenl Account Converhbi|ly


(d) EconomicReformsand Employment

M s3 @ 1,000
MANAGEMENT PROGRAMME
Term-End Examination
Decembet, 2OO5

MS-4@ : ACCoUNTINGAND FINANCE


FOR MANAGERS

Time : 3 hours MoxtmumMorksrl00


(Weightase70qo)

Note : Attempt ang five questions. All questions corry


equol morks.

1. (a) What do you \rndersiand by Management


Accounting ? How does it diller lrom Financial
Accouniing ? Discuss.

Ol Whv does an accountant Jollou., the principle


"anticipate no prolit, provide {or all losses" ? On
which accountingconcept is this based ? Explain it
and discussits significance.

Ms4o PT.O.
2- From ihe followingparticularsextGcted{rom the bookso{
Mr. Gandhi,prepareTradingand Proltt and Loss Account
for the year 2004-05 and BaianceSheetas on 31't
Mafch 2005 after makingnecessaryadjustments.
Bs.
Mr Gandhi's Capital a/c 54,050
Stockas on 1-4-2004 23,400
Sales 1,44,a00
SalesReturn 4,300
Purchases 1.21,550
.
PurchasesReturn 2,900
CarriageInwards 9,300
Rent 2,850
Cash with TradersBank Ltd. 4,000
DiscouniReceived 1,495
Inveslments as on 1-4-2004
{50,6) 2,500
Furniture(ason 1-4-2004) 900
'
DiscountAliowed 3,770
Genehl Exp€nses 1,960
Salaries 4,650
SundryDebtors 12,000
Sundry Crediiors 7,400

MS4o
Loan from Dena Bank @ 72% 10,000
Inieresi paid 450
Printing and Stationery 1,700
5,600
InteresiReceived 725
Audii Fees 350
FrreInsurancePremium 300
TravellingExpenses 1,165
Postageand Telegrams 435
Cashon hand 190
Deposits@ 10% as on 1-4-2004(Dr) 15,000
Drawjngs 5,000

Adjustments :

(a) Value of stocksas on 31-3'2005 is Rs. 39,300.


This includesgoods returned by customerson
31-3-2005to the valueof Rs. 1,500 of which no
entrv has beenpassedin the bool€.

{b) Purchas€s includefurniturepurchased


on 1-1-2005
for Rs. 1,000.

{c) Depreciaiionis to be p}ovidedon furniture@ 13%

M54@ P.T.O.
(dl The Loan Account lrom Dena Bank in the books of
Mr Gandhi appearsas foliows I

Amount
Date Particulars ranrculars
Rs

31-3-05 To Balancec,/d 10,000 1-4-04 By Balanceb,/d 5,000

31 3-05 By Bank a,/c 5,000

10,000 10,000

lnterestpaid includedRs. 300 paid to Dena Bank.

(t Inter€st rec€iv€d representsRs. 100 lrom sundry


debtors and ihe balance on investm€nts and deposits-

\g/ Proude lor interesrpayable to Dena Bank and lor


interestreceivabl€on investmentgand deposits.

(h) Provide r€serve for doubtfr.:l debts at 5% on the


balanceunder sundry debtors. No reserveneeds to
be createdlor th€ deposits.

3. (a) What is a Jlexiblebudget ? How does it diJfer trom a


fixed budget and a rolling budget ? Erplain th€ utility
ol a llexiblebudgel.

(b) Explain th€ tbree impoltant control ratios to judge


the actual per{ormance with the budgeted
Performance.

MS4o
4. (a) "Return olr lnvestment is a primary ratio but it is noi
free from ambiguity." Discuss this stalement and
explain the various versions o{ ROI as used in

(b) What is the role of financial maoager \rilh regard to


dividends and divi&nd polica ?
(cl How does Depreciation act as a tax shleld ? Explain
with an exampie.
5 . From the following particulars relaijng to ABC Ltd.,
prepare a stalernent showins Changes in Working Capital
along with Funds Flow Statem€nt :
31st December
Particulars
2004 2005
CurrentAssets 1,35,000 1,27,200
lnvestments 15,000 21.,400
Land 9.000 9,000
Plant& Machinery 81,000 1,05,000
(AccumulatedDepreciation) (24,000) t26,000)
Patents 76,200 . 1 2 , 6 0 0
TotalAssets 2,32,2002,49,200
CurrentLiabilities 24,600 34,800
12DkDebentwes 43,400
1470Debentures 39,000
EquitgShareCapital 90,000 1,00,000
Reservesfor future loans on inlestmeris 6,000 3,600
Retainedearnings 68,200 71,800
Total Liabiiities& Capital 2,32,2002,49,200

Ms4o P.T.O.
Add,tlonol Infof'nation.
(i) A r€conciliatjonoI the balancesin retainedearnings
is as tollows :

Beginning Balance 68,200


Net Income for c$rent yeat 3,000
AwaYd received frcm settlement of
patent inlringement case
15,600
Dividends paid {15,000)
Ending balance 71,800

(i') Net income of the current year 2005 includes a loss


oI Rs. 4,800 on ihe sale ot a -palt of plant. The
plant was of the value of Rs. 19,000 at the
beginning of tie yeat accum\rlated deprecation
being Rs. 6,000.

{iij) Investments of Rs. 15,000 was sold during the year


at a loss. The loss was charged io the reservefor
future losseson inveslmentsand did nol appeal on
the Income Statement.

{iv) During the cLrffent year 12a/odebenfures were called


fot rcAemption. Most of th€m were retunded through
tbe issuaDceol new 74o/odebenturesand the rest
werererred Jorcash
(vl The equlty shares were issued in exchange.of
machinerv. The resi of the plant and machinery
were purchasedfor cash,

M54@
6. The trading resr ts of Ashoka Ltd. fof the fiFt year ol
businesswhich ended on 31st March, 2004 are as
Iollows r

Bs.
Sales(at Rs. 40 per unitl 32,00,000
LessMateyials 12,00,000
lnbour 4,80,000
Vatable overheads 2,40,000
Fixed overheads 5,00,000 24,20,000
Pro{ii 7,80,000

Duing Il',e year the factory has been working at 50o/o


capacity. The marketing manager has estimat€d thal the
qt/aniity so)d could be doubled dur;ng 2004-05, il ihe
selling price was reduced to Rs. 35 per unit. No change
is anticipated in unit va/iable cost but cefiain
adminisirative changes to cope with additional volume of
work would incrense tix€d ovethead by Rs. 40,000_

You ore rcquired to :


(a) E\,nluaiethe marketingmanager'sproposal,and

O) Assuming ihe sllling price wns reduced as propos€d,


unii variable cost remaining as in 2003 - 04 and
Iixed overh€ad increased by Rs. 40,000, calculate
what quantitywould need to be sold in 2004 ,05 in
order'io yield a ptoiit Rs. 10,00,000.

MS4@ PTO-
7. (a) What do you understand by variance analysis ? Why
are the varjancescompur€d?
(b) XYZ Lid. which has opted standard costing,
furnishes lou the {ollowing jnformation :

Slandard

Material for 700 un s of


finished products 1000 kg
Price of materials . R€. 1 per kg
Actual

Output 2,10,000units
Openins stock Nil
Purchases 3,00,000 kg. Ior k. 2,70,000
Ciosing siock 20,000 kg

You are requled io calculate :

la) Direct Matenal Usage Variance


(b) Direct Materiai Price Varianc€

8. Wriie explanatorynoies on the following:

{a) Tradiig on equity


(b) 'Ftst in. 'Lasi
First out' vs. in, First out'
(c) Cash cycle afld Operating cycle
(d) Rights shares and Bonus shares

MS-4@) 1,000
MANAGEMENT PROGRAMME
Term-End Examination
December, 2OO5

MS-s@ : MANAGEMENT OF MACHTNES


AND MATERIALS

Time : 3 hours Maxinrm Mdrks : 100


. (Weishtose 70%)

Note : Section A has fiDe questionsthat cairg 20 marks


each. Attempt any three questions from this
Section- Sect'ion B is computsory ond carries
40 morks.

SECTION A

1. {a) Explain Product Design. How do€s Product Design


influenceProc€ssDesign ?

Ms-5@ P.T.O.
(b) Processingtimes (including set-up times) and due dates
for six jobs waiting to be proces*d at a work centre
are given in the following table :

ProcessingTime Due Date


Job
(Days) (Days)

2 7
't6
B 8
C 4 4

D 10 17

E 5 15

F 12 18

Determine :
(i) the sequence of jobs

(ii) the average flow time


(iii) tl"€ aueEge job lateness, and

(ivl. the averdge number of jobs at the centre, for


each oI these rul€s :

{a) SFt, and


(b) Earlier Du€ Date. 10+10

2. (a) Describe ihe product selection process with the


help of an €xample of a new small car.

Ms.5o
(b) A producer ot f€lt-tip pens has received a forecast
of a demand ol 30,000 pens for ihe coming
month from its marketing department. Fixed cosis of
Rs. 2,50,000 per month are aliocai€dto ihe felttip
operation,and variabiecostsare Rs. 5 p€r pen.
(i) Find the break'even quaniiv if p€ns sell fot
Rs. 15 each.
(ii) At what price must p€ns be sold to obtain a
monthly profit of Rs. 20,000, assumingthat
esiimaied d€mand materialises? 1O+1O

3. (a) You have been retained as a consultant for


(
establishm€ntoI a steel plant. Discussat least ten
important factors that 9ou consider are relerant for
Iocahnga steelpla']t. Give examples.

{b) Factory Buiit Homes, Inc. (FBH)purchasespaneutng


comlronents from a nearby Westem Mills for Rs. 5
per unit. It expectsto use about 4,000 units dunng
the coming year. FBH estimatesthai it costsRs. 30
to place an order and Rs. 1.50 per unit per year for
carrying and storage costs. Westem Mills can provide
FBH with immediate delivery of any r€asonable
quaniity.
{i) What is ihe most economical quantity for FBH

iii) Ho\r mang orders per year should be placed ?


{jji) Whai is the toial yearl_vcost associated with
ordering, carrying and purchasing the EOQ
10+10

MS-5@ P.T.O.
a. (a) Supplier selection and supplier relations are
consider€d imponant for the p}tchasing departhent
Should the quaLity assuranc€ department €ver
become involved in these is$es ? Why or why noi ?
(b) The v,,areho\rse manager at the Textile lmport
Ceoter eltinrates that the purchase o{ a
Rs. 7,00,000 forklift would save2 hours per shift of
labour time. The firm works 2 sh'fis per day and
300 days per year. If the labolr cost io the firm
(includingbenefits)is Rs.320 per hour, how long
would ii take lor the forklift io pay lor itsell ? 10+10

5. la) What is ihe cost ol quality ? Explain acceptance


sampling. Disc\rsssampling plans.
ib) The ume .equired io completeeach ol six jobs on
two machin€s are shown in the tabl€ below. Each job
must Jollo\x the same sequence, beginning wilh
machine A, and moving to $achine B-
Time (houts)
Job Machine A Machine B
30 80
B 720 100
c 50 90
D 20 60
E 90 30
F 110 10
(i) Find the s€quence that will minimise total
compleijontime.
ili) Coostruct a chart ol rhe resulting sequence and
find machine B's idle time. l0+1o

MS-5@
SECTION B

A project contains the following activities aLongwith th€ir


pred€cessors and times for completion,
Activity ImmediatePredecessor Tim€ (Days)
5
B 4
c 6
D B 2
E c 1
F C,D 7
G E 8
H F 4
I G,H 3
li) Drawthe networkdiagram.
(ii) Showthe earlystartand earlyfinishtimes.
(iii) Findth€ criticalpath. 8+6+6

7. Writeshortnoteson anytour of th€ following: 4x5=20


(a) LearningCurve
(b) ABC Analysis
(c) Robotics

{d) Cons m€r'sRisk


(e) Bath tub curve
(0 P|eveiiiveMaintenance

MS-5@ 1,000
MANAGEMENTPROGNAMME
Term-End Examination
December.2OO5

MS-6@: MARKETING
FORMANAGERS
Tlme: 3 haurs MaximumMqrks: 100
Weishtase 700/d
Nole I
(I) Attempt ony three guestionsliom SectlonA.
(iil Sectlon B is compulsory.
(iii) All questionscarry equal morks.

SEC?ION A

r. (a) Marketing success largelg d€pends on the effective


STP strategiesadopted by the lirm. Discuss the
above statement with t\ro examples of your choice,
(b) What would be suitable segmentation basis fot the
following and why ?
(i) Premium brand of after shave lotion

{ii) Rs. 1 lac amall car proposed by a leading lndian


cat manutafiu{€r

Me6o P.T.O.
2. (a) What is MarketingR€search? Discussits scopeand
major adlantagesthat Iirms derive by undertaking
this activity.
{b) Wbat are the cdteriamarketersrnustevaluatebefor€
seleciinga brdndname?

3- {a) Sal€srnanshjp is jn-builtwith everyattractivepacking.


Do you agrce with the above statement? Discuss
with the h€lp of suitableexamples.
(b) Bnefly discussthe Iactors ihat marketers shouid
consjder whjje deiermining ihe promotion mix {or
their offerings,giving suitableexamples.

4. Write noleson any thtee of fhe ldlotwtg :


(i) Hou,ard-ShethModel
(i0 Characteristicsolorganisationalconsumers
(iii) Routinisedresponsebehaviour
(iv) Functionsof Retailers
(v) Ad!"ntages ol cybef marketing

Ms6@
t
SECTION B

5. Sh-dythe casegivenbelowand an$r,erthe qkstions given


at tbe end.

Mr. Harish Jain, CEO oi Eneigetic Enterpdses,has


estabiishedthe firm for the manufactur€and marketingot
an innovativeproduct.The firm earneda repuiationof iis
product wjthjn two yeays of its inception and enjoyed
monopoly position in the mark€t lor its product. Now it
has a turnoveroi aboutRs. 80 crores.

Three yeats back, some firms entered the market


and offeredcheapsubstiiuteswhich were of betterquaiity.
This year, Mr. Harish Jain is wonied becauseabout 40%
of the markelsharehas alreadyb€en iaken away by the
new firms and he is not able to check this trend-

Mr. Jain has been looking atter both productionand


marketing functionsthough finarce is being looked afier
by a finance manager having a professionaldegree in
charteredaccountancy.Mr. Jain has recentlyloweredthe
price of his p/odutt to fight corDpetjtion,but even this
has not helped. He has now approachedyotl for adrice
to stabilisehis salesvolume.

Ms6@ P.T.O.
@uestions I
ia) What is tle orientation of Mr. Jain in seliing his
product?
(b) What environmentalfactoF have cau*d a worry to
Mr. Jain ?
(c) As a consultant,what strategieswill Vou suggestto
checkfurther tall in marketshare?

MS-6
o 1,000
MANAGEMENT PROGRAII4ME
Term-End Examination
December, 2O05

MS-7@: Old : INFORMATION


MANAGEMENT
AND COMPUTERS
Revised: INFORMATIONSYSTEMS
FORMANAGERS
Tine : 3 hours Maximum Marks , 100
(Weishtoge70Yo)

Note :
(i) There ore two sets ol question paperc i.e. Set 1 &
Set 2. get 1 is for those students uho hove
registered lor this course prior to July 2004.
Set 2 is Ior those students uho haue registeredfor
this course lron July 2004 onwatds.

lii) There are two sectionsin each set Attempt any


three questionsJrom Section A ot' edch set.
Section B is compulsory.
(iiil A total ol tioe questions hooe to be attempted.
All questions corrg equal marks

Ms-7@ P.T.O.
Ser I (Old Coutse)
(lnformation Manag€ment and Comput€ls)
SECTION A

l. (a) "The lufure bblongs to knowledge workers.'


Explain. 10
(b) Explain the use ol Lotus 1-2-3 in modem business
grvingexarnples. 10

2. "Modem day management actMiies involve many


computer appllcations." Explain. Discuss the use of
computersin Human ResourceManagernent. 20

3 . Discussthe role of MIS at differentle'"€lsof management.


Suggesta suitable structure for MIS clearly giving the
of MIS.
desirablecharacteristics 20

4. ExplainDecisiontablesand Decisiontrees.Der,elopa flor


chart for someactivitythat you are lamiliar with. 20

5. Using graphicalmeihod, find out the minimum tota' cost


for ihe following 20
M i n i m i sze= 3 x 1 + 5 x 2
(in Rs.)
Subjectto

\+x2=200
xls80
xz> 60
xl,x2>0

Ms7@
SECNON B
6. Rolta Co. has three planis X, Y and Z, which can supply
to the distributorslocatedat A, B, C, D and E. Monthly
plant capacitiesare 80,50 and 90 urfts respectively.
Monthlgrequirements of distributors
are 40, 40, 50, 40
and 80 unitsr€speciively. cosric given
Unii rransportarion
below in Rs.

Find out an optimaldrsrribution


Ior rh€ companyin
ord€rto minimisethe totaltransportation
cost.

To
From
B c D E
X 5 8 6 6 3
4 7 7 6 6
z 8 4 6 6 3

7. Five different machines can do any oI the five required


jobs. with different profiis resulringlrom each assignmenr
as shown below. Find out maximum profit through optimal
assignment. 20

Machine
Job
B c D E
I 30 40 2a 40
2 40 24 27 21 36
3 40 32 33 30 35
4 25 38 40 36 36
5 29 62 41 34 39
r Ms-7o P.T.O.
(Intormatioo Systems for Managets)
SECTION A

1. (a) Discusslniernet and its applicationin buslness How


does computer help us in decision making ? Give
examples. 10

(b) Explain diflereni hardware components of a


computer.What ar€ varioustypes ot computers? l0

2 . Discuss the importance of MIS in modem organisations


What are computer viruses and how can they be
Prevented ? 20

3. (a) What are the various phases of a sofi'r,/are


developmentcycle ? Explain. 10

(b) Discuss applications ot computers in Financial


Managementfunction.Give examples 10

4. Explain the processof SystemsAnalvsisand qesign What


are the main requirem?naslor implementai,on and
maintenance of the designedsystem ? 20

(a) Give the f€aturesof VisualBasicand its applications


in industry. 10
(b) Why has JAVA become so popular ? What are its
leatur€s? Give its application 10

MS-7o
SECTION B

6. Write descriptivenotes on :
(a) Neural Nehrorks
(b) Artificiallntelligence
(c) Daia Warehoirsingand Data Mjning
(d) Value and Cosl ot lnlormation

?. (a) Explain ERP, CRM, SCM and BPR. How bave they
helped organisaiions? 10

{b) What are the main differencesbetween DSS, MIS


and EIS ? cive lew examples. 10
,/

Ms-7@ 1,000
MANAGEMENT PROGRAMME
Term-End Examination
December, 2OQ5

ANALYSISFOR
M$€@ : OUANTITATTVE
MANAGERIALAPPLICATIONS

Time: 3 hours Maxinum Marks: 100


(Weightase70o/o)

(i) Section A hos 9ix questions, each ctlrrying


15 marks. Attempt ong four.

(ii) Section B has two questions. Section B is


compulsory and carries 40 marks.

(ij, Stdttsftco/ tobles may be supplied on demand.

MS-B@ P.r.o.
SECTION A

1. End the qtrartiledeviationtrom the Jollowingdata :

Class Inlerval Frequ€ncy

0-15 8

15-30 26
30,45 30
45
60-75 '20

75-90 77

90- 105 4

ll 2 perceni of the books bound at a cefain bindery have


defective bindings, us€ the Poisson approximation io tbe
binomial distrjbution to determine ihe probabilitg that five
book oI 400 book bound by this bindery, will have
defeclivebindrngs.

3 . Gve some situations where ihe exponentja' distribution


has been found useful. A highway petrol purnp cao serve
on an average 15 cars per hour. What is the probability
tl]€t lor a Wftic.]lar car, fhe time taken will b9 less than
3 minutes ?

MS,e@
The coef{icient of rank correlation of the mark obtained
by 10 sfudenis in hro particuiar subjects are found to be
0 5. lt was latet discovered th;t the diff€r€nc€ in ranl.s in
two subjectsobiained by one student was wrongly takeD as
3 ihsiead of 7. What should be the co{tect value oJ the
coefficient of Rank correlaiion ?

5 . An oil cornpany claims lhat less than 20 percent of all car


owners have not tried its gasoline. Test this claim at 0 01
leuel of significance, if a Endom check reveals that 22 of
200 car ownershavrznot tried the oii company'sgasoline.

6. Write short noteson any ttrre€ of the lollowing :


(a) Sequenceand Sertes
(b) Censusand Sample
(c) Criterionof Optjmism

{d) Power CuN€


(e) Useof Autoconelation
in identilyingtime seies

'Ms-8@ P.T.O.
SECTION B

?. In 16 one hour test runs,the gasolineconsumption of a


heaw d\rly engite aveftged 16 4 gallons{,ith a standard
deviationo{ 2 1 gallons.Test the clalm fhat the averag€
gasolineconsumptionof this engine is 120 gallonsper
hour.

8. In a surveyoi populaiion, it was fotmd that 40% of the


maiepopulation work
wereil)it€ratell th€ irendcontinues.
otd the probabiliiy that out of a random sample of
2,00,000malepopulaiion,ihe numb€fol illiterates winbe
(i) iessthao 79,500 (ii)morcthan 80,500.

Ms-8o 1,000
MANAGEMENTPROGBAMME
Term-End Examination
December,2005

Ms-9 @ : MANAGERTAL
ECoNoMtcs

Tihe : 3 hourc Maxiatm Morks: 70O


(Weightose70%)

Note : This p{rper consistsof thrce SectionsA, B ond C.


&ction A is to be.dttempted by studentsregistere.d
for MS-9 prior to Jdnuary,2004 i.e. upto Jul9,
2003. SectionB is to be attemptedby students
registercdIor MS-9 Jrom January, 2004 onudrds.
Attempt an! three questionsJrcm Se.tionA or B
d€p€nding upon gour ddte oJ rcgistrotion.
Seclion C is conpul^ory for all. Section C
comprisesof 40 marks while SectionsA and B
comprise 5 qrestions oJ 20 marks each-

Ms-so P.T.O.
SECTIONA
l. Whai are the impotiant characteristics ol perfeci
compeiiiion ? Explain (with the h€lp ol graph) the protit
maximisinE outpui under perlect competition in short run.

2. D€scrib€ the followjng in lerms of short-run costs i

(a) Total cost

(b) Average lixed cost

{c) Average variable cost

{d) Average iotal cost

{e) Marginalcost

3 . Explain ihe optimal prcduct mix in the case ol a joiDt


product firm.

4. Discuss Price and Output deteymination under perfect


coinpetition and monopolistic competition in the long run-

(a) Explain the Dscounting Pnnciple with examples.

(b) Distinguish bet\reen Demand function and Demand

MS.9@ 2
SECTION B

l. Piscuss, with suitable examples,how managerial econornics


!s an integral parl ot businessactivitg.

2. What are the major marketing approaches io demand


measurabenl ? Examine.

3 . Discuss in detall the optimal combination of inputs in the


long run.

4. Define cartel arrangement. Explain profit maximization


under cadel anangement,

Write shon noteson any thre€ of l}le following:

(a) Value Maximisaiion

{b) Produciion Possibilit Cu'!e

(c) ExponenttalSmoothing

(d) Income Elasticity of Demand

(e) Strategic Entty Baftiers

M se @ P.T.O.
gECTTON C

6. Stafe wheih€r th€ followlng statements ate ttue ot to'lse. ,


your answet.
Jus(iF,J 20

(l) Scale is a shori run concept

(it The slope of the short nm produciion function is


equal to average product oi variable input.

( i i i ) The average product and marginal product oI


variable inptrts are equal at ihe level o{ output that
corresponds to the infl€ction point of the short run
production lunctlon.

IM The long run totai cost curve is derived lrom the


firm's expansionpath.

In ger,eml, a firm should continue to hire addiiional


units of an input so long as the marginal revenue
product o{ the inp\rt is greaier thaD ihe marginal
resource cost of the inpul,

MS-9O
7. (a) Given ihe Jotlo\ringdata,calcrrateth€ price eiasticig
of demandrrhen pfice increasesfrom Rs. 3 00 per
unit io Rs.4 00 per unit. 10

Px ber unit) Qx

750

1250

2000

3250

4650

8000

(b) Considerthe {ollowingd€mandand suppivihctions :


Demand:200-2P
Strpply,20+4P
What are the equilibriumprice and quantitysold ?

Ms-s@ 1,O00
MANAGEMENT PROGRAMME
Term-End Examinalion
December, 2OO5

MS-10@: ORGANTSATIONAL
DESlcN.
DEVELOPMENT
AND CHANGE
Time : 3 hours MoximumMarks: 100
(Weishtose70%)

Note : There ore tuo SecttonsA ond B. Section A has


tuo sers.Set I is meontJor the sludentswhohaue
registercdJor MS-10prior to Julg 2004 i.e. upto
Januorg.2004. Sef 1l is meanllor the students
t ho houe registeredlor MS-10 for Julg, 2004
semesteronu)ords.Attempt any four questions
lrom Sectioh A. AII questionscarrg 75 morks
each. SectionB is compursory Jor oll, ond coffies
'
40 morks.

MS,loo P.T,O.
SECTION A
Set | (Pre-Revised)

l. Compare the struchne of Matrix with Mechanistic


organisations.How are they relevaDt ? Cite suitabl€
examples,

2- DescribeBureaucraiicand ScientificManagementtheories
Lvith relerence to organisingwork and its significancein
the presentscenario-

3 . Describe the relationship behveen strategies ol


organisationaldevelopmentand structue.

4. Ljescribe lhe skills required by a change agenr in rhe


organisationalcontext with rclerence lo technological

5. Wrile shorl noreson any thrce ol the following :


(i) Formal vs. Informal organisation structures
(ii) Alternativeforms of Work Restructuring
(iii) Force Iield analysis

{iv) Ofganisationvs. Institution


(v) Purpose of organisaiional analysis

MS-10
@
SECTION A
Set ll (RevGed)

l, Descrjbe in detail the importani factors which alf€ct


organisationdesign.How are they relatedto organisational
effectiveness
?

2. Describe Hybrid and Virtual organisational structlrres and


their merits and demerits.Cite examples.Where are such
structur€9 effective ? Discuss.

3- Describe the €merging trends in work organisations.

4. Discusshow interview method is useful as a diagnosiic tool.


Provide suitable examples {rom orgaiisational context.

Write .hon noteson any tbr€e of the ,ollowing:


(i) Managingresistanceto change
(ii) Role analysis
(iii) Ergonomics
(iv) Boundarylessorganisation

(v) 7 S model to understandorganisation

t\ls-l0 @ P.T,O
SECNON B

6 . Read the case carelully and answer the questions that


follow :

Ramesh Ambani was appointed as Operations


Planning Manager in the Surepleasure Airlines last
monih. Immediately aiter reporting to his new job,
Ramesh recognised that there were dilliculties with the
time control section. This section consistedol sixteen
J'unior cierks, eight senior clerks and a supervisor wfio
reported to him.

The basic function of time control section is to


maintain records of all rotatable parts used on a fleet of
eighty-Iive passenger aircraft. Records rellect the date on
which a part was installed or removed and iotal aircraft
Ilying lime. To mdintain identification,serialnumbersare
recorded for each controlled part. The Airport Authority
rules require strict control of parts having limited life.
Parts that have operated beyond their approved limit bave
to be removed and overhauled.

The problem that Ramesh noticed was that there


were numerous instanc€s of irnccurate records. The
recotds were in violation of Airport Authority regulations
and poor record-keeping resulted in excessive overhaul
costs. When Ramesh studied the situation, he observed
that the employees had formed an extremely strong

M s r o@
inlormal work-group. The members woujo accept or
rqecl new eiplogees into the qJork-group based on
factors completely unrelatedto the iob. Some outsiders
obseru€d that il a nerv employee's behaviour was nor ln
tune with ihe infofially determined norros, he or she had
little chance of s.uccesswiihin the group.

The group members anarged for coDtributory


dine-and-dance palty every week-end and had a great
time tog€lher. Those who did not like io particjpat€ and
socraiisewere ignoredand 'nd& ro {eet uncomlonablelo
the potnt that they would quii the organisaiioD. This
r€sulted in an annud tumover |ate of apptoximatelg 200
petcent. Ramesh realis€d ihat this probtem had to be
solved immediaiely or major problems could result for the
Surepleasure Airlines ln future.

Qa€$tiors
(a) What are the dgs{unctions of.infoymal orgarisarron
in the Surepleasure
Airlines ?
h) Could there be any bene{its of inJormal organisaiion
in this company?
(c) Whai $uggestions!^rouldyou give to the manag€ment
of tlts compang to cope wiih tbe inlorma)
organisation ?

MS-lo@ 1,000
MANAGEI'{ENT PROGRAMME
Term.End Examination
December.20O5

MS-llO: Pre-Revised
: CORPORATE
POLICIESAND PRACTICES
Revised: STRATEGIC
MANAGEMENT

Time : 3 hours MaximumMorksi 100


(Weightoge70Vd

Note : There ore three Sections A, B and C. Section A is


meant Jor the students ..lho hooe registered for
MS'tt : Corporoie Policies ond Practicesprior to
Januory 2005 i.e- upto July 2004. Section B is
mesnt lor the studenti who haw registercd Jor
MS17 : Strotegic Manogement Jrom Januory
2005 onwords,Attempt any three questionsJrom
Section A or B. AII questions carrg 20 matks
eoch. SectianC is colnpursory lor all, and carries
40 marks.

MS11@ P.T.O.
SECTION A
. (Pre-Revised Course)
(Corporate Policies and Practices)

1 . D€scribe the role of BODs in th€ ov€rau performance of


the company. Discuss Erious measures to improve the
effectivenessof BODS- 20

What do you understandby "Mega" environment ? Discuss


the differentconstituentsof th€ Mega environment. 20

3 . Explain Porter's framework to anaDse industiy structure.


How does it help in d€terminingthe attractiveness
of an
industry ? 20

4. Distinguish bet'r,,e€n related and unrelated div€rsification.


Explain with the help of examples.

What are the causesof cor?orate decline ? Briefly discuss


strategiesto arresi the decline.

MS-l1o
SECTION B
(Revis€d Course)
(Strategic Managem€nt)

l- Explain the \,"rious steps involved in Strategic Management


process in a single businessfirm.

2 . Explain the following I 20


(a) Types of resources

{b) Strateglc imporiance ol resources

3. Discuss the importance of differeniiation .strategg in the


pr€s€nt compeiitive. environment. Explain taking into
consicleration its advantagesand disadvantages. 20

4. Every strategic alliance incurs certain costs and comes with


a s€t of risks- Explain any five costs^isk of entering into
a strategic alliance.

Corporate culiure plays an important role in the successof


an organization. Explain giving suitable example.

M S - 1@
1 P.T.O.
SECTION C

6, Read the case carefully, analyze it and ans,wer the


questions that foflow : 40
NEECO IIMIT.ED

At the end of the recent fivejear plan, it was


estimated that there would be a considinble demand in
the manufacturing capacitg of power transformers in the
country. It was further projected that the gap between
' demand and the manufacturing capacity would be even
larger in the subsequentplans. Thus, anticipating th€
country'sdemand in future, Ne€co Limited decidedto set
up a new unit for manufacturing transformers- This was
in addition to the manulacturing capacitg already built up
at one of the existingfactories.

Formal Planning Process

Formal planning was introduced jn ihe very filst year


of the commencement of activities at tbe new rmit, The
planningprocessat Neeco Limited includedthe settingup
of broad objectivesand the preparationof the three yeat
fotecast. In a letter addressedlo.departmental heads, the
General Manager, Mr. S.K. Patel said, "The tjme has
come to put down on paper the objectives and goals ol
our organisation and to develop a proper framework
whereby we can take a more systematic look at the future
we are heading for. This, I believe,can be achievedby

M S - l 1@
iNoh,erient and cooperalion ol all ihe departmenlal
heads in putting into practice a cultufe of planning."

The responsibility for developing a three year plan


\ras entrusted to th€ planning cell, which reported dir€ctly
to the Generdl Manager. There was a separate

I controller's oflice which looked after budget preparation


I and the subsequentmonitoring of actual perlormance.
I
Goal Setting Proc€ss

Planning began each y€ar in the month of April wiih


the €stablishment of goals by the top management. The
top managementgroup consistedof the GeneralManager
and his variousIunctionalh€ads.The goals w€re set both
in qualitative as w€ll as quantitative terms. The
quantitative goals were in tenns of growth in sal?s and
profits- Com$€nting on lh€ quantitative goals, Mr. Patel
observ?d that : "ProJit is th€ primary goal".

The planning cell, after collectingdl the infotmation,


consolidated and integrated the data and prepared
op€taLingresull<and ca-h llow proiectionsfor rhe three

MS,l1o P.T.O
Exhiblt-l
NEECO LIIfITED

Ia) A Tempotul Oueniea of the plannjng process

Date Steps in Planning


1stWeekof April Planning call sends felevant
Iormats to departm€nial
heads, with deiailed
explanations,
4th Week of April By the fouth week, the various
departmentssend th€ lormats
back to the planning cell, duly
filled.
lsr Weekof Mag Planning oflicer compiles all
the data and puts it in an
integrated form
2"d Weekol May The tirst draft ol the plan is
discussed in a management
group meeting, wherein
several suggestions emerge
and the &aft is thoroughv
revised by resoleing ihe
conflicting objectives of
various deFartments.
The final dmft of the plan is
readgand is sent to corporate
headquarters,

MS-l1o
(b) R€sponsibiJitytor Plan Preporotion

Segment oI the Plan ResponsibiliB'


Sale PIan (Order Book Position) Manager{Marketing)

Produciion Plan Manager (Production)


Materials Plan Materials Manager
Manporrrer Plan Personnel Manager
Township DevelopmentPIan Manager(Projects)
Integration ol Various
Segmentsof Plan and
Final Plan Document Planning Olficer

Be{ore finalisins the plans as prepared by the various


lunctional/departmentheads, a detailed discussionwas
held. Durjng these discussions,,"?rious departmental
heads explained the basis o{ their res!,ective plans. The
final plan was arrived at afier resolving the conflicting
objectivesof variousdepartments.

The Ptanning-Budgetirg Linkage

Immediately aft€r the plan was approved b9 the


ManagemeDt C,roW, the process o{ preparing a deiailed
budget for the next year was initiated. The three year
plan set otrt the broad objectives for the first year and
projections lor the next txo yeays. A detailed exercise
had alr€ady been conducted in preparing the broad
objectiv€s,which servedas a basisfor th€ preparationof

MS,ll @ P.T.O.
a detailed budget for the ensuing year. Thus the first year
of the plan document, with necessary mcldifications,
became the budget for the next year. The three year plan
document and the budget w€re, therefore, -closely
inter-related. lt was said that the preparation of a three
year plan was, in a way, a process of creating an
oiganisational climate for a rigorous and time'consuming
Ptocess of budgeting.

R€actlons of Execuiives

A tew executivesfrom the produclion depariment had


the following comments to make with regard to th€
formai plan : "The forms are lime-consuming and
tircsome. Here, when w€ are already ov€rbudened with
our daily roltine, who has got time to fill in figures in
these lengiby forrns. We are more worrjed about
day-to-day problems, rather than about the 3rd year from

Executives from other departments commented :


"Planning has led to greater participation in the
managem€nt process and thus, has created a culture of
rnanagemenlby participation.The involvementoI all of
us in the planning process has given us an opportunity to
take a wider p€rspective and has broadened our horizon.
Some of us now undelstandeach others ptlrblems in a
better way. This has paved the way jor bettet mutual
coordination."

MS-l1
o I
"Some of us now leel more confident of taking up
higher responsibiliti€sin tulure. Planning has helped to
groom lead€rc. lt has tumed us from technocrais to
managers and has prevenled us {rom becom:ng
bureaucrats.''

"Figures by themselves are not very impodani.


Figures without unde/standing the prcces9 ol how io
obtain them, are irrelevant. What is importanl is the
process oI plannlng, rather than filling th€ ftgures in
prescribed formats. This process had led to greater
managementparticipationin shapingthe future. Planning
has become a frame of mind and a way oI thinking. It
has become part o{ our managemeni process,"

Que$rions :
(a) Evaluate th€ lormal planning process at Neeco
Limited.

{b) Critically examine the reactions oI ihe various


exec\rtives as statgd in the case.

{c) Discuss the imporiance ot a three year plan.

M S . 1@
l 1.000
t*"'@-l
MANAGEMENT PROGRAMME
Term-End Examination
December,2OO5

MS-21@: SOCIALPROCESSESAND
BEHAVIOURALISSUES
Time : 3 hours Moximum Morks : 100
(Weishtsge70o/a)

(i) There are tuo SectionsA ond B.

Section A hos two sets: Set I is meant lor the


sfudents l,rho hooe registered lor MS21 prior to
Januaty, 2004, i.e. upto July, 2003. Set II is meont
Jot the students who ha)e registeredlor MS-21
Jron Jonuary, 2004 onwsrds.

(iii) Attempt ony totn questions lrom Section A. A1l


questions carry 15 ntarks each.

liu) Section B is co','rtulsgry for all and carries


40 marks.

MS-21
@ P,T.O.
SECTION A
S€t I
(Pre-Revised)

l. D€scribethe evolution from an agradan to an industrial


envronmelr and how L alfecteorhe work environinenr.

2- WhV is it imlro(ant to $derstand human behaviour tn


wofk enviro;ment ? Describe the factors which affect
indNidualditferences.Cile *rirdbtpexamptes.

3. Describethe processol perception.Why is it r€levantin


the organisational€nvironment and functioning ? Cite
suitableexamples

Explain transactional analysis and how ii helps in


understandingInterpersonalstyl€. What are the pitfalls of
this process if used in isolation in the organisational
context ?

5 . Wfiie short notes on any ttrree of the followtrrgl


(a) Power vs. Authority
(b) ContingencyControl Model
(c) Paii€rns of Communjcation
{d) lTpoadnce ot feeduck
(e) Schein'sSoc'alisationModel

MS-21o
SECTION A
Set ll
(Revised)

l. Describ€any two theoriesof motivationand their relevance


in the modern work context. 20

2. Identify the difference between a sroup and a team.


Explain the importanceoI teams in an organisationand
suggest suitable plan to build effective ieams 20

3. D€scribe different channels ol communicationand the


importancefor enhancingHR functioning.Cite examples. 20

4. llir'ngy''1'' 6"*n"n dn emotronanc p.nperdment Why n


iL inporant lo underrdndenorioncat work ? Drscu>:. 20

5. Write short noies on any rhree of the following : 20


(a) Valuesand ethics
(b) Cosnitive approachto leaming
(c) Imporianceof behaviourmodificaron
(d) Crossculturaltraining
(el Importanceof understandingpersonal'ty

MS.21@ P.T.O
SECTIONB

6. Readth€ followingcasecarelullyandanswerthe questions


given ai the end i

Sudhakar. the technical head at Moni. El€cironics


Ltd.. was happy that he was able to fill up the i\ro
vacanciesin the sysiemsdepartmentby selectingAjay and
Ganesh. Ajay was a M.Sc. (Electronics)degree holder
along with 6 months of industrialexperience.Ganeshwas
a fresher, wiih first class degree (in elecironics)
engineering. Both Ajay and canesh w€r€ intelligent,
hard working and usually were able to complete the
projectsassignedto ihem.

After a f€w months. at Mont. Sudhakarr€alisedthat


Ganesh always perceived himself to b€ more qualified
ihan Ajay, becaus€ol his engine€ringbackgroundcoupl€d
with his betier communication skills. Once h€ had
o\erhpa"d Cdnp\h raking rhe fu l credii lor overseerngaF
overseas project completed in an efficient manner.
Sudhakarspoke to their (Ajay'sand canesh's)immediat€
superior Naveen, who confirmed that completion of the
said project was a combined effori, wiLh Einer more
inputs hom Ajay's side. He also told Naveen to have a
talk with Ganeshand rectify communicationgaps, if any.

M S2 1@
t-

The talk seemedto help Ganeshwho realisedthat if


he had to come up, he had to itanslom his image for
ihe good and iry to mould himselfintc a sirategicthinker.
Ganesh honed his skills and staried attending marketjng
and strategicplanning classesto upgrade his knowledge.
The classesopened his eyes on bow lo make assltmptions
on projeci sales,r€venues,costs and profits. Very ofien
he was seen voluntarilyhelping his superior to study the
company data and analyse it. Seeing the change in
Ganesh, Sudhalsr was hapDy. Observing Ganeshs
motivat'on, at the €nd oi the probaiion period, he was
olfered a conlirmed job as an assistant manager.
Meanwhile Ajay was offered the position ot a database
analyst, after his probation period was completed
succe\(lullv.

- Sudhakar had presumed that both Ajag and C,anesh


wolrld be happy on being conlirmed and promoted. One
day le happened ta meet the Humar Resources head
during an informal meeting of the various heads of
depatttnenis to discuss about the managerial problems
they each grappl€ with. The HR head was speakingto
Sudhakar,"l was satisfiedand feeling quite competent as
a HR person, when somethlng like this has ro occur."
"Why ? What happened ?", Sudhakar enquired. "li is
concerning yolrr boy Ganesh. Based on his sup€riors
feedbackand your observationof his area ol competence,

MS-21
@ P T.O,
he \ras offered a conlirmed job of an assistanimanager.
But yesterdaV he came to meel me and said'l feel I am
mv own enemy. May be il would be bertp; jl I go ba.k
io fixing ihings becauseI was quii€ comforiablewith that.
No\u I realisethat I have to l€arn on how to managethe
impressionsthat oihers have of my role'. ' On further
questioning the HR head he realised that quit€ often
Ganeshbecame,by default,the technologicalfix-it person
on proie..s because of hir eyperience
"ngineenng
background.Also he had to learn to communicatewith
non iechnical people. Now the HR h€ad asked Sudhakar
whether he could suggesta remedy or solution to the

Qaestions :
(a) Drj you think Sudhakar was wrong in his initial
perception about Ganesh being more suitable for
carrying out manageial functions ?
(b) Which facet ot socialisationis relevani in ihe above
case ? Justify your answer.
(c) Can you sugsest way(s) to solve the problem
mentioned in th€ case ?

Ms-21@ r,000
MANAGEMEM PROGBAMME
Term-End Examination
Decernber. 2OO5

MS-22@ : HUMAN RESOURCE


DEVELOPMENT

Time : 3 hours MaximumMarks, 100


(Weishtase70%d

There ore three S€ctionsA, B ond C.

S€ction A is meont for students uho hoDe


registeredlor MS-22 prior to Jonuary 2005, i.e.
upto July 2004.

Section B is meant lor students who hoDe


rcgistercdlor MS-22 lron Januarg 2005 onaards.
Attempt ony thtee questionsfrcm Section A ar B
contingent on the regisration period. All questions
corry 20 marks esch.
Sect'ion C js compulsory Jor all, and corties
40 morks.

M S2 2 @ P.T.O
SECTION A
(Pre-Revised)

'The
1. nature and objectives of the organisation detemine
the role and type of HRD processes to be chosen ior
developmentalinterv€ntion.'Comment on this statement
and disclrssunderlying concepts with suitable examples 20

2 . What ar€ ihe salient causes of dis_satisfactionand


frustration ? How is frustraiion passed on to others ?
Explain with example, how a supenasor can cope \',lih the
lrustrationoI the employeeworking wiih him 20

3 . Identify the major areas of integration b'enueen the


dev€lopmentof HRD and IR jn an organisation-Describe
various pre'requisites for successfulHRD'OD approach to
tR, 20

4. Define and desc.ibeHRD cjimate.How are HRD climate


and organisational climaie relaied to each other ? 20

Wrlte shorr nole- on any three ol rhe lollowinq 20


(a) Task Analysis
(b) HRD Insiruments
(c) Approach and Avoidanc€ aspecis of management
(d) HRD in voluntaryorganisations
(e) DevelopmentalSuperviqion

MS22@
SECTION B
(Revised)

1. Define and discussthe conceptand obj€ctivesol coaching


and mentoring. Bnefly discusstheir applicationsin the
organizationalcontext. 20

2. Whai is Action R€search? How does it difler from OD ?


Discuss the important iactors io be consideredin the
development of internal selfrenewal lacilitators, with
suitableexamples. 20

3. Define and describe th€ objectivesand ad\,antagesof


MultisourceFeedbackand AssessmentFeedbackSystems
{MAFS). Discuss what are the indicators of an
organisations readin€ssto participatein MAFS. 20

4. Discussihe means ol managingtechnologicalchangesin


work organisation. Briefly describe the factors which
tacilitate developing the change mind-set. Explain with
suitableexamples. 20

5 . Wrire shon norpson any thr€e o{ rne following : 20


(a) DesigningRewardSystem
(b) OperationalisingHRD tor workers
(c) Career Transitionand Choices
(d) DevelopingBusinessEthics
(e) HorizonialRe-skilling

Ms-22o P.T.O
SECTION C

6. Read ihe following case carefuliy and answer th€ questions


given at the end.

Tha eleven workerc whose annual increments were


stopped made a representation to the management of
XYZ Limit€d ihat the action taken was not justified and
that they want€d to Lnow what was their fault. The
managementwhich acted upon the recommendationof
the department head concemed, Mr. Rog, felt guilty
because such an action was tak€n for the first time in the
history of the company.

XYZ Limiied was a large paper manulacturlng


company in South India. The major departrnents of the
Iactory were

1 Chemical processing : The raw material l,as mixed


with certainchemicalsfor makrngpulp.

2. Pulp department , Pulp was mixed with other


ingredientsaccordingto sp€cifications
for each order
ol paper.

3. Paper machin€ department : This was the heari of


the factory where processedpulp was fed into the
paper machines.Act first, a wet weak paper was
formed which was subsequentlydried and rolled.

MS-22@
4. Finishing department : The paper rolls were then
moved to the processing department where the
r€quiredcoating was given.

5. Grading,winding and packingdepartments.


6. Qiraliiy controi departm€nt.

Twenty eight workers worked in the paper machine


department in four groups - each group attending one
machine. The nature ol the work on each machine was
such that all the seven workers had to work in
cooperation. Because no indMdual iaikJ could be
spacilied, the group was made responsibl€ for the work
turned out by them. All the workers working in ibe papet
rnachine departm?nt had be€n with ihe compang for over
ien years.

The company did not have any incenti\re wage system


for any class of iis €mploy€es. They were all given
staight saliiries with normal annual increments. The
annual incrementswere sanctioned€ach year in a routine
way. Ii was the policy of th€ company thai the
increments should not be siopped unless the department
head concernedrecommendedsu(h an acrion.

Ml. Rog was placed in chargeol the pap€r machin€


departm€nt a Vear ago. Though Mr. Rog wae a
n€wcom€r in the organisation,he proved himselt to be a
very competentman. The managem€ntnoted that he was

MS-22@ P.T.O
very aggressiveand enthusiasticand that he know his job
weii At ihe end of ihe year when incrementswere due
to be sanctioned,he recamme$Aeltto the management
that ihe incrementsdue to eievenmen in his department
shoLrldbe stoDped,for, in his opinion thev were lazy and
inefficient The elevenmen concernedbelongedto all the
tour groups operaiing in the department..

The managemeni,though puzzledabout the action


recommendedby Mr. Rog, acted upon ii and sroppedihe
increments dLre to the eleven men concemed. The
managerDent were aware that sLtcban action was ihe firsi
ot its kind in the hisiory of the company. Most of the
employe€swere with ihe companyfor a fairly long p€riod
and there was never an instance ol strained relations
b€tweenthe managementand the employees.

Soon altpr the a( on was rokpn, the eleven


employees concemed mad€ a representation to the
r;aoagementrequesiingthem io iet them krow wnar q/as
wrong wjih their work as to wanant stoltping of their
increments.The managem€ntwer€ in a fix becausethey
did not have specific reasonsto give except Mr. Rog's
r€port in which he simply mentionedthat ihe etevenmen
concemed werc "lazy and ioefiicient".

MS22o
The management were naturally concerned about
the representationand therefore,they tried to ascertain
from Mr. Rog the detailedcircumstancesundef which
he recommended the sioppag€ of incr€ments.When
Mr, Rog could noi pin poini . the reasons. the
manag€mentsuspectedthat Mr. Rog's recommendation
was based on his "impressions" rather than on {acts.
They, therefore, advisedMr. Rog to mainiain a r€gisier
from then on noting ih€ detailsof day to day incidenisof
"lazy and in€fficient" workers and obiain the signatures
of the workers concerned.Mr. Rog was to make the final
appraisalof each worker in his departmenton the basis
of this regisier and recommendeach case giving specilic
reasonswhy incrementsshouldbe stopped.

Mr. Rog started mainiajning a register as suggested


by the manag€m€ntibut he'found it difficult to report
satislactorilyany case of lazinessor inefficiencyfor want
of specificreasons,

The managementw€r€ convincedthat their action o1


stopping increments o{ eleven men on the strength o{ Mr.
Rog's report was not a proper one. They realis€dthat no
similar action 'n futur€ would be taken bas€d on
inadequate information. But, they were wondering
whether the suggestionmade io Mr. Rog was the proper
course of action to prevent occurrence of similar
situations.

MS22@ P,T.O.
Quesaions :
(a) Identily and discussthe core issuein the case.
(b) Was management of th€ company jlrstified in
implementing the r€;ommendations of Mr. Rog, in
the absence of proper report ?
(c) How would you vi€w the action ol Mrr Rog, if you
were the M.D. of the company?
(d) Do you think the reward system instituted by the
company needs io be reviewed ? Substanhate your
ansrwerwith logic.

MS-22@ r,000
MANAGEMENT PROGRAMME
Term-End Examination
December.2O05

MS-23@: HUMANRESOURCE
PLANNING

Time : 3 hours MaximumMorks, 100


(Weightage700/o)

Note :

(i) There ore two SectionsA ond B.

(iil Section A hos twa Sets I ond II.

(iii) Set I is meont Jor the studentsuho hooe rcgistered


lor MS-23 prior to Julg 2004 i.e. upto Jonuary,
2004.
(iD) Set lI is mednt lor the studentswho hoLv registered
lor MS-23 fron July 2004 onwards.

Attempt onv ,hree questionsJrcm Section A. AII


questions corrg 20 marks each.

bi) Section B is compulsory Jot dll ond carries


40 marks.

MS-23@ P.T.O.
.
SECTION A
Set I
(pre-Revised)

1. ',There
is a definite link betweeneducanonand economic
groMh, and lack of adequate skilled manpower impedes
grov,/th-'To which one of the approachesof HRP would
you ascribe this stat€ment ? Discuss the steps involved in,
and limitationsof this approach. 20

2. Whal are lhe basic requisitesof an elfective Human


Resource Information System {HRIS) ? How is
compuierised HRIS advaniageous for an organisation at
diflerent levels ? Discusswith suitable examples. 20

3 . How would you formulate ihe aims and objeciiv€sof a


training programme ? Explain with an example. How are
training needs identified for different levels ot employ€es in
an organisaiion? Discuss. 20
4 . Descnbe the basic assumptionsof Job Elaluation. Explain,
with example, the major areas where job evaluation can be
applied successfully.

Wnle shorl noreson anv trr.ee of fie following 20


(i) Qualitative methods for demand forecasting
(ii) lssuesand problemsof manag€rialselection
(iii) Measurementol Human ResourceCost
{iv) Dimensionsof HR Planning
(v) Peer group appEisal

M S2 3 @
SECTION A
Set ll
(R€vised)

l. Is Human Resource Planning a win-win plocess Jor


employersand ejnployeesboth ? Explain with reasons.
Discussihe importantissuesin demandforecasting. 20

2. Define and describe the concept of Role. Are role


expectations changing in th€ mod€rn woid of wo* ?
txplain wirh rhe belp ot dirieren!aDp)odchesto changing
roles in the organizational context.

3. DescriL€ competmcy aDproach. Discuss how it is used by


the organisation. Briefly discuss various approaches to
colnpeaencymapptng, 20

4. What are the pre-requisites for int/oducing Human


Resource Informatjon System (HRIS) in an organisation ?
How is HRIS ad\,antageousov€r manual system ? DscLlss
how it can be made more €ftecti!€ and usetuJ. 20

5. Wtite short notes on any lhree of the following : 20


(i) Career Planning
(ii) Coping wjth dislocation
(iii) Executive Searcb
(iv) Steps in Job Analysis Process
(vl Induction Tlainins

MS-23@ P.T.O.
sEcrtoNc
6. rhe que(tionsgiven
Read rhe cas€g)uenbelow and dnr,,,/?r
ai the end ot the case. 40

Pearl EngineerjngCompany was a large


heaw-engineeringunit. It altached greai importance to
the recruiiment and iraining of its senior supeFisors.
Apart lrom s€lecting them from v.rithin ihe organization,
the company recuited, every.alfernaie y€ar, about ten
yo'$g engjneeringgraduatesand oflered them training
for a period of iwo years, before they were appointedas
senior supervisors. Such appointments wete made to
aboui 40 per c€nt ol the vacanci€s of senior supervisors
thai occurred in ihe organizaiion.fils was considered
necessary by management as a plarned programme ol
impa ing vitalily to the organization. Besides, many of
ihe old-timers, who had risen from the ranks, did not
possess the necessary academic background wiih the
r€sult that they could noi keep pace with ihe
technological changes. Maoagement also believed that in
the rapldly changing conditions of iodustry. a bank of
technically competent supervisors played a pivotal role,
besi&s senang as a pool from which to select luture
departmentalmanagers.

Engine€ring Sraduates were selected from amongst


those who applied in r€sponse to an all-lndia
advertisement. For the selection of one engin€er, on an

MS-23o
averager eight applicanis werel call€d for interview- A
seLection conmittee consisting o{ ilie Generd Manager,
th€ Produciion Manager,the PersonnelManager and the
Training Officer intervi€wedand selectedthe candidates.
The selectign interview was preceded by a written t€st
and only those who secured40 per cent marks qualified
for interview.

The eniline€rs thus selected had to undergo a


two year intensive th€oretical and practical training. A
well-staffedand €quipped Tralning Institute was directlg
responsible lor the trainjng of the graduate engineers,
becdes lraining lradp apprenli(es and operaiivesrequired
by the company. L€ctures on theoretical subjectswere
given at the Trainjng lnstitute and practical training was
imparted in all the works departmentsunder the guidance
ot qualified and experienc€d instructors. A few lectures by
senior officers o{ the company were also arranged to
acquaint them Mth the company policies on diff€rent
matters. During the last quarter ol their two,year training
programme rhey were deplrted to work iulltime ro
familiarizethemselveswith the conditionsin departments
where lhey were to b? absorbedeventually.

On successful completion o{ training, the graduate


engineerswere ofler€d appointments,dependingon their
performance and aptitude as revealed during training. On

MS-2so P.T.O.
placement in the work departm€nts, however, most of
them faced some difficulty or ihe other.

Accordirg to management, some oJ the heads of


departm€nts, who were themselves not qualified
engjneers, did not have sufficient confidence in tlese
younger men. They preferred the subordinates who came
up from the ranks to hold positions of responsibilitg. A
few discredited them saying that ii would take years
before these youngsters could pick up the job. Besides,
some of the employees,whose promoiional opportunities
werc adwrsely atfected by the placefient ol gaduate
engineerc, tried iheir best to run down the latter as a
class, sometimes working on the group feelings oI the
workers. Some oI the super/lsors who were not graduate
engineers also spoke derisively of them as "the blue-eyed
boys" of the organization. Management knew that many
of the graduate€ngineerswere not utilized accordingto
their capacity or training, no! was any atiempt made to
test or develop their poteniialities. They aiso knew that
many of the graduate engjneers wefe, therefore,
dissatisliedwith their work life. Some oI them who did
not get equal promotional opportwities as their
colleagues placed in other depadments, were Iooking for
better jobs ebe\\;bere.

On the other hand, accordingto management,the


yormg graduate engineers were th€mseives partly

MS.23@
responsible for th€ hostile attitude of others in the
organization.Some of them failed to appreciate ihat a
newcom€r invited hostility in the beginning and it took
time before he was accepted as a member of the
work-group. They did not realizethat they would be tully
productive only alter gaining about five io seven yearc'
experiencein the organization.A few thousht that they
belonged to a superior cadre and threw their weight
around. TheV did not bothef io understand and appr€ciate
th€ problems of the rank-andjile of employees who
worked under them.

In spite of ihese drawback, the General Manager of


the company telt that thesemen were a set of disciplined
supervisors.They had a senseof pride in their profession,
and with the extensive training they had received, they
would be able to take up any responsibleposition in ihe
organizationin course ol time,

The General Manager could not allow the situation to


continue especiallywhen it was a difficult and costly
processto recruii and train young engineeringgraduates
ot lhe requisile rype and calibre. He knew rhat $e
prosperity of the company, io a large extent, d€pended
on these young men. In addition, a large number of
lucrative employment opportunities were availabl€ to
these young engineers elsewhere and there was a

M S2 3 o P.T.O.
systematictaid on them, He, therefore, called a meeting
of all heads of depaftm€ntsto re\,lew the situation.

Questions t
(i) Identjfu th€ issuesrelatedto manpower planning as
evideni in the case.
(ii) Discussthe strategiesto iackle the percentage of
internal promotion at the organizationallevel.
(iii) Wbai type of additionaltraining programmesshould
be imparted lor dir€ct entrants?
(rv) Supposeyou are the head of the personneldivision.
What would be your suggestionsin the meeting.
which has been called by the General Manager ?

MS-23@ 1,000
MANAGEMENT PROGRAMME
Term-End Examination_
Decernber, 2OO5

MS-24 O : PRE-REVISED:UNION-IIiIANAGEMENT
RELATIONS
REVISED: EMPLOYMENTTEUANOTS

Time : 3 hours MoximumMqtks: 100


(Weightose
70W

(t) There ore two SectionsA dnd B.

(ti) Section A hos ,t o sels ' Sei I is meant for the


students LahohaDe r.gistered Jot MS'24 prior t'.t
Jonuorg, 2005 i.e. upLoJune, 2004 $et lI is meant
Jor the studentswho hduercgisteredfrcm Jonuqtg,
2005 and onuards.

(iiil Attempt atlg three questions Ircm Sectian A. All


questlons coffv 20 morks eoch.

/io) Section B is compulsory lor oll ond corries


40 morks.

MS24o P.T.O.
SECTIONA
S€t I
(Pre-Revised)

1 . Digcuss ih€ role oI staie and constitution in


union manag€ment relations. Discuss with suitable
20

2 . ldentily ihe factors r€sponsible for the formation of


white collar managerialunions. What kind of implications
are there lor the organization for such a process ? 20

3 . DeJine co[ective barga]ning. L\plain ihe bargainable issues


and types of bargaining with suitable examptes.

4. How do the cross-cultural aspects aff€ct the


unrcn'manage..(ent relations ? Cite suitable a\amples to
suppoft your answer. 20

5. Write short notes on any three ol the following r 20


(a) ILo
(b) Trade un'on recognition
(c) Interpersonalconflict
(d) Models in workers' pa(icipation in managemmt
(e) Negotiationskilts

MS24@
SECTION A
Ser ll

(Revis€d)

l. Discuss the salient foundations ol indusirial relations.


Describe in briei the influence oI th€ori€s and modets on
industrial relations practice. 20

2. Describe the chaGcteristic features of structure ol irade


unions. Outline the structure o{ trade unions in India. 2A

3 . Detine coll€ctive bargaining. Examine the unique features


of collective bargaining in Indian context.

Define industrial grievance. Explain various approaches ol


grieEnce resolution.In your opinion, which approach is
the most elfeciive and why ? 20

5- WriLeshon notes on any lhre€ of the (ollowing. 20


(a) Function of employers'organisation
(b) Negoliationskrlk

{c) Models of workers' participaiion in management


(d) Misconduct
(€) Adjudication

M924o P.T.O.
SECTION B

6. R€adrhe (dsecar€firllyand answerrhe questions


givenat
the end. 40

Geetha L"aboratories P vat€ Limited was established


by Mohan Ramnath in lc88 at Ch€nnai. He, a solt
spoken gentleman,was Ph.D. in chemjstry,who did not
believe in working under pressure. The company was a
small scal€ unit manu{acturing non-patented artimalarial
medicines.The company had 6 days per week working
making 26 working days per month and was running
smoothly. ln 1978, CITU suppoded union came into
existence. The industrial r€lalions started deter:orating
making it difficult for the company to sunnve. In 1988
Ramnath decid€d to enter into paftnership with three
other partners, Chandan Keshav, Bharat Pathak and
Veenu Ramachandran to overcome the dilliculiies faced
by him. The compang came to be kno',,,i as Geetha
Laboratories Limited. Even alter this the industrial
relaiionsdid not improve lill 1990 and it was during this
penod that 14 work€rs wer€ sack€d.In 1990 Ramnath
decided to sell his shares to Emission Pharmac€ulicals, a
multinational,ihough other partnerscontinued.Now, the
'company
was calledGerman Drug House (GDH). During
this period CITU withdrew support to ihe union and
Bhartiya Mazdoor Sangh {BMS) came into the picture. An
average increment of Rs. 225 was give$ to all !,,,orl€rs
and induslrialrelationsimprovedlo some extent

MS-24(>
IMPIA Pharmaceuticals Limited was another
non'patented antimalarial butk drug rnanufacturing giant
having units at Pune, Mysore, Hyderabad,Coimbatore
and Corporate office at Baroda. It wanted to have
nionopoly in ihe antimalanal drug manufacturing by

.laking ov€r GDH. bLrt before ldk'ng such a srep. they


wanted to assessthe intemal condition of the company.
Therefore, in January 1994 Vishal Shrivastav,a qualified
chartered accountant was inducted as Director by
purchasinga r€quisitenumber of sharesof the company.
In September, 1994 after IMPIA was convincedabout
the lavourableconditions ot cDH ii lormally took over
ihe company. At that time the manpower strength of the
plant was 210 in which 130 were workers and 80 were
executives and staff l.nembers. After taking over, IMPLA
made many changesand the major ones were ;

(i) They incr€asedthe salariesol executivesand stafl of


the unit to reduce the gap in th€ pay shuciure of the
executivesand staff of this unit and the:r other units.
(ii) They investedRs- 3-4 crores Ior upgradationof
the plant.
(iii) They shifted from 6 days working per we€k to 7
days working per week to improv€ the productivity
and enhancecost-elfectiveness
of the unit.

The shift from 6 days to 7 davs working without any


financial gains made workers resist the change. At this

MS24(t P.T.O.
juncture, Sumeet Joshi, Corporate Manager (lR)
intervenea and promised the ur'ork€rs that theg Qrr]uB be
paid for 30 days instead ol 26 days, but Ravi Shriman,
Director (Personnel) and Vishal Shri\rastav, CM
(Operations) refused io agree to this since they were noi
involvedwhen SumeetJoshi made the commitment.The
pfomise was not fulfilled which further complicated the
ploblems. The issueskept on Iingeringfor 6 months. No
decision could be taken because of the difference ol
opinion among s€nior executives.in June, 1995 the
workers gheraoed Vishal Shrivastav to pressuris€ the
managementto take the decision.They were successiul
to some extent as it led to the agreement of management
with workers that financialbenefitswould be given wiih
retrospeciive years making it one additional
.effect ol 4
year over and above 3 years of normal agreement
_They
were askedto give a notice of changewhich the workers
could not give iill December, 1995 because of
disagreemeniamong themsetves.Ii was felt at this point
ot time by Shrivastav that plani should have an Assistant
Manager (personnel)insteadof having PersonnelOfficer.
Ajjt Dubey, Assjstant Manager (perconne))was appoDreo
in October, 1995 but even this appointment took 3-4
months because of discord in opinions of Shrivastav and
Sbriman.

. In December, 1995 the workers gave a norice of


clnnge demarding an increase ot Rs. 2200 per month.

MS 24@
In January, 7996 a notice of change was g(,en oy
maDag€ment.ln February,1996 the n€gotiationsstarted
and continued till July, 1996. Shrivastav,Rajkumar, the
new Corporate Manager (lR), Ajit Dubey and Kishore
were to represent the management side and nine
menbers Of the unjon were to represent the Workers,
besidesV.D. Agrawal the GeneralSecreraryof BMS.

The first two rounds of meeting did not lead to any


outcome as none of ihe padies w€r€ readyto budge.This
made V.D. Agrawal withdraw as he was fed up with the
rigid stand of th€ union leaders.

The thjrd neetjng was held wjthout Agrawal


wherein the union leaderscame down to Rs. 1,200 from
Rs. 2,200 p.m. The minutes ol the rneeting werc jotted
down but the union teaders refused to sign. Taking
advdnlage of the occasion, Dubey and Sbri,"astav had a
seoet meeting with Agrawal in a hotel. Agrawal advised
the representatives of the management to maintain a low
key for a f€w months to crack down the workers'
aspirations as they had very high expectations.It was
observed by Dubey that there were perceptual differences
betweensenior and junior union leaders.Tal,jng cue from
this, Dubey adopted a policy of divide and rule and took
into confidenceDevilai, the senior union l€ader dnd had
a secret rDeeting wjth hjjn to explore ibe last setleDent
amount and apprisedhim that the managementcould go

MS.24o P.T.O.
only upto Rs.450. He also took Janak Singh, the junior
union leader into confidence and convinced him that
managementwas not going to berjd befor€ their demands
and as such ihe workers were going to be the uliimate
suf{ere{s. Besides ihis, Dubey sptead the messagethat no
wages would be given retrbspectively.

The next day meeting resumed in which union


r€pr€sentativescame down to Rs. 750 {becauseof the
pressurefrom the workers)beyond which they were not
ready to com€ down. Ii was decided tbat instead ot
having m€eting with all the members,only two members,
one senior unjon leader, D€vilal and one junjor union
leader, Janak Singh would slt in the negotiations.
Immediately a m€eting among Shrivastav. Rajkumar,
Devllal and Janak Singh was held and jl was resolved that
Rs. 575 average per month would be given for 4 years
retrospectively. An MOU was dralted by the legal
consultantat the corporateoftice and was duly signed bv
Shtivastav, Rajkumar, Dr-rbey and all the union
representatives. In the eveniog a dinn€r was hosted in
which all the negotiators were in\,iied. When the papers
v,/ere sent to R. Shnman, he obiected to the MOU on two
pomIs,

First, th€ other plants were having 30 days pav


system leading to less alerage pay per day and in
Chennai pfant it was to be given for 26 days leading io

MS24o
higher averageper day. Secondthe nilk allowanc€given
for ovedime at Ch€nnaiunit u'ashigher than other units.
it look Shri\rastavand Rajkutnartwo months to con\,'rnce
Shrirnan about the ag€ement and lhercatler,
impl€meDtingihe same.A toial amountof Rs. 14 to 15
lakhs was given to all ihe 160 woykerswithin a wesk as
afiears and the issuewds s€tiled.

Questtons :
(a) Was it right tor VD. Agya\ralto withdraw half way
during the negotiations?
h) ldentify the tactics used by managementin the
case.Ar€ thegjustified?
(c) Sho'rld Director (Personnel)hn!,e Iaised objections
after MOU (Memotandumof Understanding)was
signed? Give reasons.
{d) ln view of the informationgiven in the case,suggesi
tbe strategies that could have made IMPIA
Phatmaceuticals a morc progressiveorganisatjon-

MS,24@ r,000
Fsrs@-l
MANAGEMEM PROGBAMME
Term-End Examinalion
December,2OO5

. MS-25@: MANAGING CHANGElN


ORGANISATIONS

Time : 3 hours MoximumMorks: 100


(Weightage70%)

ti) Ihis popel conslafsol ttho SectionsA and B.

lii) Section A consistsol tuo sets.Set I is to be


attenlpted b9 students uho haue registercd Jor
MS-25 before Jonuory, 2004. Set ll is to be
attempted by studenls uho haDereglsteredlot
MS-25Jron Jonuary,2004 onwords.
(iii) Anstneran[ lour questionsfrom SectionA, eoch
corrying15 matks.
(iv) Section B Is compulsory t'or o1l and carries
40 morks.

Ms,25o P.T.O.
SECTION A

SET | (Pfe-Revised)

|. Discuss rie impotunce of Social and O'ganisationdl


culiures in bdnging about change, with suitable €xample9.

2 . Degcribe survey leedback as an jntervention in bringing


about change a$d its advantagesand disadvantages,citing
suiiableexamplec.

3 . Describe the meihod of role eliiciency to bring about


change in an organisation and discusg its significance ln
€mphasizing the relatlonship oI employe€ and employer.

4 - Discuss the key roles iNolved jn managing change. How


are the interaciions within the organizatio[ {acilitated ?

Wile shon notes on any t&re€ ol rhe lollowing :

(i) External '"s. Internal consultant

(ii) Manageras ag€nt of change

(iii) Processconsultatjon

{iv) Transformatjonalchange
(vl A model of implementation of change in an
organisation

Ms-25o 2
SECTION A
,

SET II (Revlsed)

1 . Discuss the pyocess of Leveraging Struciure in bringing


change in organisations. Expiain with suitable examples.

2 . Evaluate critically the impodance of Weisbord's Six-Box


Model {or diagnosis in an organisation,

3 . Discuss the concept of Total Quality Management and its


relevancein bringingchangpin dn organis<tion.

4 . Discuss how cultural change can be brought in an


organisation. E\plain with examples.

Write short noles on dnu thr€€ ol the following :

(i) Managingtransitioo

(iil Managing resbtance to change

(iii) Transactional vs. Transformaiional leader

(iv) Types of change

(v) Behaviour modelling as an intervention

MS.2s@ P . TO
. .
SECTION B

6. Read the following case carefullv and answer t}]e questions


given at th€ end :

In 1995 Ford Motor Company announced a major


reorganization called "Ford 2000." The idea,
champion€dby Chairman and CEO, Alex Trotman and
Vic€ Chairman Edward E. Hagenlacker,eliminatedmore
than a dozen engineering design centres aromd ihe ,xorld
and consolidatedthem inio only live - Iow in Dearbom,
Michigan, and one in Europe. The one in Europe was
responsiblefor creatingone basicdesignfor small cars lor
the world market and then making minor modifications
fo{ local markets. For example, the same template will be
used in Europe, South America, and Asia. The four
design centres in Dearbom will do the same for large
fronfwhe€l-drive cars, rear'wheel-drive cars, pick'up
trucks, and commercjalvehicles.The consolidationeflort
requires that nroie than tweniV-five thousand salaried
employees relo€ate or at least report to new managers.
Manufacturingand assemblywill still take place in plants
around the world.

The purpos€ is to integrate Ford's operalions around


the world and revolutionisethe wav it designsand builds
more than seventylines of cars and trucks, which it sells
in mofe than iwo hundred markets. Th€ goals are
reducedduplicaiionol €liort, increas€volume purchasing,

MS.2so
save more than $4 biilion per year, and double
profitability.All this lar a companyihat mad€ $3.8 billion
prolit from auromotive operations in 1q95. and $53
billion overall. Trotman continuesto have the support of
the Ford family, who still controls 40 percent ol the
voting stock in the company.

Part of the new plan is a top-secret strategic


document that outlines every new car and truck Ford will
design,produce,and sell around the world through 2003.
The plan calls for reducing the basic design platlorms
from 24 to 16 and increasingthe total number of models
by 50 percent, while saving billions of dollars. For
example, the new 1996 Taurus serves as the platform for
several other models, both in ihe United States and
around the world.

.ln sttucture, ihe new system is really a matrix. Rather


than workins in a functionalorganizationwith iraditional
hierarchiesand cenlraliTedde(r.ion making. emptoyeej
are assignedto a design centre, such as small cars, and
then to a group accordingto their specialities,such as
drive trains. Managers ihen mediate the disputes that
occur between lhe design c€ntres and the. specialiii€s.
Employees will have to change their ways of doing their
work as ihey design cars and trucks to fit global markets
rather than a single, relatively homogeneous one.
Management knows that employees feel a great deal of

MS-25@ P.T.O.
insecurity and uncertainty about the compaoy and their
jobs as they make the shift. Carrying the message to all
employees has been a constant job for Trotrnan and
Hagenlackersinc€ the original announcement,

Management also knows that Ford tried a similar


design integration with their ''World Car" in the late
1970s, which failed primarily due to turf battles among
designers and engineers. The cars that result€d were
yarely the cost savers Ford hoped for and were so dull in
iheir design that no one bought them. Trotman expects
different result this time becauseof the consolidationof
the design centres, the new organisationstrucrure,and
because advances in technology hav€ made the inner
working of cars so similar ihat only the outer, visible
portions of the cars need to be different to satisfy
regional tastes.

By mid 1996, however, the reorganisationwas not


gojng so well. The transitionhad left many employeesstill
wondering whom they worked for and with a feelingihat
everything was oui of conirol. The culprit seems to have
been a reorganizationof the reorganisation! Trotman
now plans to r€duce the number of d€sign centres from
five to ihree. People are moving and reporting
relationships are changing once again. Gtoup Vice
President Jacques A. Nasser, who may succeed Trotman
by 1998 or so, has promised$11 billion in savingsunder

MS-2s@
the new system. Some have claimed that lhe "new"
reorganisationreally puts ihings back ihe way they were
belore the first reorganisation.However, three design
centres is a lot Iewer than th€ dozens that existed L\e{orc.
But this second r€organlsation,before employeesreally
got settled into the first one, may have d€vastating
ellects. Suppliersand,employeesdo not know whom to
contart to get questions ans.rered or disputes resolved.
All they get on the phone is voice mail, since everyone
is in me€tlngstrying to work oui the new reorganisation.
Top managementclaim9that theseproblemsare inherent
trying to turn around such a bjg organisation that has
be€in relatively success{ulihrough the years. Theg say thai
th€ oruanisationneeds to evolveto meet their ambitious
goals and the comtretition.

Qrreslions ;
(a) Describe the changes in structure that Ford expects
from the Ford 2000.

(b) How do you a\plain ihe continuing problem that


employees arc having witb adapting to th€ new
structure ol Ford 2000 ?

(c) ls a mairix sttucture the proper shrcture for Ford


2000?

Ms-25o I,000
MANAGEMENT PROGRAMME
Term-End Examination
December, 2OO5

MS-26€) : ORGANISATIONAL
DYNAMTCS
Time : 3 hours MaximumMorks: 100
(Weishtase70%)

(i) Therc arc tuo &ctions A and B.


(ii) Attempt qny three questiansJrom SectionA. Each
question carries 20 marks.
(iii) Secrion B is compulsory ond corries 40 marks.

SECTION A

l. How would you descib€ a group ? Explain t"?rious stag€s


oI group development and probable effects of $oup
proc€sseson t€am performance.

2. Brie!|9 descnbe the concept ol 'stress,, and discuss!"€riou5


typ€s of organisationalsir€sswilhin the .Rolesystems,.

M s2 6 @ P.T.O.
3. Define and describe the concept of power. Dscuss the
bases of power in the context of Managerial Effectiveness.

4. Define and describe ihe process of delegation. Discuss ihe


Iacilitating and hind€ring forces which in{luence ihe
cleiegationprocess.

Wrile sho"t nore\ on any three of the lollowing .


(i) Compliance

lii) Empowerment.Audii
(iii) Organisat'onalCuliure
(iv) Cross-culturalDlnamics
(v) TransformationalLeadership

M5-26@
SECTTON B

Please read tlle case and answer the questions given at ihe
end.

Th€ Roopchand Departmental Stor€s, New Delhi,


has a separate s€ction to sell winier garments. This
section consisis of two groups : permanent clerks and
temporary clerks appoinred during wint€r season.
Unfortlrnately, this section has always witnessed a
tug'ot-war situation betw€en p€rmanent and temporary
clerks.The stoiy begins thus.

The permanent clerks, by virtue ol their long and


fruitful association in the Departmental Stores, had
developed intimate social relations among ihemselves.
Usually, they take their lunch and tea together. They
worked, like a well knit group, in closeproximity and not
surprisingly, developed close relations even after the work
is over. The slackperiod startingfrom Aprii to September
€very year had given them a wondertul opportunit to sii
together, discussabout rratters of common interest,peep
into each other's minds and, il possible, solve their
problemsto mutual ad\rantage.Presently,all of ihem are
unnarried with the exceplion al twa.

MS-26@ P.T.O.
The temporary clerl.,s were school girls who are
normally appoint€d before the commencement of winter-
Often, they have complained about th€ir work in the
Winter Garments Section and two of the clerk even
desir€dtransfer to som€ other section after a short stay.
The teasons are quite obvious : The perman€ni clerks
bothered littl€ about helping the newcomers- lf the
newcom€r is not able to sirike a deal with a customer,
often. the permaneni clerk joined hands in making
uncharitablecriticism before th€ Section-in-charge.
Being
small in number, the temporaryclerkscould noi resistihe
frequent onslaughts,which ar€ at iimes irritating and
in:inuaring.The remporaryclerks had very littie scope to
blow off thei "steam"-

In this heated atmosphere. the determination of


commissionon sales remained a contentiousissue, The
permanentclerkshad, throughan inlormal understanding,
agreednot to boost up sales.They feared,inherently,that
fluctuations in sales would lead to fluctuation in
e$ployment. At the same time, they could not tolerate
the sight of a temporary salesperson pocketing a
fair share of commission. They always feh thai the
temporaty on€s do not deserveany commission,because
they have not contributed angthing on a continual basis
for the developmentof the D€pariment. Sulfering from

M3-26o
an jnfiated ego, they also believedthat they had the righi
temperamentand skill to boosi up sales.Thur, in everj
\,,ray,the temporary sales c)erks are inferior io them.
Rec€ntly an unhappy situationhas d@eloped when one
ol the t€mporary clerks js able to make a subsiantial sale
to oDe of her acquaintances.

Now, ih€ permaneniclerksb€ganto make a hue and


cry (egarding tbe commissiolr payment. The poor sal€s
clerk came io ihe Assistant Sale$ Manager 'J,,iihiears atter
haling been scoldedbitterly by the perman€nt clerk for
t having sold a few s^,eaters in th€ir absence.

The tempotary clerks were always anxious to show


good performance so as to earn a position in the
Department. To prevent this, the p€rman€nt clerks used
to furnish fals€ information about garments'qualiiy,
n€gotiable price etc. Wh€never ihe iempolary clerk
sought ihe help of a senior in seliing ihe garments to ihe
cr.6tomer, the latter would take over and claim th€
commission on sale herself. The temporary clerks were
often assigned to insignificant task like aftanging
displays, rearanging garmenis, bringing garm€nts from
stores, etc., and were prevenied from sinking it rich wiih
customers. At every stag€ tbe temporary ones ar€ taken
{or a ride by the permanent clerks.

MS26@ P.r.o.
Quecttons .
(a) Disclss the issLes relevant tor group behaviour_

{b) Explain tbe behavioi/ al WmraneDt ernployees ol


the department. Are iheg justified ?
(c) If Vou were the managerin-charge,what siepswouid
you have laken to preveDl such a situatjon ?

{d) How can an organjsatjon be proactive in such a


situation?

Ms26@ 1,000
ll,rs"'o-l
MANAGEMENT PROGRAI,IME
Term-End Examination
December, 2005

MS-27O : WAGE AND SALARY


ADMINISTRATION

Ttme : 3 hours Maxit'um Msrks: 700


(Weishtase70o/o)

(i) There are two SectionsA dnd B.


(ii) Attempt ony fo r questionelrom SectionA, eoch
question corrying 75 marks.
(iii) Section B is cornpulsory, and cdrries 40 marks

SECTTON A

l. Explainuarioustypes of inceniiveplans.What stepsare to


be taken for elfeciive implementation of incentive plans ?

2. Compensation management is a multidimensional concept


incorporating such asPects as pay structure, pay level,
gystems {or r€cognislng indMdual dif{erences 1o
p€rlormance.and emnloyeebenefirs.Discuss

MS27o P T.O.
3. Discuss some of the wage theories developed by economic
and behaviouml sci€ntists.

4 - What are the objectiv€s of compensation survey ? What


Upe oI information is io be collected through
compensation su^7ey ? Give details.

5. Aiiempt the lollowing questions i


(a) What are the main provisionsof ihe Paym€nt of
Bonus Act, 1965 ?
(b) What are ihe salient features of th€ Equal
RemunerationAct, 1976 ?

6. What are the insiitutional mechanismsin lndia for fixaiion


of wages ? Disclrss.

Write short notes on any ,hree :


(a) Pay equity
(b) Voluntary Retirement Schemep(VRS)
(c) Job evaluation procedure

{d) International labour standards on wages


(e) Compensation issu€sand trends

MS,27
SECTION B

8. Read the following cas€ and answer the quesnons gNen ar


th€ end .

CASE

A bright young M.B.A., fresh from one ol the top


manag€ment instituies, took over his faiher's
r€sponsibilities as president in a manulacturing compang.
The company employed approximately on€ thousand
peopl€ in the productiondivision.It had never laced with
a demand for colleclivebdrgarning.So lar as was known,
none of its €mployeeswere union members,

The new president,after three y€ars of climbing the


ladder io his posihon, had a conference with three
long-term employees,who explainedthat they and their
associates had been discussing the desirabiliiy of
bargainingcollectiv€ly.They gained through such formal
representation.Th€ employeeshad not vorceoany srong
criticism of manag€ment, but they had held several
meetings and had invjted representatives of a national
union lo talk with rhem. They concludedrhar rhey ought
to try collectivebargainingand for this purposeformed a
union and enlist€da majority of worksnop €mproyeesas
members.The three representatives had been elected to
the bargaining committee in order io present a written
memorandum to ihe president with a request for
coll€ctive bargaining agreement. A series of issues

MS-27 P.TO,
carefully spelted out by the union were handed over to
the president by the three representatives

The young executive received th€m cordially and


listened carefully. He acceptedtheir memorandum and
suggested that he would like to have time io shrdy it
carefully. He proposed a meeting with ihem for Tuesday
of the tollowing week.

When ihe committee members retumed, the


president reminded them that th€ company had been
carefulto maintainwages and working conditionsat least
on a par with those in unionised companies in the same
indusiry and region. He expressedthe opinion lhat the
specific proposals they had presented seemed to him
quiie reasonabl€ and approprjate. He had been thinking
of many of th€ same changes and would probably have
made them without their request. He was pleased to hand
them their memorandum with a notaiion indicating his
acceptance.The members lelt, quite satisfiedwith the
effectiveness o{ their negotiations and promising to repotl
back to him as soon as possible

One week later, the presidentfound the same group


of representaiives waiting to se€ him. They appeared
somewhat clestfallen and embarrassed They reported
thai they had gone back to the membership, presented a
Iull report ol iheir discussionswith him, explained.his

MS27
favourabLe attitud€, and recommended tormal ratificatjon

I ol rhe memorandum as a new collectrve bargaining


p agrp€menr.Alter exrensivediscussion.when lhe molon
l" Ior ratification came up {or a vote, a maiority of ih€
m€mbetship voted against raiificafion

Questions I
(a) What are the problemsin thjs cas€?

lb) Why did the members refuse to ratity the


agreement ?
(c) ln a slfuation lilie the one above, hc,v can the rmion
and management play an ellective rale in fixing and
revising wages and ben€fits through the process of
collectiv€ ba$aining ?

MS.27 1,000
MANAGEMENT PROGRAMME
Term-End Examination
December,2OO5

MS-2S@ : LABOURLAWS

Time : 3 hours MoximumMorks: 100


(Weightase70Yo)

Note :

(t) Thereore tu)oSectionsA ond B.

(it) Attempt on9 frue questionsfrom Section A. AII


quest'ions
ln SectionA corrg 15 mcrks.
(iu) Section B is compulsorgand corrles 25 morks.

SECTIONA

l. The Fuidamental REhts ard Directive Ptincipleg of State


Policy are the backbone of industrial iurisprudence in India.
Explain.

Ms,2so P.T.O.
2. Attempt any tuo of the following,
(a) What are the safe! measures under the Factories
Act, 1948 ?

(b) What are the provisions regarding registration of


establishmentsand licensing of contractors under
the Contract Labour (Reguiatton and Abolition) Act,
1970 ?

(c) What are the main provisions regarding prohibition


and regulation ol child labour undef the Chlld taboir
(Prohibitionand Resulation)Act, 1986 ?

Explain the procedure for registraiion of a trade union


underihe TradeUnionsAct, 1926. What are the provisions
regading "General Fund" (Sec-15)and "Political Fund"
(Sec16) under the Act ?

4 . Examine various Fovisions laid down under the Industrlal


DisputesAcL. 1947 for s€ltlemenrof induslrialdispures.

What is the objeci of the Minimum Wages Act, 1948 ?


Explain the procedure ihe government has to follow lor
fixing and revising minimum wdges under the Act.

6- Who :s an "employee" under the Emplogees' State


Irsurance Act, 1948 ? Gve an account ot v.a|jousbenefits
extended to ihe employees under the Act.

Ms.28@
7. Write shon notes on any tiree of the following I

(a) Objectives of labour laws

(b) . Iniernational labour standards and iheir inlluence on


labour legislation
_
(c) Notice of Change (Section 9-A)

(d) lndustrial djscipljne - concept and objectiv€s

le) MarernityBenefitAct, 1961

Ms-28@ P.T.O,
SECTION B

8. Redd the case given bdorv and aniwer the questions given
at the end.

CASE

ANC is a composite textile mill employing ovet


8000 employees including olficers. The rnill has recently
modernised its technology and has introduc€d many cosl
and latDur savins devices. lt has a very sophisiicated
quality control department having many costly
€quipn€nts. The jndustrial rclations in the company has
been very $nooth. The management also adopied a
!€lfare-orient€d approacb lolvards ih ernployees. The
company organised regular training and d€velopment
programmes as a part of its huma! resollrce development
etforts. The general morale and motivation level of
employees was quite high though symptorns ol tension
and mistrust w€re hidden.

Mr. Anand is in chargeol rartous costly €quipments


us€d in the qualily control department. One day he used
th€ services of Mr. Borade, a worker, to clean the

<uipmeDts of various sizes and asked him to keep back


the eqJipments in lhe cupboard after cleaning theft. This
had to be done as Mr. Anand had to attehd the dailg

Ms-28o 4
coordination meeting with the general manager of the
company which cannot be ignored. When Mr. Anand
retuned. he did not check immediatdy the total number
of instrumentsas he did not have any doubt about
Mr. Botade's integrity. Mr. Borade is an old-timer who
servedrhe company honestlyand ,in.erely.

Alter about hvo hours, Mr. Anand found a small but


costly measurement equipmeni was missing from the
cupboard. The initiol search yielded no positive results.
Mr. Anand accordinglyinformed the security about the
missing ol the instrument. The security staff suspected
Mr. Borade as he was ihe one who cleanedand put back
the instruments.When questioned,Mr. Borade deniedthe
charges. After initial questioning, Mr. Anand onc€ again
separately requested Mr. Borade to say whether he saw
the missing instrument. Mr. Borade spoke privately to
Mr. Anand and admitt€d that by a sheet loss of control
of mind he iook away th€ instrument.He conless€dthat
be never did such a thing in the past ir) his 15 years oi
seruicein the company and knowing welt that he couldn't
us€ the instrurnent, h€ stif took it away. He pleaded for
mercy- Mr. Anand was at loss to understand what had
happ€ned.He couldn't b€lievethat a man who servedthe
company faithfully could €ver do such a thing lik€

M S2 8 o P.T.O.
he do now ?

Questions t

(a) Advise Mr Anand as io the nature of action to be


taken against Mr Borade ?

(b) Advise Mr Anand as to ihe merits and demerits of


holding a domesiic enquiry.

{c) Also suggesi to him various steps to be taken in a


disciplinaryproceeding,in caseol an enquiry,and the
nalue of punEhmentro be awdrdedil rhe chargesare
proved-

Ms-28o 1,000
MANAGEMENTPROG&{MME
Terrn-End Examinatlon
December' 2OO5

MS-41@ : WORKINGCAPITAL
MANAGEMENT

Time : 3 hours MoximumMorks: 700


{Weishtoge70o/o)

Note :
A, B ond C'
Thereorc three Sections

s,ectlon A is meant only lot the students, t,ho


houe rcgistetedlor MS'47 priot to July, 2004 I e
uptoJonuary,2004 semester

Seciion B is meont only lor studehts tegisteted


Jor M3'41 frcm July, 2004 semester onurards

Secfion C is compulsory t'ot oll the students

P.T.O.
MS-41@
SECTION A

Note: Attempt any tbo questions. AII questions carrg


20 marks each.

1. "The most challenging job before a finance manager is to


ensure a proper trade-oJ{ bet$,een risk and return v,,hil€
making financial decisions." In the light of the statem€nt,
illustrate with suitable examples, how a Iinance manager
ensures desired irade-off behre€n varlous wo*ing capital
decisions_

2. (a) State with r€asons whether the {ollowing statemenis


are true :

{i) A finance manager must enswe a Cunent Ratio


of 2 for his^er firm - neither more than 2 nor
less than 2 for achieving optimum working
capital structure,
(ii) A sound working capital financing plan shorld
ensure that all the current assets must be
. financed through current liabilities.
(iii) Cash Flow Statement is same as Frmds Flow
Statement.
(w) The only motive to hold inventory in a firm is
Precautionary Motive.
(v) Carrying idle cash involves an opportunity cost
which is measured through int€rest rate.

Ms-41
@
State the objectives a finance manager follows while
managing working caPital lor a manulacturing lirm
Also describe the differences between ihe P€rmanent
Current Assets and Temporary Cunent Assets.

3. (a) Whal do you understand by Ageing Schedule of


Acco$ts Receivables? Explain lts significance a|1d
discuss any tro methods to anpedite the cash
collection from the debtorc in a firm.

Explain and discusssix C's a tinance manager mighl


consider in evaluating the credih^/orthiness of a
prospective customer,

MS.41 P.T.O.
o
SECTIONB

Note t Attempt on9 tu'o questions. All qu3tions cong


20 morks each.

1. (a) Deline any fioe ol rhe following:

(i) Fluctualingor TemporaryCunent Assets

(ii) Acid-T€st Ratio

{iii) OperatingCycle
(iv) Trade Credit as a Source of Working Capitat
Finarce

(v) CashFloruSratemerlt

{vi) Liqrjdit as ar objective of Working Capital


Management
(b) A firm's cunent assetsand curent liabilities are
Rs. 160 lakhsand 100 Iakhsrespectively.How much
maxirnurncan it borrow on a shorf-lerm basisso as
to meet its demandfor additionalinventory wjthout
reducingihe cuffent ratjo below 1 25 ? Assr.rme
that
the whole arnount boftowed on shoft-term basis
wodd be in,,€stedin inventoryonly. Nam€ ant four
sourcesof financingwo*ing capitalrequirem€ntof a
firrn.

M s - 4o1
2, {a) "Merely increasingcLrrrentassts oi a Jirm does not
nece5carilvr€duce iis liqu'diryrisk : insreadrt is the
compositionof liquid assetsand nonliquid assetsin
the overall siructure of current assets which is the
'
most important factor in reducing its liquiditg risk.
In the light ol this statement, define Liquidity Risk as
a relevant concept in the overall managementof
working capital. Also, discuss your opinion/position
in this regard.

(b) "Holding of cash in a firm is similar to that of


holding ol inventory and therefore, the principles of
Invenlory Management can be applied to Cash
Management." Do you agree with the statement ?
Support vour answer wth nec€ssary r€asons and
aQmPles.

MS-41
o P.T.O.
The BalanceSheetsof Narr-rlaLid. as at the end of
2003 ar'd 2004 arc gven belo\^]:

x x x x x x x
O O O \ O r O L o ! )
O O @ F - \ o +

o S F O N O O
o o

€ 9 6

* : i E p

fl
$ s : ; 9 *
E d | i s o a 6 ( i
3 ?

o r l o F o \ o \ o o r o l o

o o o

p . ?

, e S5 , n I
-uFgjeE-ili$
.5
Y t T s € s F 5 , 3 i :
J ##.'FE.g--F;J
MS-41@
A ptant purchasedfor Rs. 4,000 (depreciation,
Fts. 2,000) was sold for cash at Rs.800 on 30th
Spptember,2004 i an item of fumiture was prrchased
for Rs. 2,000 duringthe year. Depreciation on plant
was providedat 80/6on cost (excludingsold out item) and
. -^l^,
cost.A dividendol
on fumitureal I2;% on average

221% on onqinalshareswas paid


2 -

You are requiredto prepare a Fund Flow Statement


lor the year 2004.

Ms-41@ P.T.O.
SECTIONC
Note: Attempt any three questions.All questionscarry
20 morks eoch.

4. The bottom-line ol Reliable Industries Limiied is under


heaw pressure. The management of the company is
worried about th€ same. They have appointed a sftategic
group working on a strategic plan named as 'Shoot-up
2005' to lmprove both the topline and bottomline ol the
company jn tuture. Amang otbers, one issuewith wlicb the
group is confroniing is that of th€ credit policy. At present,
the company provides 30 days credit to their customersand
is having salesof Rs. 50 crores. lt is estimatedtbat the
companys cost of capital is 10% and the ratio of cost to
salesis 850/o.The group is planningto increasethe credit
period from 30 days io either 45 days or 60 days so as to
give a boost to sales. It is forecastedthat such an extension
in the credit period world push sales by Rs. 3 crores and
5 crores respectively and the proportion of bad debts on
the additional saleswould be 8%.

Assume thai you are a member of the group and have


been requested to evaluate the impact of extending the
credit period on the profitability of the company.
Therefore, you are required to evaluate both the proposals
and give your suggestion whether the group should go
for an extension of the credit period or not. lf yes, ther'
should it be for 45 days or 60 days ?

Ms-41
(t
5. (a) The followinginformationhasb€enotltainedfrom the
rccords of PYN IndustnesLimited about th€ir ten

AnnualUsage PricePer Unit


It€m No.
(Quantity; (Rs.)
3 1,500 10.00
19 1,000 080
22 500 190.00
23 2,500 o.77
2,000 72.50
36 150 1000
47 2,250 010
54 900 833
6a 1,200 62 50
82 650 20 00
Classily lhe items inio various caiegories using ABC
classificationtechnique.

(b) Rajiv Tools Pri,,raie Limited is expecting to have


Rs. 25 lakhs in cash ouilayslor the next year, The
company has determined its opportunity cost oI
holding cash as interest |ate of 12.5% and wil( incur
a cost of Rs. 90 each time it bonows (withdraws).
Assuming that ihe cash outlays will be steady and
uniformly distriblrted over the next year, determine the
optimal borrowing or rviihdrawal size for the company
usingBaumol'smodel.

M S - 4@
1 P_T.O.
6. (a) An organisationordersan item in lots of 1300 units
for which it has 5200 units yearly demand. The
holdingcost per unit p€r year is Rs. 10 and lost sale
cost per unit is Rs. 50. Quartedydemanddistribution
is as follov,rs:

Demand{in units) Probability

1150 0.10

1200 0.05

7250 011

1300

1350 009

1400 0.07

7450 0.03

You are requiredto determineoptimum safetysiock


for the organisation.

(b) Explainthe following :


(i) CommercialPaper
(ii) C€rtificateof Deposits

MS-41
o 10
A firm has filed a request for working capital finance wiih
a commercial bank. You ore requested by the bonk to
preqre an estimote of the borking capital requirements
oJ the lim. For that you may add 10% of your estimated
figure of working capital requirement to accoum ror
exigencies. In this regard, you have been provided following
necessaryinfofmation :

Projected Profit and Loss Account of the lirm for


the next year

(Rs.)
A. Sales 22,47,OOO
B. Cost of GoodsSold 16,37,100
C. Gross Profit (A - B) 6,09,900
D. AdministrativeExpenses 1,49,800
E. SellingExpenses 1,39,100
F. Profit before Tax (C - (D+E)) 3,21,000
C. Tax Provision 1,07,000
H. Prolit aft€r tax (F - G) 2,l4,OOO

The cost of goods sold is calculatedas Lhu" .

(Rs.)
Mat€rialUsed 8,98,800
Wagesand other Manufacturing
Expens€s 6,68,750
Depreciation 2,51,450
18,19,00
Less: Stockof FinishedGoods(1070of
Productsnot sold) 1,81,900
Cost ot Goods Sold 16,37,100

Ms-41@ 11 P.T.O.
The ligures given above relate only to th€ goods thai have
been finished and not to work-in-progressi goods equal to
20o/ool the year's production 1in terms of physical units)
are in progress on an average requiring full material but
onb 50% of other €xpenses.The firm has a policy ol
keeping two months' consumption of material in stock. All
expenses are paid one month in anears. Suppliers ol
material grant on€ and a half months' credit; salesare 25%
cash while remaining sold on two months' credit.

8. Wnte short notes on any lioe of the tollowing :

(i) STIFP (Short Term Integrated Funds Planning)


Process

(ii) Role of the Discount and Finance House of India


(DFHI) in the.developmentof Indian Money Market

0ii) Srone Model for ellicientcashmandgement

(M Basicprinciplesof bank lending

{v) Trade Cfedit - a cheap source for financing workjng


capital

(vi) Collection Period Analysis - a tool for Receivables


Management

Ms-41
(D 12 1,000
fM"-rO-l
MANAGEMEM PROGRAMME
Term-End Examination
- December,2OO5

MS-42G) ; CAPITALINVESTMENTAND
FINANCING DECt$rONS

Time: 3 hours MaximumMarks, 100


(Weiehtase
7Wa)

(i) There are three Se.fions A, B ond C it) this poper'


(ii) Students who hove registercdlar MS'42 prior to
Julg 2004 semester'shauld ottempt an? fioe
guestions from SectionsA snd C.

liii) Students uho hooe registetedfor MS-42 lor July


2004 semesierand aJtetu)atds,should ottempt any
'
fioe questions fron Sections B ond C.
(iv) All quesrionscarrg equal morks.
(v) Present Value tablesuill be protided, iJ ssked Jot

MS42o P.T.O
SECTION A

I. {a) What do you understandby Time Value of Money ?


(b) "Obtainins PositjveNPV implies ihe same thing as
minimising the cosi of capital.' Explain this
statementwith examples.

2. Dstinguish betw€en Commercial Banking and Merchant


Bankng. How do both difter from Development Banking ?
Explain the funciions undertaken by State le\€l
developmenl banks in lndia.

3 . What are Righis shares ? How do ihey ditfer from Bonus


shares ? Why do companies issue Rights shares ? Explain
with an €xample tbe ellect on the shareholder's posiiion ii
he does noi subscribeto ihe Rjghts issue.

MS42o
, SECTION B

l, What do you understandby Corporate Restructurjng?


Whai lorces motilbte the corporai€s to restructute ?
Discuss the dimensions of Corporate Reshuciuring.

2. What innovationshave taken place in equity products ?


Discuss their significance.

3. Wnte explanarory notes on the foljowjng i


(a) Supplier'sCredit
(b) CommercialPaper
(c) Factorihg
(d) Pordoliolnv€stment

MS42@ P . TO
SECTIONC

4. of a companyis as follows:
Th€ €xistingcapitalstructure
Paid up share capital of Rs. 10 each Rs. 10 crore
Resewes and Surplus Rs. 15 crore
14oloDebentures 8s..15cror€
An expansion plan requinng Rs. 20 crore is being
considered.The plan is expected to increaseEBIT by
Rs. 6 crore ov€r its present level of Rs. 8 crore. The
. following three plans of financing have been sugg€sted I
Option 1 - To issueequity sharesof Rs l0 each at a
premium of Rs. 40 per share. The share issLreexpenses
will result in the net proceedsof Rs. 40 per share lor
every new share issued.
- To borrow at 15% from f'nancial instituiions
94j9!3
Option 3 - To boftow Rs. 10 crore @ 15Vo and the
balanceto be rais€dby issueof equity sharesas p€r th€
t€rms indicatedin Option 1.
The applicablerate of income iax is 40%. Find out
(il Tn€ expansionptan ic to b€ coasrdetedor v il rhe
EPS increases from its pfesent leve! lndicate
' whether the expansion plan qualifies for
consid€ration?
(ii) At what level ol EBIT, will the EPS be equalto zero
under each of the financingaliernatives?
(iii) Determine the indifierence level o{ EBIT among the
ihree alternativesand also the correspondingEPS.

MS-42@
A manulacturing company wighes to dei€rmine th€
cosi of capital o{ €\'aluating capital
b,rdgeting pYojecis its BalanceSheetis as follows :
Balance Sheet
Liabililies Rs. Assets Rs.
Equiiy shar€
12.00,000FixedAssets 25,00,000
capital
Prel., shar€
4,50,000Curent Assets15,00,000
capiial
Retained
4,50,000
earnin9s
Debeniures 9.00,000
CuIrent
10,00,000
Liabilities
40,00,000 40,00,000

Add it io nal lntor matio n :


{i) 20 year 14% Debentur€sol Rs. 2,500 lace value.
redeemableat 5% premium can be soLd at par;
flotation .cost is 2%.
(ii) 15%opreferenceshar€s : Sale price Rs. 100 per
sharei Flotationcost is zEo.
Uiil Equrly 'ha'pc : Sdle p'ice Rs. ) 15 per ,hare.
Flotaiion cosi is Rs. 5 per share.
The corporale tax raie js 55%.
The expecied grounh in equiiy divid€ndis 8olop€r year.
Expected dlvidend at th€ end oJ the current financial year
rs Ks. ll per snare
Calc\rlatewe:ghi€daveragecost oi capital.

MS42@ P.T.
O
6. Whai tools ar€ available to the project manag€r to use in
conholling a project ? ldentify some characteristjcs of a
good coffror s9stem.

What do you understand by ;ocial cost benefit analysis of a


project ? What lactors wolrld you ial€ into consideration
whjle undertaking such an anallsis oJ a project ? Explain.

8 . ABC Co. Ltd. is consideiingihe acquisitionoI a large


equiphent to set up its factorv in a baCkwardregion for
Rs. 12,00,000. The €qujpment is expectedto have an
economic useful life of 8 years. The equipnell can be
tinancedeither with an 8 yearsterm loan at 1496,inier€st,
repayablein equal instalmentsof Rs. 2,58,676 per year,
or by an equal amount of Jeasereri pgr year. In both cases
paynents are due at ihe end of the !€ar. The equipmenl is
subject io the straight line method of d€preciation for tax
purposes. Assuming no salvage \ralue after the 8 year
useful life and 50ok tax ftte, r{'hicI of the financing
alLernativesshoLd it selecL?

MS42o 1,000
I MS.43
o I
MANAGEMEM PROGRAMME
Term-End Examination
December,2O05

MS-43 @ : MANAGEMENTCONTROL
SYSTEMS

Time: 3 hours MaximumMarks: 700


(Vletghtase7V/o)

Note ,

(i) Thereore threeSecrionsr SecfiohA, B and C.


(itl Section A is meont for the studehts a7hohoDe
registercdJor MS43 beforeJdnuaryZ0OSt.e. upto
July 2004 semester.
(Iiil SectionB is fieant lot the studehlsuho hoee
regtstercdlor M5.43 Jrom January 2OOSser,ester
onuJdrds.
(io) Attempt cny thtee questions
lrom SectionA
or B. Each questionh SecttonA ana B crfties
20 marks.
(v) SectionC is compntsory
lor oll ond corries
4O 'l|,otks.

MS-43@ P.T.O.
SECTIONA

1. {a} Desclibe the characierisiics of Management Control


Ststems.

tb) Can an organization have more than one type oi


responsibilii!' centers ? lllusirate with the help of a
charf/dia$am.

2. lu) What ar€ the advaDtages o{ performance related


compensation in th€ context o{ pro{it ceniers ? What
pitfalls should an otgarization guard itself against
wh€n jntroducing su.h a schem€ ?

{b) Dscuss lhe merits and demerits ol "oppodutrty cost"


a. u mLthod lo. t untf"l pri"ing-

3. (a) Explain ihe three comppnenis ot Revenu? Budgeis'

(b) What is an MIS and \,,rhatate the elernentsot a solrlo


MIS ? ln what way is an MIS helpful in an MCS ?

4. (al Dis<uss in general the structure of a multi_projeci


organization. What general arrpfoach codd be
folloqJedin designing proiect orga;izations ?

{b) Dfferentiate belween ptofit and oon'profit


organizations. What implications would the
difierences, if any, have {or management conttol m
non-profit organizations ?

lils-43@
5, Explain any thr€e ol the folowjng:

(4 Mujtipje Wrfom€.nce criteria for evaluating


perforrnance of investm€nt center md|agers

(b) Zero,basedbudgeting

(c) Molivationalvalueol p,ofir cenlers

{d) R€sponsibilityReporiing

(e) Residual Income (RI) ds a measure of performance


measur"menlol ;n!€stmerr center5

MS-43@ P.T.O.
SECTION B

1 . Explain the relevance ol Porter's five forces in designing a


compeiitive slmtegy for knowledge based organisations

2 . Critically analys€ the various methods us€d lor determining

3 . Wdte short noles on the fo[owing :


(a) Total Quatity Management {TQM)

(b) BusinessProcess Reengineering(BPR)

(c) Management Control in Proiect Organisations

(d) Investrnentcentres

4. How is a senlce organisadon differenl from a


rnanufachning organisation ? How do these dilferences
aflect tbe coBftol Sstem design in a service organisalion ?

Hoff do c\rltural diffelences across nations aher the


ManagemenL Control System.? Explain.

Ms-43o
SECTION C

6. Read t-hefollowingcds?carelulfuand answerlhe quesrions


giv€n dl (he eDd j

Abtarns Company

Abrams Compang manufaciur€d a wide vari€ty


of
parts ldr use in authomobiles, trucks,
buses, and larrn
' equipment. There were three maj.or groups
o{ payts :
ignition parts, bansmissioo parts, and
engine parts.
Abraing paris \^,ere sold boih to original
equjpment
manufacturerc (OEMS) and to wholesalers.
Tl1e
wholesal€rs, in tum, resold the paris to retailers
who sold
tt|€m as repiac€ment pafis to consum€rs.
The latter
market was ca ed th€ ,.attermarket,,(AM).

Product and Marketing Dlvislons

As showh jn the pafial organizafionchart in


E\hibit
1, Abrams had a ,,productdivision,,for
each of ts fhree
pad groLlps. Each of these produrt
dMgions was managed
b9 a vice president and general manager
who was
expected to earn a target reiuri on investment (ROl).
Lach producl divisionmdnulncrured
ndils in 5everalplanls
and sold a major portion o{ its manufactured parts
to
OEMS. Each product divisionhad jts separateOEM
sales
department (see Exhibit 1) that work€d ciosety with
OEMS
- io develop new products or
change existing products. The
remaining manufactured pads were sold by the pfoduct

Ms-43@
P,T.O.
tlre AM
division io Abrams' fourth division catled
"AM Division" as it
Markeiing DMsion (see Exhibit 1J or
also managed
*as known to managejs This dMsion itas
and was solelv
by a vice president and general mataget
ot patts to
responsible for marketing Abrams' entire line
se!€ral
AM wholesalers. The AM dMsion operated
in the us
company-own€d pads distrjbution watehouses
expected
and {oreign markets The AM division was also
to earn an annual teturn on inveslrnent talget'

lnside and Outside Sales


8s 500
ln 1992, tbe {our divisions' sales totalled
100 million
Inillion, which included"inside" salesof Rs
division The
from Lhe three product divisions to the AM
approximately
Rs. 500 million sales ivete recorded as
Rs 100
Rs. 130 million for the ignition Paris division'
Rs 90 mlllion
million {or the transmission paris division'
miltion tot the
for the €ngine parts di\ sion' and Rs 180
sales' Abtans'
AM division. Alter elimtnation of Inside
Because of
outsid€ sales totalled about Rs 400 mlllion
due to tbe
anticipated gto&th in ihe Pads' atiermarket
ddven and their
irr..nu.e in ihn number of vehicles being
the AM Di'isron
ages, one o{ top management's goals {or
of Abrams toial
was fot jts sales to reach 50 dercent
outsid€ sales

ROI {or the Manutacturing Plants


the compang's ROI strategy' each
Continuing-plaht
riranufacturing within the three product di!'rsions had

Ms-43o
an annual ROI target to meet. Each product division,s
OEM sales were traced to the plants ihat made the parts.
The plants maintained finished goods inv€ntories and
shipped parts directiy io OEM customers.A plant s ROI
target was based on budgeted profit (includins allocations
of division and corporate overhead erperrses and an
imputed income tax expense) divided by actual
beginning-otthetear net assets(delinedas totat assetsless
current liabilities), Exhibit 2 contains an exampte oi the
Rochester plani's ac|ual 1992 ROI computation. Actual
ROI was actual profit divided by actual
beginning-olthe'yearnet assets.

EXHIBIT 1 : Partiat Orsanizaton Charr

ChielExeculive
0llicer

Chisl0perating
0flicer Indusrrial

GenelalM
AL4ltark€tins lgnirionPans Engine
Pads
0ivision Division

0all

Ms-43@ P.T.O.
Top manag€ment's stated reason for including
allocaied overhead expenses and iaxes in determining
prolit was to have the plant profit figute resemble the
profii calculation for extemal financial reports io
shareholders.The CEO lelt this gav€ a plant manager a
clearer perspectiveof the costs ol doing business,the
plant's coniribution to the corporate bottomline, and
added more realism to ihe plant's results

The beginning_of-the_yearnet assetsamount was used


in the ROI measurementbecause,in managements lTew'
investmentadded during a given year resultedin liide, if
any, incremental prolit in that year' The inlesbnent
would lik€ly increase luture profits Top management telt
lhat sueh irNesL'rents mighi not be proposed i{ }r}anagers
werc penalized (in the form oI higher net assets and
lowet ROI) in the Jirst year of the |1ew investm€nt'
Because the investment base for the year was "frozen at
(he beginning-oFlhe-year level. lnaximizing profit during
the year was €quivalent to maximtzing ROI FoT
beginningof-the_year net assets. cash and receivables
were allocated to plants on the basis oI sal€s revenue'
while inventories, propertg, plant, equipment, and current
liabilities wer€ traced specifically to -each plant' Histotical
cost tess accumulated depreciation (book value) was used
to value propefg, plant, and equipment The AM

MS-43o
division's ROI was measured in ihe same ftanner as the
plants' ROIS.

Marketing Strategies

The OEM sales departm€nt within €ach of the three


product divisions worked with the OEM s etgineets to
develop inno\raiive and cost-e{tective new pa s lo meer
the cuslomets requireftents and serviced c\rstomeY
accoutts lor'parts already being supplied to the OEMS'
Each of these OEM sales departmenls iras expected to
meet an annual sales revenue target Becuase the prcduct
divisions' custom€F (OEMs) were dillelent {rom the AM
dir,islon's ;ustomers, top management did not teel ihat
the OEM and aftefinarket sales organizalions should be
cornbined- Even the three product dMsions' OEM
marketing efforts were not €onsoljdaied in one sales
orsanjzation because each iivision's OEM marketers
tended. to work with dilferenf people within a given
OEM'S organization (i.e, ignition, ftansfiission, and
engines). Moteover, trvo oI the three produc( dMsions
\tete independent companies before being a'quired by
Abrams. Thus, there was a long ttadition ol doing their
own OEM marketing.

Ms,43o P.T.O.
Exhibit 2 : Acfual 1992 ROt Computation _
Rochestar phnt
Abra|t|s Compary - Thnsmi$ion parts Division
ROCHESTER PI"ANT
Profit and ROt Statement,DeceEber31, 1992
Sal€srevenue Rs. 124,866
Costof goods 73,230
Grossmargin...,.,.,,...,,.. 51,636
Operating
expenses,...,,.,.,,.. 20,792
Dlrsion e\pensesassigned 11,340
Corporate a\penses assjgned 3,420
Profit before ta,\es 16,084
Ta,Yesimputed 4,825
Profit Rs.11.259
Net Assels Assigned as ol January I, r992

Toial Assets:
Cash and receivabtes Rs.25,000
Inventories 12,875
Prop€fty, plant & equipment 86,560
Totalfus€ts 724,435
Lesg current liabilities.,. 26,135
Rs.98.300
Return on rnvestmeri 11-50k

Ms-43@ 10
According to Abrims' executives, the factors critical
to successin the OEM market were ; the abiliiy to design
innovative and dependable paris that met the customer's
qualitg, pedormance, and weight speciJications; meetrng
could
delivery schedule requirements so that the OEM
costs
minimize its own parts inventonesi and controlling
very
Cost control was important because the market was
pnce comPeiitive ln the aftermarket busin€ss' a"?ilabil8
to th€
of parts was by far ihe mosi importani factor
wholesaler, followed by qualilv and price

Incenti!€ Compensation Plan

ApProximately 50 Abrams line and staf{ managers


padicipated in an incentive bonus plan The rupee
pool was established
amount ol ihe co$orate_wide bonus
per share
by a fixed lormula linked to corporate eamings
o'
Each Participant in the bonus plan received a numbet
was in
standardbonus poinis The higher the padicipant
points he
the oryalrizationd hierarchy, ihe more standard
points lor all
or she received The iotal of these
pool amounr
participants was divided into the total bonus
this
to arrive at a standard rupee award per point Then'
number
standard rate was multiplied by ihe pariicipant's
siandard
of standard points to arnve at the pai'ticipaDt's

11 P.T.O.
MS-43
bonus rupees. How€v€r, the actual bonus
couid vary
{upward or downward) by as much as 25 percent at lhe
diacretionoI the participant'ssuperiors.

ln the case of a plant manager,the standard


award
also was adjusted by a formula that r€lated percent
of
rlandard award to the plant s profir vdridnce (budgeL
versus actuaj.profit). For exampie, if the plant,s
actual
profit tor ihe year exceeded its
budgeted profii by
4 percent, the plant manager'sbonus was
raised from
100 percent ol standardto 110 percent
ol standard.ln
making ihis bonus adjustm€nt,th€ plant,s
actual profit
was adjusted {or any favourable or unlavourabl€
gross
margin variance caus€d by sal€s volume
to the AM
divisjon being higher or lower than budgeted.
For
exampler if a plant's lavourabl€ profit
vaiance was
attributable to a favourable gross margin voume
variance
on salesto th€ AM division, the plant manager,s
bonus
would not b€ raised above 100 percent
of standard.
Simiiarly,the piant managerwould not be penalized
if the
AM divison actually purchased less from the plant
than
the amount that was agreedto by AM dMsion
when the
plani's annual prof't budget was
approved by top

MS,43 12
Managemenl Comments

ln general, top management was satislied with the


present management systems and performance

with the casewriter,


measurementscheme-In discussions
however, they mention€dihree areas ol concem

First, there always seemed to be a lew disputes over


transler prices of parts sold by the product divisions to
the AM division. Whenever possible, by corporate policy,
internal sales of parts were made at outside OEM ma*et
prices- If a part had been sold as an OEM part s€veral
years earlier, the orjginal OEM market price was adiusied
upward for intlation lo anive at rhe salespr:ce lo the AM
division. This procedure caused vittually no disputes
Problems occurred when the part being transferred was
strictly an AM division part.That is, it was a Pait never
sald by Abrams in the OEM market and for \;hich th€re
was neither a cunent OEM outside market price nor a
former OEM market plice that could be adjusted Ior
inflation. Usually, such transfer price issues were resolved
b9 the hxo divisions involved, but occasionallv the vice
pesidml o{ finance was askedlo aYbiuaiea dicputd

MS-43 13 P.T.O.
Second, top rnanagement fett that the p;oduct
divisionstoo often lended to treat th€ AM division as a
captive customer. For example. when the AM division
and an outside OEM customer were placing compeiing
demands on a specific manufacturingptant, ii appeared
that the plant often favouredthe OEM cusromeroecause
the OEM austomer could take its businesselsewhere,
whereas the AM division could not purchase parts
outside. Top managementwas not willing to l€t ihe AM
division seli a competitor's product, fe€ling this would
rellect adverselyon th€ overall image oI the company.
The AM division was expected to convince ih€
appropriaie plant managerto undedakethe.manufaciure
ol ils pan needs.

Thid, top manag€ment felt that both the AM


division and the three product divisions canied excessive
inventories most ol the year. The vjce president of
planning said, "Thank goodness we have a generous
Christmas vacation policy here. AL ledsl fie invenlories
get down to a r€asonablelevel at year-end when our
production volume is low because of a large number of
employeeholiday vacations".

l\rs-43 14
Questions ,

(a) Elaluate each of the concerns expressed by top


management, and iI necessary, make
Yecommendationsappropriate to the ctcumstances
described in the case.

{b) What is your overall a'?luation ol Abrams's


management control system ? D€scribe any strengths
or weaknessesthat you identilied but did not include
in answering the previous qu€stion. mai changes, if
any, would you recommend to the top management ?

MS,43 1.000
MANAGEMENT PBOGBAMME
Term-End Examination
December,2OO5

MS-44 @ : SECURITYANALYSISAND
. PORTFOLIO MANAGEMENT

Time : 3 hou's MoximumMorksr 100


(Weightage70%o)

Note : Attempt ony fiue questiohs. All questions cotty


equol morks.Presentuolueand annultgtablesare
to be prootded, il askedlor

l. {a) How are the investors in lndia a{fected by taxes in


respect ol investrnenis ? Should ti€ i ,lasiorsconsdef
tax inc€ntii,eswhile plaDningtheir pordolios ? Discuss.

(b) Betneen selfregulation nnd legisiatile regulation, $,hjch


js more relevant for India and why ?

2. (a) What is interestrate risk ? How are '"aluesof bonds


affectedwhen the Darket Ete of inierest changes?
lllustrateyour answer.

Ms,44@ P.T.O.
{b) A Rs. 100 perpetual bond is cur.ently seiling for
Bs. 95. The coupon mt€ of interest i. 13.5 per cent
and the appropriate discouni rate is 15 per cent.
Calculate ihe vaiue of the bond. Should it be
bought ? What is its curent gield ?

3 . Criticaily examine the role of Securities & Excbanse Board


of lndia (SEBI) in regulating the Primary and Secondary
markets in fhe country.

4 . What are ihe ilajor cdticigmsot the Technical analysis? Do


the Tecfinical analysis and the Fundarn€ntal anal)lsis give
complementary informahon about s€cudii€s for making
inlormal decislons ? Explain.

(a) What is an eliicient poftfolio ? How does the efficient


frontier €hange, when the posstbilit! o{ lending and
bonowing at a lisk free rate is introduced ?

1b) Prathiba & Co. recently paid an annual dividend. of


Rs. 4 50 per share. During th€ same period, the
eamings ol the company were Rs. 8.00 per sha(e.
The required {ate o{ rcturn \rith similaf risk i9 10%.
Divid€ndsare expecledro grow at a consiantrati of
8o/oper gear foi an indefiniie period- Calculate the
nomal pdce-earnings ratio oI the company.

Whg do people inv€st in Mutual Funds ? Dscuss the tariolrs


{actors that are tal@n into consideration b9 the Fund
Manager while inaestins in {i) equily shares and {ii) debt

MS-44@
Write short not€son any lour of the lollowrng:

(a) Defaultrisk

{b) RandomWalk hypothesis


(c). ConstantRupeePlan
(d) Bond Convexig

(€) Odd Lot tradins

(0 Fiherrule

Ms-44@ 1.O00
MANAGEMENTPBOGRAMME
Term-End Examlnation
December, 2OOB

Ms-45 @: INTERNATTONAL
FINANCIAL
MANAGEMENT

Tifte : 3 hours MaximumMorks: 700


lWeishtoge70o/a)

Note: Attempt any five questlons.All questionscorry


equal rnsrks.

l. (a) Elabonte the economic problems which '"arious kinds


of imbalanc€s in international flows could create.
What measureswould you recomm€nd to correct such

(b) What are diff€rent theories of internaiional trade ?


How rel€vant are M€rcantilists '"rews in the modem
day world ?

2. Wriie an essay on "Evolution of international monetary


system and Indian exchange rate system".

Ms-45o P.T.O.
3. What do you understand bV Letter of Credit ? Explain its
significance in international trade and briefb explain vari a
\pes of Letters of Cr€dit.
4 . la) "The extent anal nature of exposure dep€nclson ihe
market segment in which the firm operates." DiscLrss
with suitable examples.

(b) What do 9ou mean by Jorelgn €xchange rate ? Do


{oreign exchange rates show reciprocal relationship ?
Give suiiable examples,
(a) "Under current Indian busines,s environment,
minimising exchange loss is a better approach than
maximising€xchang€gains." Comment.

(b) Briefly a\plain the special features of the Eurobond


market.

6. (a) What do you understand by exposure measurement ?


Explain the intemal lechniqu€s of exposure
measurement,

(b) How can you evaluate global linancial choices ?

Disctlis. in brief. any tour of lhe loilowing :

(a) Posi-shipmenifinancing
(b) EcGc
(c) Strategiesof Direct Inveslment
(d) lnternationalEqui& markets
(e) Uniquemarketsin lnlernationalFinancialSystem
lD T€chniquesot transl€rpricing

Ms-45@ 1,000
MANAGEMENT PROGRAMME
Term-End Examination
December,2005

MS46O : MANAGEMENTOF FINANCIAL


SERVtCES
Time : 3 hours MaximumMarks: 100
(Weishtoge70Vd

Note t There ore thrce Sections A, B ond C.

Secfion A is compulsory for oll the students.

SectionB is meant only lor the students.who hdue


registeredlor M346 prior to Julv, 2005 i.e. upto
Jonuary,2005 semester.

SectionC is meant only Jor the studentsrcgistercd


Jor MS46 lron Julp, 2005 semestero4words.

AII questionscany equol marks.

M546@ : P.T.o
SECTION A
AtremFt any anfee quesr|ons.

1 . Explain tbe concept and inpoftance oI. financiat seMces.


In what ways are the products offered by ihe linanoal
seMces firrns dilfereni from the products oI the indusirial
enterprises ?

2- D€sitbe'ttie axtemd and ihtenal dsks fhai are iaced by


financial s€rvices companies. offering (i) stock broking
sewices, and (ii) fee based eMces.

3- Discuss in detail ih€ wotl'ing mechanism and {unctions of


an Assets Management Co.-rpany set up to rnanage the
Iunds of the mutual fund.

'Crgdit
4. card organisations do not offer any additional
facilities and seruicesto their customers besid€s providing
them the credit faciiig.' Do Wu agree ,,nth this
statement ? Gve r€asons and explain the present position
in lhis regard.

5. Wirat are the important aspects which need to be


incorporat€d in factoring agreenents ? Enuirerate the
advantagesand disadvantages of {acloring.

MS46o
SECTTON
B
Atrernpldny turoquestpns.

6. Dscussthe variousPre-lgsueand Postigsueaclivitiesthat


are petormed by the merchani banker ior rdising funds
trom th€ capitalmarkel.

7- What are Debt ma*et' and 'Equi& rnarket' ? Explain th€


initiativesthat are necessarylor making the debt market
vibrant,

8. Write short noies on anyfour I


{a) Zero Coupon Bonds
{b) Credit Ratlngprocess
(c) Processof Securitisaiion
(d) GlobalDepositoryReceipts

{e} Nalional StockExchange


(0 PrivatePlacement

Ms46@ P.T,O.
SECTION C
Atternpt ony trro questiohs.

What do you undeistand by Dematelia)isatjon of shares ?


Discuss the various stages invofued in the proc4ss of
switchjbg over to d€pository sgstem, Give advantages o{
the depository system.

De$ne lnsuranc€ indicating its basic characteristics.Discuss


lhe \,?riousrisk which are inlurable.

a. Write short notes on any torr :

{a) Financial and Technical Appraisal of Projects


(b) Burglary Insurance
(c) Product Liability Insurance

{d) Securitisationol housingloans


(e) Hire Purchasesystem

{ll Inan Syndication

Ms46@ 4 1,000
MANAGEMENT PROGRAMME
(Banking and Finance)
Term-End Examination
Decembet, 2OOS

MS-422@ : BANK FINANCIAL


MANAGEMENT

Time ; 3 hours MoximumMarks. 100

Note, Attempt ony fite questions.All questions carry


equol morks.

l. Why do the firms raisefundsttyough saleof their securities


in the prlmarymarket? Bdefl9explainthe debtinsiruments
that are issuedin the Indianmoneymark€t.

2. What are diflerent sourcesof owned funds of a Bank.


Briefly explain the \,ariousitems that comprisetiet-l and
tier-ll capitalol a Bank.

3. Elucidateihe factorsthat aflect ibe cost of fundsol a Bark.


AIso anaiysethe reasonsfor higher mte of inte/€stin India
as comparedto develop€dcourtries.

)" l,rs-+zz@ P.T.O.


Dis€ussthe salient feah.lresof :

(a) Money Market Mutual Funds, and

(b) Treae.rnJBills.

5 . Explain the methods general$ {ollow€d by managefs while


e\,aluating projects. Brie& discuss the techniques of
m€asudng the project {isk.

6 . Bnng out the signilicance ol 'operational risk rnanagement'


in Banl(s. Dscuss briefly the rnethadologi€s {or
measurement of operational r'isk9 ar outlined by Basle
Commitiee.

Whai do you mear by 'liquidity risk' in the conta{t of a


Bank ? Discuss the tools of liqujdiiy nik m€asurement and
the issuesinvolved in managing this risk.

8. Wby is pricing ot products important for Bank ? Briefly


explain the Pricing sirategies followed by Banks.

MS-422@ 1,000
MANAGEMENT PROGRAMME
(Banking and Finance)
Term-End Examination
December, 2OO5

OF FINANCIAL
MS-423@: MARKETING
SERVICES

Time : 3 hours MaximumMorks'100


70Yd
NVeishrose

Note; This pop€r consistsoJ two Seclions A ond B


Section B is compulsory' Attempt any three
questionsJrom SectionA. All questionscorry equal

SECTION A

l. How does ihe family life cycle alfect the need for banking
products and se$ices ? What io your vrew would be the
most lucraiive F.L.C. segments lorihe following and why ?

(a) Ove(dratt facility

(b) Housing loans

MS-423@ P . TO
. .
2 . What is the significance of branding for marketing of
bdnkng product. ? Whardre the key consrderations
that th€
bankers must keep in mind while iaking the branding
decision?

3 . What are the sources of proj€ct financing in lndia ?


Critically commert on Ventwe Capital as a financing
mechanism for hew projects.

4. What are the differeni lypes of risl,s associateclwith pension


plans ? Comment upon the issu€sinvolved in pension
tund

Wrire shon notes on any lhree ol t}le foltowing:

(jJ Performance measurement ol pension funds

(ii) Limirarionsin marketingof Insuance seruices

(iii) Marketing mix for financial senrices

{iv) Housing finance market jn JDd;a

(v) R€lationshipmarketing

Ms-423@
SECTION B

6. Read the casegjven bejow ard a answer the questjonsgiyen


at the end of the case :

Case Study

Indian Bank of Commerce and Industrg


0BCI) was
€stablishedin the pre{ndependencedays in th€ 1930s
with the headquarte$ in Calcutta and mainly serving the
then growing industrial centres ol eastem India. The
industries in eastern India wer€ dominated by steel, coal,
cok€, and manufacturingof engineeringproductssuch as
steel castings,machineryand equipment,railway wagons,
jute products,tea, etc_ Together with other
major banks
in the country, the IBCI was nationalizedby the Congress
Government led by prime Minister Indira Gandhi.

The det€riorationof IBCI startedin mid-1970s when


the engin€ering and iute industry in eastern India was
badly aflecredby sLrikes.and industrialunresL,and decline
in the traditionalbuslness.Th€ bonk with a large number
of bftnches in eastem lndia was badly atfected. Ii tried to
expand its coverage by opening new branches,
particularly in the westem regjon, wh€re mere was
an
irdustrial boom.

MS-423@ P.T,O.
The barrk had small branch presence ihroughout the
country. Even while dealing \rith corporate matket, its
specialization and concentration was on relail customers'
ln addition to a great reliance on an intensive dislribution
sgstem that \,.,asbased on banks with large and incr€asinq
number of branches, IBCI l€lt that branch managerc w€re
the perfect channels of communication with its customer'
It was believed by the top management that branch
managers could push up th€ bank's grounh by projecting
the bank's image on both retail and corporat€ clienls
through organizational a5 w€ll as pe$onal influenc€' With
the government's emphasis on spreading banking services
to the less prildeged sections of the socieiy, tbe bank
implemented drives to attract r€tail customers, and carried
out expansion in line with this policy. These w€re th€
types of objectives and policies sei out by almogt all the
nationalized banks. The bank board felt that the clients
should be given personalized service with fYee access to
the branch managers at any time to solve their problems'

During tie last decade oI the tw€ntieth century, the


emphasis siarted to shift from service towards bank
profitability. lt was also feli that the competition in
banl'jng had become intense With the entry and grclt'th

MS-423o 4
of private and foreign bank promoted by ihe
Iiberalizationpoliciesof successivegovernmentsin India,
the luture gro!.,th and profitabilityseemedto be difficult
'targets to achieve. It was felt that some of these new

entrant banks were quite capableof giving some oI the


seruces hee due to ihe com?etition. There{ote, the
competition to attract and hold on to high volume and
profitable customerswas b€comingstiff

Despite the hish costs. the bank managemeni looked


at increasing the market shar€ by using 'ntensive
advertising, paficularly through television and print
media. However, this policy was also opposed by a
number ol senior managersdue to the high expenditure
that would be involved. They feli that the firnds should
instead be used io carry out direct marketing as it enabled
the bank to target and focus on large or important
customers and markets.

In line with the other nationalizedbanks, product


d€velopmeni was quite stagnant The same standard
schemes were promoied by all nationaliz€dbank, and
yield€d similar rates ol interest. Due to its low protitabiliiy
too, the bank could not afford larg€ investmentin product
developmentto promote high grov,{h. tn fact, when one

Ms-423o P.T.O.
of the smaller lndian bank decided to invest Rs. 50
crores in home banklng seruices,the matter was discussed
with astonishmeni by the board of directors of the bank

Current Situation

By January 2001, the profitabitityof the IBCI had


declined sharply, Yet reports published in Economic
Times and other business magazines suggested the
following trends in retail banking ;

(a) Retail linancial sewice customers were becoming


<tuite sophisticated in their reqLri{ement. They
e\pected to be given necessarysewic€s as cheaply as
possible. The knowledgeable writers and experts in
the industry were touting customer contact as the
main differentiator bet[een sen ces of different

(b) At the same time, a lot of experts were emphasizing


the electronic delivery system as ihe main vehicle of
deliveringcustomerconvenience.

The bank's curreni situation was such that the bank


was able io recov€r only 25ok of Ihe cost of ptouding
banking services through direct recovery lrom the clients.
Of ihe non-interestbeaing costs 60-70 per .enI wete

MS-423@
accounted lor by the payment and accounts maintenance
servicesof the bank. In the case of individualcustom€rs,
only about 10 per cent of the costs could be recovered
by direci billing. The bank had to find som? ways of
reducing the costs or iDffeasing the charges \tilhout
surrendering the competitive advantages.

The bank appoint€d a market research consultant on


a trial basis to carry out a survey of its customets in one
area of a city (Table 1).

Table 1 : Market Research of IBCI customets

A. What ls the best thing about vour bank branch

P€rcentageof the
Factor
Custom€rs

Friendlyand helpfulstaff 450k

Convenientbanking services 30%

Quick banking seMce 15%

Eflicient operations 10%

Good advisory service 5o/o

Generallygood

MS-423o P.T.O.
l B. What is the rtorst thing about
Percentageof
the Customers
Long queuesduring working hours
lnconvenieniopening hours and hotidays
Uncooperaiiv€ staff
Poor reception and seating
High charges
Nothing in particular

When Mr. Vyas, the marketingofficer responsiblefor


this pilot project, presented the findings, a nlimber of
senior managers wete unhappy. They felt that since they
had been in banking businessfor well over 50 vears, they
should know what the customerswant. "We have our
bronch managers who are daily in touch wiih our
customers, We need not waste any money on such
superfluotrsresearchschemes,"they said.

Recenily, the management seruce commission of the


nationalized bank conducted a survey ol all the market
bank in the country, including some ol the leading
privaie and nationalized banks. The head of lBct's
customer servi.es division, Mr- Khanna, extracted the
following information from the national data, and, in
addition, prepared a comparison with a slightlv
smaller sized private bank as a further reference.

MS-423@
Table 2 : Main ltems forming part oI the Proflt and
Loss Acount for vear 2OOO- 2OO1
Typical
Item
IBCI
Private
Bank
Bank

N€t lnterestincome(lnterest
€arned less interest paid)
5.lvo 5 1o/o

Other income 7 04/o 2.7%


Total 6 7o/o 78')
Costs
Operatingcosts 56%
Provisionsfor doubtful^addebts 0 5vo 0 8o/o
6 2Va 64%
Profits before tax 0.7% 7.40k

All the aboveligures are a pefcentageof the total assets


employedby th€ respectivebank-
Tabl€ 3 : Customer distributioo by ag€

Major
Private
Age Group IBCI Bank o/o Indian
Bank 7o
Banks %

15-24 16 17

25- 44 29 39 35
45- 64 30 30
Ouer65 24 19

MS-423@ P.T,O,
Table 4 : Socio-economic distribution of customers
Socio-economic IBCI Bank Private Malor lndian
Group Bank Yo Banks
8
c1 18 28 21
C2 37 30 29
79 34
Table5:Depositmix
IBCI Bank Private
Deposit Mix
Bank %
Current account deposits 10 23
Fixed and large deposits 16
Savingsdeposits 74 23
As a part of th€ bank'sinvestigations
for futureexpansion,
ihe bank had also coDducteda smallsuwey of about 250
existing customersto understandtheir inter€st in home
bankihg.The resultsol this surveyare shown in Table 6.
Talrle 6 | Customet Interest in horne banlid.rg
For Home
Lettel ol For Home
Banking/Shoppingnravel
lnierest Banking% ak

High 30 25
Medium 25
Low 25
None 20 23
Not sure 0 2

MS-423o 10
The board ol directors'meetingwas scheduledto be held
. shodty to discllss the future dircction of fhe banks efforts
to retun to profitabili\r. It was f€lt that any such decision
would have to be made after weighing all th€ issues of
costs and long,term growth and profitability. The board of
directors planned to discussthe lollowing issu€s :
(i) Problem o{ current lack of profiiability

{ii) Whether to condqct further market r€search

. (iti) Whether io look at a proglamme ol opening 100


. more branches as a key groMh strategy
(iv) lmplementation of home banking syst€m at an
estimated cost of 30 to 50 crore rupees.
The board of directors neededto make the dectsionurgently_

Questions :
(a) Identify the mdn marketing problems faced by IBCL

(b) What should be the main sbategies along which the


bank can or should develop ?

(c) Should the bank invest jn 100 new branches ?

(d) Should the bank invest in the home bankng system ?


Justify your answer.

MS-423o 1t 1,O00
MANAGEMENT PROGRAIIIME
(Banking and Finance)
Term-End Bramination
December.2OO5

MS.424g : TNTERNATIONAL
BANKING
MANAGEMENT

MaximumMarks, 700

Note : Attempt any tine questions.All questlonscarry


equol marks.

'lntemational
l. What are Enancial Organisations' ? Briefly
de$ribe their main categories and highlight their role in
the World €conomic development.

2, Briefly explain ihe deta s ol Basel Capital Accord, 1988.


Also discussthe amendments firade therein to incorpo{at€
market rtsk.

3. Is it nec€ssary for the parties to stipl ate the 'Choice of


Law' while carrying out int€mational banking transactions ?
What lactgrs in youl view influ€nce 1ie choice of law ?
Briefly discuss.

M5424@ P.T,O
4 . Discuss the diflerent types ol risks involved in the Forex
Dealing Room operations of a participating bank. Aiso
explain the process of eliminating/reducing these risks.

Discuss the 'multi fitcior approach' to risk management.


Btietly explain some of lhe lypicat factors ihai could be
consid€redin this approach.

'online
6. What is banking'? Explajnthe roie of E-commerce
in Cross Border Trade.

Whai do you nrean by 'Currency Swaps' ? Bdefly expjain


different types of cwrency swaps. Also discllss thg benefits
and risks assoclatedwiih these swaps.

E. Write short notes on any tour of the folloLving :


(a) Classification of Regulaiion on Intemational Banking
(b) Private placement
(c) Foreign Cunency Loans
(d) Foreign Bonds and Euro Bonds
(e) Risk Management Framework
(f) Causes of G'obalisation

MS-424@ 1,000
MANAGEMENT PROGRAMME
(Banking and Finance)
Term-End Examinatlon
December, 2OQS

MS-425O : ELECTRONIC BANKINGAND


IT IN BANKS

Time : 3 hcturs Maximum Marks : 70O


(Weightoge7O%)

Not€: Atrempt any floe questions.All questionscafty


equol morks.

r. (u) Explainhow daiaminingis us€fulto managecustomer


tor\dhJ.
(b) What are th€ management issues of netwo*
implementation?

2. \a) Draw an E-R diagGmin a bankingenuronment.

(b) l^4rai is cryptography? Describethe prowrties ol


goodcrypLographrc algoflthm?

Ms-425@ P.T.O.
3. (a) Explainthe technolog risksjn banks.
(b) ExplainIT Act, 2000. How is it rclevantto banks?

4. What is SystemAldit ? Whai are its basic objectives?


DistinguishbetrreenSystemaudit and Cornputeraudit.

5 . (a) E'(plainthe applications


of multimedia
in banks.

lb) How does data mining $pport customer


' intelligence?

6 . What is ElectronicData Interchange(EDI) ? Dscu6sthe


main elementgol an EDI system.

(a). What is a Smart Card ? Dscuss diflercnt types of

(b) Explalnthe componentsof ATM.

8. Write short noies on the following ,

{a) One-to-onebiliing
(b) ComptrterTelephonylntegntion

{c) One-to-on€market,ng
(d) wAN

MS.42s@ t,oo0
Fs6rOl
MANAGEMENT PROGRAMME
Term-End Examination
December,2O05

MS-52€) : PROJECTMANAGEMENT

Time , 3 hours MoximumMarks: 100


(Weightage
70o/o)

Note :
(i) Atternpt an? three questionsJrom SectionA, each
corrging 20 morks.

Iti) Section B is conpulsory and corries 40 m'xrks.

SECTION A

1. "Project Management has assumed great significance in


modem organisations." Explain. What are the different
tools and iechniques used in Poect Managemeni ? 20

2 . Explain Project development cycle. How do you conduct


Pre F€asibility and Feasbility studies ? Give examples. 20

3. What are the different t!'p€s ot organisation structures in


projects ? Give a suitable structure for a road constructron
project.

MS-52@ P.T,O.
Discussth€ importanceof a good accountings!,stemfor
6uccessfulproject implem€ntatjon.What are ille factors
that afleci ptpject.costand how can they:be cotrtrolled? 20

(a) Discuss the usd of Cybemetica in Projeci


Managemenl. ..10
(b) Explain Contractsplanningand ContEct negotiation
giving examfiles. 10

MS-52@
SECIION B

6, "The time spent on preparinga DelailedProject Report


(DPR) is rellected in tfie successot a project." Explain.
Enum€ratevariousasp€ctsthat are coveredin DPR,

7. The tollowingiable givesdeiailsof a project I

NoImal Crash
Immediate
Predecessor Time Cost Cost
(week) (Rs.'000)(weeks)(Rs.;ooo)

10 20 7 30
B 8 15 6 20
c 5 8 4 74
D 6 11 4
E It 8 9 5
F E 5 5 4 8
G A,D,C 1' 3 8 4

Indirect cosi is Rs. 400 per day.


(i) Find critical path for normal duration-
(ii) Find ihe oDiimnl duratjon.
(iii) Rnd the minimum projeci cost,

MS.52@ 1,000
MANAGEMENT PROGRAII{ME
Term-End Examination
Decernber, 2OO5

RESEARCH
MS-51@ ; OPEBATIONS

Time: 3 hours MaximumMorks: 700


(Weightose
70o/a)

Note :

Ii) Section A hos liDe questions carrying 20 morks


eoah. Attempt anq three questions.

tiil Sectioh B is compulsory ond carries 40 ma*s

lli' Graph poper will be supplied on demond

SECTION A

L Explain the chaEcterjsiics of OperatjoJrsR€search. Dscuss

its importance iro decision making ptocess. Giue the

limitationsof o.R. 20

Ms-s1@ P.T.O.
Solve the follo$,ing Linear Prcgramming Problem
graphrcdlly
i

z = 4x1 +6xz
Maximise

Subtectto the constraint5

*r>2
x2s+

xt.x2>0 20

3 . A compaoy is spending Rs. 1000/- on ttansportaiion ol its


units from three planis to four dishibution centres. It has
calculatedthe cosi of transportation from different plants to
distribution c€ntres, giv€n below in Rs. The supply and
demand of unlts is also given below, Will the company save
some money by optrmumschedulingand how much ? 20
Distdbution Centfe. ,

Plant D1 D2 D3 D, A\'?ilability

19 30 50 t2 7
P^ 70 30 40 60 10
' 3 40 10 60 20 18

Requirements 5 8 7

MS51o
4. The annual demand Ior an iiem is 3200 units. The unii
cosi is fu. 6/- and jnvpnrory canyrng cosr is 25% per
annum. lf the co:t o[ one procurementis Rs. 150/-. flnd
out 20
(r) Econoorc Order Quanlity

lii) No ot orde)s per yPar


(iii) Time behi,€en tv"13consecuti\reord€rs
(iv) The optimal cost

Mention assumptionsmade, if any.

5, {a) Discussihe appljcationot dynamic programming in


decision making. How is this djff€rent from linear
programming? 10
(b) Explain Convex and Concave lunctions as related io
nonlinear programming- Discuss Kuhn - Tucker
conditions. 10

MS,51
o P.T.O.
SECTION B

6 . A self-s€rvice grocery stot? employs one cashier at iis


counter. Eighi customers arrive on an average every
5 minutes while the cashier can s€rve 10 customersin
5 minutes. Assuming Poisson distnbution for anival and
exponential distribution lor seMce rate, lind
(a) Averagenumber of customersin the system
(b) Avefage number of customersin queue-
(c) Av€rage time a cu.tomer 5p€nds in the syst€m.

ld) Average time a custom€r waits belore being served 20

(a) Find the saddle point in the lollowing case and also
lne game value. 10

^l:
B
14 11.l
s -111
-s 141
(b) Explain th€ need lor simulation in management
problems.Give various steps in simulationprocess'
giving€xampl€s. 10

Ms-51 1,000
o|
MANAGEMENT PROGRAMME
Term-End Examinatlon
December, 2OO5

MS-53O : PRODUCTION/OPERATIONS
MANAGEMENT

Time: 3 hours Maximum Marks : LOO

Note : Section A hos liDe questions corrying 20 marks


each. Attempt ony thrce qrcstions from
Section A. SectionB is compulsory snd carries
40 marks.

SECTION A

l. (a) Explain the variousfactorsthat are to be taken into


account for plant locaiion. Discussin connection with
settingup an electronicequipmentplant.

(b) What is meant by qualiiy assuranceand by total


qualiiy control ? How is each different from the
modern conc€pt of TQM ? ,6*19=20

MS-53@ P.T.O.
2. (a) Stopping waste is a vital t,art of JIT. Identify some
. sources oI u,aste and discuss how they may be

(b) A managet must decide b€tlreen thes€ altematives :

Machin?, Cost {Rs.)

4,00,000
3,00.090
8,00,000
Product lorecasis and processing times on tbe
rndchin€s ar€ as lollows i

Which machine would have the lowest total cosr, ano


how many of ihai machin€ would be needed ?
Machin€ op€rat€s 10 hou|s a day, 250 ddys a year.
10+10=20

Ms-53@
3. {a) What eltect does pull produciion hav€ on product
BOMs ? Why do products in pull production need
fewer'level BOMs than the same products produced
in an MRP type push syslem ?

(b) A mirnufacfurer of farm equipment is considering


three locations(A, B, and C) for a new plant. Cost
studies show that fixed costs per year at the sites are
Rs. 2,40,000, Rs 2,70,000 and Rs, 2,52,000,
rcsryctively, u,,hereasuariable cosfs are Rs. 100 per
unit, Rs. 90 per unit, and Rs. 95 per unit,
respectiv€ly. Il the plant is designed to haL€ an
effectivesystem capaci! oI 2,500 per y€ar and is
exp€cted to operate at 80 percent efficjency, what is
the most economic location, on th€ basis of actual
ol.rtpul ? 10+10=20

4- (a) What is group technology ? What is iis purpose ?


How is it used in product desigD ? Horr is jt used to
form product families?
(b) A work-sampling sludy o[ customer ,enice
repr€sentatives in a telephone company olfice
showed thai a receptionist was working 80 percent
of the ane at 100 percent performance rating.

t This rec€ptionisthandled 200 customersduting ihe


8 hour siudy period. Company poljcy is to give
allowancesof 10 percent of total on-the-job time.
Find the normal time dnd the standard time per
10+10=20

Ms-53@ P.T.O.
5. {a) What is the purpose of long,range resource
planning ? Whai information is need€d for such
planning ?
(b) You are to tesi the validity ol your forecasting
model. Here are the forecastsfor a model you have
been using and the actualdemandfor that period.

l 800 900
2 850 1000
3 950 1050
4 950 900
5 1000 900
6 9'15 1100
Compuie th€ MAD and tracking signal. Then decide
whether lhe forecdstingmodel you ha\e been using
is gjving reasonabler€sults. 10+10=20

MS-53
@
SECTTON B

6. (aJ What is the elfect of saleiy siock, safety lead tine,


and safety margin on inventory ? Discuss ihe
ellectiveness of these m€asures in oftseltjng
uncertaintyand processvariability.

(b) A company is about to begin production of a new


product. The manager of the d€pattm€nt that will
produce one of the components for the produci
wants to know how often the machin€ used to
prcduce the iiem wjll be avajlable lor ather work.
Th€ machin€ will produce ihe item at a rate ot
200 uniis a day. Eighty units will be used daily in
assemblingthe final produci. Assemb9 will iake
place fiv€ days a week, 50 weeks a year. The
manager estimatesthat it will take almost a full day
io get the machine ready for a production run, at
a cost ot Rs. 60. lnventory holding costs will be
Rs. 2 a year.

(i) Whai run quantitJ should be used to minimise


ioial annual costs ?

(ii) Wbat is the length of a production run in


days ?

(iii) During production, at what rate wlL inventory


buildup ? 10+10=20

MS-53@ P.T,O.
7. Write short notes on any tiue of the lollowing :

{a) Job enlargement


(b) Cause and effect diagram
(c) Intemal customer
(d) Flexiblemanufacturingsystem
(e) Design versus natural toterance
(0 Howe of Quality mairix
(g) lncrease o{ entropy
(h) Feedback conirol sysiem

Ms-53@ 1,000
l-c*ol
MANAGEMENT PROGRAMME
Tem-End Examination
December,2OO5

MS.54O : MANAGEMENTINFORMATION
SYSTEMS
Tirre : 3 hourc MoximumMarks: 100
(Weishtoge
70%l

Note I Attempt any ,hree questions from Section A.


Each question corries 20 marks. Section B is
compulsorg ond corries 40 marks

SECTION A

l. (a) Explainstructured
and unstruciured giving
decisions
e"\amples. 10
ib) Gle salient features oJ Simon's and Massie's
decisjonmaking models. 10

2. ia) Discuss the process of syst€ms design giling


10
(b) ''System
Developmentis a very important step in
desisning MlS." Explain. How do you design for
gmooih systemimplementation? 10

MS-s4O rP.T.o.
3. (a) "MlS project implementation needs careful
planning." Explain. Give various steps in
implementation. 10
{b) Djscuss expert systems and their application in
decisionmaking. 10

4. (a) Discuss the functions of daia communication


software.What ar€ multiplexersand concentrators? 10
(b) Explain the use of DBMS sofhrare. What are ihe
maih criteria for selecting a DBMS ? 10

5. Wtite fioteson any fout:


(a) Compui€r neivnork
(b) Query languages
(c) Computer r,rrus
(d) Legal Dimensionsof Computerisation
(e) RDBMS

MS-54@
SECTION B

6, Read the case study given below and answer ihe questions
given at ihe end ol the case-

KLM lnternaiional is a medium sized company in


India with about 1500 employees. The company
manufactures special machines for th€ agro-based
indlsiry, boih Jor farms and lood processingplants. The
company buys materjal and componentsfrom about 200
@ndors located in lndia, Japan and Gemany. Products
ar€ sold either io wholesol€rs or directly to clients (from
a mailing list of about 1500 customers).The businessis
quite rompeiitlve.

The company has the foilowing inJormationsyslems


in place: marketing, iinancial accounting,engineering,
research and dev€lopment and manuJaciuring(CAM).
These slstems arc inAependent o[ each othet and od9
the financial accounting system is on a tAN.

The companyis having proliiabilityproblems.Cash is


in high demand and short supply. lt is proposedthat the
company explores ihe possibility of using inlormatio[
technology to improve the situation. However, Vice
President of finance objects to the id€a, claiming ihai
most of the tangibie b€nefits of informatjon fechno)ogy
are already being realized.

MS-54@ P.T.O,
Questions :
Assxm€ you are hired as a consultani to th€ Presidenl of
KLM lnternational-

{a) Prepare a list of potential applications of informalion


technologles that you thlnk could hdp the company.
(b) Suggest a suitable d€sign of informalon svstems.
Would you recommend any t€lecommunication
arrangements? Justib such de'ign.

MS-54@ 1.000
fMs55@I
MANAGEMENT PROGRAMME
Term-End Examination
December, 2OO5

MS-55@ : LOGISTICS
AND SUPPLY
CHAIN MANAGEMENT

Time : 3 houts MoximumMorks: 1O0


(Weightage70Ya)

Note : Sectlon A hos six questions eoch corryhg 20 marks.


Attempt any lour questions. Secf,on B is
compulsory ond conies 20 morks.

SECTION A

1. It is said that the or,erall aim of logistics is to achieve high


customer satisfaction or perceived product lalue. This must
be achigved with acceptablecosts. How would you find the
besl balance ?

2. Explaln the t,arious issuesin logistics with sp€cial relerence


to transportation. What are the stages tor selection of the
appropriale iransport mode and why ?

MS-5s@ P.T.O.
3 . Distinguish behreen independent and dependent d€mand
inventory systems. Why is conventional inventory control
system not practised lor dependent item mat€dal
planning?

4. What were the reasons for the fall of management


accounting ? Explain activity based costing mentioning its
benefitsover managementaccounting.

5. The mosl cormon melhod lor evalua-ingnon.economic


factors in a faciliry location study is to use a scoring model."
Why ? Justify your answer.

6. Can a supply cha'n be both elficient and responsive ?


Risk-Hedging and Agile ? Why or Why not ?

MS-5s@
SECTIONB

7- Read the follo{ring case and answer the questions gjien.at


the erd oi the cade :

Supply Chain Sccc€sses - Mumbai Dabbatvatas

The DabbawaltasoI Mr-rmbarare d unique (ase o[


Logislr.s. NedrlyslackeCdobbos lTlfin. Lunch Boxesj are
a common sight every ]Ete .r]-oming at Mumbai, in lndia.
A man who is generally i)literate deljvers hot lurch at the
doorstep of the subscriber.'Ih€r€ are hardly any cbances
that lou will get a wroDg dabba. 'lhere ar€ millions of
offic€-goeB in Mrjmbai and instead of carrying lhe Juncb

I at an earlg hour in the moming they prefer to subscdbe


to dabbd service. For a small le€ the dobbabala TrcUsup

t ihe freshly packed lunch from the subscriber,s house and


deliversit to hisAer office at lunchtim€. Once lunch is
over, the empty dobbo is again collected by the
dabbowalo. This is done with the help oi Mumbai's
extremely efficient railwag system called .Mumbai Locals'.
There are special trains known as the dobbowala special.
The dobbos change many hands and are load€d and
oflloaded in many trains betore thetr final delivery. The€
is a scientific way of puiiing an identilication mark on th€
dobbo. Each dobbd lid is to be marked with a particular

Ms.55@ P.T.O.
' code. .For example NXXF wher€ N denotes the
dabbawala's number (assigned by the association), XX is
a combination indicating ihe btilding and F is the floor
nu,'nbe; of that building where the tif{in-box has to be
deiivered. The M is also marked with a number denoiing
the railway station wbere the Tiffin-box has to be
off-loaded, followed by an alphabet indicating the station
of pick up. Can you imagine all this is done lor what
fee ? The senice charges vary beiween Rs. 150 (US$3)
io Rs. 300 (U9$6) per monih, deperding on the
custonrer's location {and the distanc€ covered).

A Parsi banker started this when he ernployed a


cariff to fetch his lunch every aftehoon. The idea
caught on and this in6piied many uremploy€d people to
becone dobba carri,ers- Soon each dabbor.mla had a
handful of customers. To ensue that each carrier worked
only in a particular distdct and didn't intefer€ wilh other
dobbaudllas, a union called 'M\rmbai Tiffin Box Carriers
Association' was lormed in 1968. Today thbre are more
than 5000 serniliterate "'Dabbauallos" \rhich trajFport
1,75,000-boxes in a 3-hour period, through 25-km of
public transportation involvlng mulliple ilanster points. ln
1998, Forbes Global magazine conducted an analysjg and
gave them a SX SIGMA BATING OF EFFICIENCY.

MS-5s@ 4
Qu€stions ;

(a) Define the inputs, process and outputs of the


Dobbawallos.

(b) What are the other businessesin India lhat can learn
lrom Dabbawollos?

{c) Can you suggesi measuresfor betterment of service


to Dobbowollas?

Ms-55@ 1,O00
MANAGEMENT PROGRAMME
Term-End Exarnination
December,2OOS

MS-56 @ r MATERTALS
MANAGEMENT

Time : 3 hows MoxlmumMorksr 100


lWetghtoge70Vd

Note t There orc eight questions in this questlon poper.


Attempt any floe questions. All questlons cotry
equol morks.

1. What is the scopeof materialsmanagement? Defire tl€


rarious roles ot mateids Danagementin the context of
intemal and externalinterfacesto a materialsmanagement
system.

2. Why is negotiaiion an important asp€ctof purchasing?


Describ€the elementsand obiectivesof negotiation.Also
dlscussthe negotiationprocessand techniques.

Ms-56o P-T,O-
3. Briefly expiain th€ reason for the evolutiori of
manufact.ring resourcesplanning (MRP I0 hom rnaterial
requirementsplanning (MRP). How does MRPll differ
{rom MRP ? Exptainthe differencewith the help of a flow
diagram.AIso expiain how itrst-in-tlrneproducdonsystem
(JlT) ditfersfrom MRP ll.

4. CONWIP ls considered applicable to production


endronments characterized b!' longer s€t-ups and
flucluatingdemand.Why ?

Discussthe various lossegin \rarehouses.What are the


reasonsof their generation ? Discussihe procedure of
pre\€nting and controllinglossesir a warehouse.

6. Explain rhe Supply Chain OperationsR€ference(sCOR)


frameworkfor performanceappraisaland how it is uselul
in materiaismanagement.

? . What are the measuresyou recominend to rnainlain


inventorysecurt&in stores? Digcllssin brief th€ problems
and their rem€dieskl caseol valuationof {inishedgoods
and work-in-prccessinventory.

Ms-56@
E. Write short noteson any tour of the follooing :

(a) TQM in materials


management
(b) Needfor internationalpurchase

(c) OplimizedProductionTechnology(OPT)
(dJ h'anban
(e) EffectivePoint Advi6e(EPA)

{0 Mahices of performance appraisal in materials


managarent

MS-56o 1,000
tMss?Ol
MANAGEMENT PROGRAMME
Term-End Examination
December,2OO5

MS-57 O : MAINTENANCE
MANAGEMENT

Time i 3 hours Moimum Marks: 700

Note , This questionpaper cantoinsserenquestions.All


questionscarry equol morks.Attempt ony fiue
quesuons,

1. mat is preventive maintenance ? Explain different


preventive maintenance tasks. How is the total rnaintenance
cost affectedby lhe levelol p)eventivemaintenance?

2. What is the aim of spare parts inventory management ? In


what way.doeg it influence the different stages of th€ life
cycle of spare parts ?

3..Many organizations have standing con{licts amongst


maintenance and production/operations employees.
' Describe the nature and causesof these conflictg.

MS-s7@ P.T.O.
4. t{hat is the short-term determinlstic or un;t z}achine
problem ? How do you take replacement decisions in case
o[ equipment/componentwhose operating cosi increases
with use ?

What ar€ the rafious techniques, whicb can be used for


scheduling and monitoring of projects ? Why is neh,,/ork
analysis preferr€d ? Discuss their advantages over other
methods. What is the basic difference b€tq/een CPM and
PERT ?

Exptain the basis ol identilying and analyzing a problem,


while making use of temperature monitonng technique.
Name some of ihe instrum€nts used for temperature

Write short notes on any torr of the lollowing :

(a) l,'nportant features ol Totdl P{oductive Maintenance

{b) Zero-basedBudgeting

(c) System R€liability in System with Components in


Series

(d) Gamma Distribution

(e) Prerequisiiesfor Autonomous Maintenance

Ms-57@ 1,000
MANAGEMENT PROGnAIfttE
Term-End Examination€
December,2OO5 l

MS-58O : MANAGEMENTOF R&D AND


INNOVATION

MoximumMorks: 100
(Weightage70Yo)

Note. Attempt .ttt quations. All questionscarry equol

1. Explain the following techniques of enhancing/maintaining


th€ creativiiy :

(i) Brainstofmihg.

(ii) Syneciics

OR

What are the reasons lor technotogical inertia, pr€\renting


implementation ol new technologies ? Give some
examples because of which suspicions about newer
technologiesarc rampant.

Ms-58O 1 P,T.O.
2. Discuss and critically examine the \arious inc€ntives and
measures provided for in the Gov€rnment policies for
promotionrff jF]p?qrrt of R&D in lndia. What additional
measures,ifAy. have be€n laken over the past 5 - 7 years
qt- ^

i o R
What are the different types ol R&D organizations ? Wbat
kinds of R&D organizations are found in India ? Also
discussthe functions of R&D organizations

3 . Briefly describe, the following models :


(a) The Adair Model
(b) The Blanchard Model

on
Write short notes on the following :
(a) Dfferent hierarchical models of R&D organizadors
(b) Matrix organization and its advaniages

{c) Le-drningorganizalion

4. Discuss the salient features of the incentives and support


measur€sprovided by th€ following countries for promotion
of R&D technology development :
(a) France
(b) Malayasia

OR

MS-s8o
In the context of national R&D infrastructure and
institutionalfuarnerl?ok,lvrite notes on the following :
{a) D€partoent of Biotechnology(DBT)
{b) Depadmehtof OceanDevelopment(DOD)
(c) Depaftmeniol Scienceand Technology(DSTi
(d) Depanment oI Scientific and indusirial Research
(DSrR)

5. Write short notes on the {ollowing :

(a) Good practices in Technology Transfers

(b) Sources oI Patent lnionnaiion in India

(c) SPREAD

OR
l{&at is GATS ? How can jt lacjlitate accessto technology
and Technical infolmation for developing countries ?

Ms-58o 1,000
tr*.@l
MANAGEMENT PROGRAMME
Term-End Examination
December,2005

MS-61@: CONSUMER
BEHAVIOUR
Time : 3 hows MaximumMarks: 100
(Weightase7\o/o)

Noie : This poper consisis al two Sections A ond B.


Attempt any three questrcns Jrom Se.tion A.
Section B is cor'rl,ursory. All questions carry
equol marks.

SECTION A
l. Explainthe VAIS 2 Systemof lifest!,leclassification.How
can .ihis approach be used io develop promotional
strategieq
lor
(a) A health club
(b) Luxury cars

2 . What are the factors thai influence the retri€val of


information from long-termmemory by the consumers?
How would you apply the knowledgeof these faciors in
your advertisingdecisions?

MS-61
@ . P.T.O.
3. . What do you understand by socral class and social
stratificafion ? How would you as a marketer oI the
following- productg usa your knowledge of social class
influence io market your products ?
(a) Motorcycles

{b) Domestic appliances like washing machines

4. What is the role ol eGluative criteda in consumer decision


process ? List the e!.aluatlve critena that in ylour opinion
first time buyers are likely to use in the purchase of
(a) A Reridenhal house
(b) A lif€ insurance policy

5. Write short notes on any tltee of the following :


(a) After Refenal Choice heuristic
(b) Bettman's information processing model

{c) Classical conditioning theory ol consumer leaming


(d) Factors influ€ncing information search b€haviour
(e) Organizationalbuyingb€haviour

Ms61@
SECTIONB

6. Readthe casegivenbelowand answerthe questlohsgiven


at lhe end of the cdse.
Bayet Aspttiri't Une Extenston

l For many years, aspirin dominatedthe market for


non prescriptionpain\relief and Bayet aspirin dominaied
I the aspirin market. However, in recent yea6,
acetaminophen- and ibuprofen-basedpain relieverc
have taken over much of the market. By 1989,
aspirin-trasedproductsheld only 40 perceni of the total
abalgesicsmarket. Thts dropped to 35 percentbv 1992.
Bayer hns abouta 6 6 perceni shareol the total analgesic
'r.a:Jretand 79 perceni of the aspirin market.

F" Cornpetjtjon jn ihe analgesics matket is intense.


Ttpre arc three main type of analgesics - aspirin,
L. acetaminophen, ard ibuprcfen, There are several
adr€rtised brands within each lype ol analgesic as well as
private lab€l and store brands.Product d"terenceswiihln
analgeslc categories afe limited.

The int€nse cornpetition has glven ds€ to product


proliferalion and niche strategies. All the ptii,aie label and
store branG competeon price. They may sell lor hal, $e
price of the natlonal brands. Advil is the leader in the

I ibuprofen category with a 50 percent share. Motrin wjth


a 15 Fercent share has used three different commercials

L Me61@ P,T.O
to targ€t backache, arthritis, and headache pain. It
atlempts to "maintain ih€ brand's appeal as a general
analgesicwhile r€dching oul to snecr{i( groirps ol pdrn
su{{€rerslhroush ddvpnislng. The slfategyappearsto be
working as its shar€]s growrng.Nupr'n (13 percenl share)
has aiiempted to comp€tewith a locus on muscleaches,
using celebrities such as Jimmy Connors. MichaelChang,
and Joe Moniana.

Similar niche slrdtegie\ are dpneaflng in the


acetaminophen and aspirin categories. Acetaminophen
based Midol is attempting to posiiion itsel{ as "the
menstrual r€hef s)recraljsr'.li tLdher iocu.es wjlh su.h
line exLens)ons
ar lvlidolPM NrghflimeFormuraand Mdol
lB Cramp Reliel Foimula. Tylenol is increasingly
positionedin terms of arthr'tispain reli€f though it Is also
widely used for headach€relief.

Recent medical lindjngs indicatethai the regular use


of aspirin helps certain heart and coloo conditions.Bayer
jntroduced Therapy Bayer {or this application. but aspirin
saies in generaL and Bayer asp rin ;ies boih continue
rh€ir relalive decline.

Excedrinwas historicallybehind Bayer in the asplrin


category. Hqw.ever,it now has a greater total markei
share in the overall analgeslcsmarkei. lt has managedlo
grow ils mdrkel share by ' : aggressively
- adding line

MS€]@
extensions : ibuproJenbas€d Excedrin IB and
acetaminophen-based
ExcedrinAF and Excedrin PM.

Bayer miinagement is consideing introducing


non aspifin-basedanalgesjcsusing the Baget name,

Questlons :

{a) If the company wants to proceed with the line


extension, what leaming principles should they
apprya
(b) If yoLr were asked to develop an ad for Bayer's
ibuprofen'basedanalgesic,what perception r€lated
knowledge would you use to develop the ads ?
(c) Explain the aititude chang€ principles that would be
relevant to Vou in the development of the above
advertisements,

MS-61
@ 1,000
MANAGEMENT PROGRAMME
Term-End Examination
December, 2OO5

MS-62@ : SALESMANAGEMENT

The : 3 hours MoximumMorks, 700


(Weightage70%)

(l) Attempt ony three questionslrctn SectionA.

(ii) Section B is compulsory.


(iit) AII questtonscorry equol marks.

SECTION A

1. {a) Discuss the significance oI Inlormation System Iot


sal€s administration.

(b) Discussthe factors you would keep into consideration


for developing a proper information system tor sales
effectivenes,sby iaking the example of a consumer
durable like water purifier.

Ms-62@ P.T.O.
2. la) Briefly discu-cs ihe Interpersonal communication

(b) How can an understanding of Transactional Analysis


help in handling a customer ..,,,hoconlmuously lteeps
asking irrelevant questions durlng a saies
pfeseniation ? You may assLtmethe example oI a
m€dical tepresentative who is interacting with a
medical professional or any other similar situation.

3. (a) Briefly explain the issuesinvolved in the perforrnance


evaiuation of the sales personnel.

{b) What performance elaluation mechanism would you


propose for those sales personnel who are not
involv€d in direct sales but are primarily involved in
promotion of the product, like a publication hous€
hiring sal€spersons tor promoling the text book
amongst leachers ?

4. (a) What are the advantagesoJ setting sales quotas ?

(b) Brieilg e)tplain how sales quotas are set. You may
assume example of any company to answer this'

MS-62@
SECTION B

5. Caretully vead the [o ou,ing case and answer rll9 questjons


given at the end oI ihe case:

CASE
.
Anupama Cosmetics (p) Limited was one of the
leading manufactur€rs of soaps, tacial crea&, shampoo.
hair cleam, Iipstick and toilet powder in fndia. These
products were extensively advertised in all the leading
newspapers, iournals and magazin€s in the country_

The markeiing organisation of the con)pany was


headed by a marketing manager who had under his
controi six regionai sales oftices siiuated in six principal
cities of India. Each regional oJlice was in charge of a
sal€s supervisor who was responsible for execuiive
direction of a number of salesm€n. ln 1997 morc than
slxty salesmen operated oui of six r€gional sales olfices of
the company. The sales sLrp€r,,isorssubmitted monthly
l"ports to the marketing manager on the progress of
sales and performance of salesmen in their respective
regions.

Selection oI Salesm€n

The management of the company strongty believed


that intelligent selection of sales personnel was hjghiy
necessary for improving the efficiency of business. The

MS-62@ P,T.O.
Markeirng Managet was particularly anxiorLs to see tnat
ih€ salesmen oI the comapny possessedthe following :

(i) Good Appearance : A general grooming ol the


individual to make sure that he would noi dev€lop a
hurdle o{ sales lesistance bv his shabby appearanc€

(ii) Personaljty: He shouldhavea w€ll developedphvsical


as well as mental PersonalitY

{ri) A good speaLinqvoice and elfecliveexpr€ssion

(ivl Modente habits, good health and physical fitness

(v) Alertness, lriendiiness and pieasing disposition

(vi) lntegdty, hotesty, and pers€vetance

(viil Ability to get along with other p€ople and wiltiogness


- to \tork hald {or the comIEnY

Aft€I selectron each salesman was given intensive trainlng


at the head of{ice for a petiod of three morihs Thereafter
he was posted in one of the company's regional sales

Dufies ol Salesmen

Salesmen o{ ihe companv w€re required to per{orm the


lollowing duties loi' the company ,

li) io ca[ on customersol the company and bing to their


notice the products of the company and book orders
lrom them;

M s - 6 2@
(ii) to contact retailers including druggists, general stores,
and others and study their changing attitudes iowards
the poducts of the company and report ihem to their

(iit to call on dealers regularly and help them in ananging


display of the comapny's producis:

(iv) to discuss new product selling features, new policies


and campaigns of the company with prospects;

(v) to coopehte with the stockists of the companyi

(vi) to aci according to ihe terms. conditions and policies


of lhe company and lo obpy the o)ders of sdles

{vii) to nrake /eports and carry on coffespondence and


submit expense accounts to the company periodically
accordtng to the instruciions given; and

(viii) to carry out miscellaneousactivities as might be given


to them from time to time by their superior officers.
Situation in 1997

Anupama Cosmetics (P) Limited, which enjoyed


almost a monopolv position for a number of years,
suddenlyfaced compeiition h the markei in 1996. The
conpetiLion mainly cdme fro-n rhrpp newl,r establshed
foreign companies.Thes€ companiesemphasisedquality
and price of lheir producis.Theg spent a large amout on
advertisemenland sales promotion. In addition, each of
them had an etfectiv€salesorganisation.

(s
MS-62 P.T.O.
During this period the company also faced certait
Iabour problems in the factory. Due to workers' strike the
factory had to be closedfor a few wee!,sin the beginning
of the year. In addition, due to rise in excise duties
imposed by tbe Government, ihe company had to
increase prices of some of iis products. The markeling
manager realis€d that market conditions were constantly
changing and *lat in order to improve the company's
sal€s pe ormance some vigorous elforts were needed on
the part of lield saleslorce.

Questiors :

(d) Comment on the qualihesot ihe salermenwhich Lhe


marketng manager was interested in and also the
duties assigned to the sales people.

(b) Do you think these q.ialities and responsibilities are


adequaie in the changing circurnstances?

(c) What specific additional responsibilities should be


assigned to ihe salespersonsas per your advice ?

t\4s.62
@ 1,000
tr;'@l
MANAGEMENT PROGRAMME
Term-End Exaniination
Decembet, 2OO5

M S . 6 3 O : P R O D U CMANAGEMENT
T

Time : 3 hours MaximumMarks, 100


(Weishtoge
700/0)

Note : This paper consists of tuo Sections A and B.


Attempt an\ three qrestions lrom Section A.

. Seciion B is conpulsory. All questions carry


equol motks

SECTION A

l. Whaf is the matix apToach ta Votuct pbnning ? By


taking an illustrative example, explain the suggested
markeiing strategy for a multi product company on the
basisof a productevaluationmatrix.

2. Why is environmental analysis important for product


manager ? For a product categorg like air-conditioners or
washing machines, explain the environm€ntal factors you
would need to monitor,

Ms-mo P.T.O.
3 . With respect to any new product idea of your choice,
explain the proces,sof concept development and iesting.
What is the significance of this stage in the new product
developmentprocess?

4. How is prjcing atlected by ihe other elements ol the


marketingmix ? How do factorslike competition,siage in
the product lifecycl€ and costs affect th€ pricing decision
for a new product ? Explain wiih the help of examples.

Explain the piocesses of product audil and produci


augmentation.How does the und€rstandingoI these tt o
concepts enable a marketer to finalize th€ attributes of a
Proposednew product?

MS-m@
SECTION B

6, Read the case given below and answer the questions at ihe
€nd of the case.
The "Success" Story

A distribution house is handling a number of


consumerproducts of reputed companies,aci'ng as iheir
selling agent. Thg producis were mark€d by ihe company
und€r the brand nam€s of their respective manufactuing
concerns.The companylost a profitableagencysincethe
distribution of ihe products \^Bs taken over by ihe
manutaciurerhimself.

Based on iis past experience on sjmijar ljhes, the


company inirodlrced a tooih-paste in the market
manu{actured by itself, under the brand name of
''Success".
At ihis point of time, C'was lhe brand leadet
in the tooth-pasiemarket follow€d by 'B' and 'F . When
'Success'
in 'Larg€' size was launched iniijally, the
. distrbulion $ias lantastic,in both ihe rural and the urban
markets. The coveragewas extensiveand the consumer
Iesponse was very encouraging.The irodLrct was much
in demand in all the reiail outlets, including pan shops.
However, after some time, ihe company started getting a
number of complaints related to packaging. The caps
trsed were unable to \rithstand the transportation, due to
which the contents from the tub€s were leaking out.
Without tully rectifiJing the defecis, {he company came

MS-m@ P.T.O.
out in the market with bigger sizes of tooth pastes like
'Gani' 'Eeonomy'
and undey the same brand name
'Success'.
Simultaneously, th€ company introduc€da new
range of products lik€ talcum powder, hair cream, tooth
brush and shalifig cream. Al1 these products were
manufaciured by the company itself. At ihe time of
launching these products, the company was already
mark€ting shaving cream and tooth brush under differmt
brand names \rith regular trad€ schemes. But the
companydid not operat€any attractivedealerschemefor
the 'Soccess' range o{ prducis and the sLdden spu in
the number of produc[shandleddid not hav€ the requisite
adverijsingbackup.

The company b€ganto notice a suddendeclinein the


sales ol newlg inkoduced products. At this stage, th€
company also introdutpd the Success rooth powder in
two different sizes.During ihis period the suppliesol the
brand leader 'C' werg inegular and restricted.The other
'F'
fwo tooth-paste manufacturers,viz., 'B' and did noi
have tooth powder. Therefore, ihe Success' tooth
po\rder started moving wel in the market.

The company again launch€da n€w serieso{ similar


products except tooth-pasie und€r a new brand name
'Victory'.
This tumed out to be an utter failure due io
poor marketing strategies,which adversely'atfectedthe
company'simage-

Ms- @
The company appointed a new protessionai
marketing chieJ, who decided to go in for yet anoth€r
'Success'series
tange of cosmeticproductsgroundingihe
of products, orrt ol the rnarkei, The new venture also
proved io be futil€ exercise, and ihe 'Viciorg' range ot
productsalso had lo be dbandonedfinallg.

(fuestions I
'Success
(a) How do you explain the failure of the
range of products ?
'Vjclory'
(b) Is the company justified in introducing ihe
range of products ?

{c) Do you find an9 organizational weaknesses in the


functioniogof th€ company?
(d) Suggest a marketing plan to enable the distribution
'Succ€ss
house fo tum around the performance of

Ms-63€) 1,000
MANAGEMEM PROGBAMME
Term-End Examination
December,2O05

MS-64@ : INTERNATIONAL
MARKETING

Ilme : 3 hours Maximum Marks : 700


(Weightsge70Vd

Note t Altempt ony tht.ee questians from Sectlon A.


SectionB ls compulsory. All quesrpnscorry equol

SECTION A

l. {a) Dillercn ale between Absolute ad\Entages vs.


Comparative advantagein international marketing,

{b) Describe various reasons for a firm for entering


internaiional markets.

2. la) "An ittetnaiional marketer has to find out a tade-off


between standardisedand customised products as it is
diificuit to evolw a global produci." Do gou agree
with the above statement ? Justify your anslver wilh
sujiable examples.

Ms-64@ P.T.O.
(b) What are the Jactors infurencing international
marketing communication decisions ? Jusiity your
answey\rdth suitable illustrations.

(a) Explain the concept o{ Iniemational Producl ljte


Cycle. Evaluatethe relevanceof IPLC concept for the
following
(il Bicycle
(ii) GenericPharmaceuticalProducl

(b) Explain the concept oJ Mafgjnal Cost Pficing. Give


reasons for its jmplication in intemauonal rnalketing
visa-vis dom€siic marketing.

4 . Write short noies on an; three of the {oltowmg '

(a) EPRG Concept

lb) Evaluation ol country risk

lc) Influence of culture on international marketing

{d} Foreign axchange market

Ms-64@
SECTIONB
5. Read the case given below and answer the qLrestionsgi!€n
at Ihe end ol the ca-se:

Giilette had bgen a pioneer in the shaving category


since 1900 and had sinc€ been the v,/cddbader jn tbis
category. Cillette started its operation in lndia in 1968
and sells many t:lres of ftzots, razor blades, shaving
creams, deodorants, toiletries, and oral care products. The
market slnrcture for many ol Giilette's products had
mainly been rnonopolistic with many producers in the
Dayket. Technology and product dilferentiation play a key
role in marketing of razors. B€sid€s, Gilletie spends
heavily on adveriising and sales promotion.

Most of the Gillette products are inierdependent in


demand, productio; and consurnption. For instnnce, razor
blades and shaving creams are complirnentary with raaors.
The demand [ot razors ls price elastic and any reduction
in price of razors tends to increase its d€nrand which has
djreci eflect on total revenue of complimeniary products
such as razor blades, shaving creams and shavjng lotions.
Gillette's sttaiegy has been to keep the razors' prices low
and the p/ices of razor blades and shaving creams
relatively high. Razors contribute very low rewnue to the
company's lumover \r,,hereas ta?or blades contribute
almost 90 percent of the cornpany's fumover. As Gillette s
marketing strateg!, price reduciion on one brand of ra?ot
increases its demand while it cannibalisesihe demand for
other ra?ors in th€ similar categorv sold by Gillett€. The
company keeps a constant vigil on the market and
monitors ef{ect oI pricing decisions on substitutes aDd its
cannibalisaiion impnct on its other products.

Ms-64@ P.T,O.
cillette India has a market share o{ 60 percent in
bladesand razorsin Indla. The compositionof Sri Lanka's
sha\rng market (Fig. 1) suggeststhat disposablerazors
accountedfor 679o ol markel share followed by double
edg"d blades(32%o) and twin blades(1%l in 1999. As
indica&d in FiS. 2, Gillette's market sharewas a meagre
7% while BIC dominatedthe mark€twiih 91% market

Composttion o{ Srt lrnka's Shavirg Market


(re99t
Twin Blades,
1%

blades,32%

FiCiute f

Compositiot of Di.posable Razore in


srl lankn (1999'

SuparMac,
oth€r 1% ---- !%t

Btc,91%

Figu'e 2

MS-64@
Analysis ofmacro economic data indicates tha'i
sri Lanka has a populationof 19 million that is growing
at 13 percent annually,with a p€r capita CDP of
US$ 850. The literaca rate in Sri tanka is quite high at
92 percer\t in the South Asian region. The male
population in Sri Lanka is about 52 percent. Out of them
about 60 percent mal€sare above 15 years of age. This
gives a broad indication of good mark€t poiential in Sti
Lanka. Gillette's meag/e share ol 7% in value ierrns is a
matter oi serious concern for the firm and the top
management.

Questiond ;
(a) Work out in detail the type of information requred for
conducting the mark€t res€archin Sri Lanka. Also list
out specific sources of compiling secondavy
in{ormarion.

(b) Prepare a research plan to identify the Gillette's


meagre market share of 7o/ain disposable razors in
Sn l-anka.

Ms-64o 1.000
I
I MANAGEMENT PROGRAMME
Term-End Examination
December,20O5

MS-65O : MARKETINGOF SERVICES

Time: 3 hours MaximumMarks: 100


(Weishtose70o/a)
Note . This paper consistsol three SectionsA, B and C.
Section A is to be attempted by students
resisrered lor this cource lot July 2004/Januory
2o05/July 2005 semesters. Section B is to be
ottempted b9 students rcgistered for this cource
Jor Jonuory 20U semester. Section C is to be
1ttempted by oll the students. All questiorc corry
equal marks.

SECTION A
Attempt anv thrce questions.

l. (a) What are the implicationsof the servicesmarketing


triangle ? Explain gMng suitableexamples.
(b) Distinguish betueen the following with the help of
exirmples :
(i) 'Search qualiti€s' and 'experienc€ qualiti€s'
(ii) 'Faciliiating services and supporijng services'

MS€5o P.T.O
2. (a) Taking the exampleof a hotel, explainthe Gronroos
Modelof ServiceQuality.
(b) What do you understand by the term Yield
Managem€nt ? Discuss th€ importance oi yield
management
for an airline.

3. (a) Explain the term 'word of mouth communication'


and discuss its significance for a health selice
provider.
(b) Discuss ihe importanc€ of location decisions for
ealucational
seMces.

4, Write short notes on any ,hr€e of the following I


(a) Trends in international trad€ in services
(b) Roles of physical evidence
(c) Importance oI customer ret€ntion
(d) Extended marketing mlx for hotels
(e) Classification of product support sennces

M5-65
(t
SECTION B
Attempt ony three guestions.

5. Discuss the importance oI the following lor a service


providey,giving suitableexamples:

{a) Physicalevidence
(b) lnvesiing in 'people as a source of competitive
' advantage

6. (a) Describesome of the n€ys of measuing crjstomer


expectationsfor servic€s.What are the indicators
that you would useto m€asureexpectatjons?
(b) What are the issuesthat shouldbe in consideration
while brandingfinancialserviees?

(a) Ouiline the key issqes in promotion oI educatioD


services. What, in your view, would be an ellective
promotion mix for a management institute ?
(b) What are the oajor ways of segmenting the tourism
rnarket ? Explaln-

Write short notes on iny three of ihe {ollowurs I


(a) Reasons for grov,,th of ihe service sector

{b) Senjce process


(c) Posiiioning strategies for advertising agencies
(d) Relevance of marketing in logisiics industry
(e) Serlice quality

M5-65@ P.T.O,
SECTION C

9. Studg the case giv€n below and answer the questions giv€n
at the end.
Wodd c!'m

World Gym began operationsin 1995 in a large city.


The company'sobjectivewas to meet the fitn€ssneedsof
a div€rse c):enlele, lrom the prolessional body builder to
the overweight person,

Wortd G9m's pricins plan was to have a {]lness


Iacillty that targeted ihe common person - a fitness
facility that was not on the high end or the low €nd, but
in the middle price range. ln the beginnlng it pianned io
challenge the price of the top-of-the-line facilities. lt
olfered its seNices in a big spacious cenFe in a
high-t6ffic arca, a part of citv that was becoming
genlrilied. There was linle comperition in the area. The
establishment's large space, with natural llghi coming in,
set it apart from competiiors.

The company expected its m€mberc to come Jrom


other clubs and facilities not only in ihe immediate
ne'ghbourhood.bul all around rhe cjty. lt saw its lracje
area as the neighbourhoodswithin an 8-10 minute
drive.

MS65@
The company decided to begin by marketing its
product to people who were already working out but
wanted somethingunique.When the company opened its
fiiness facility, consumersreadily acceptedit. World Gym
membershipfar €xceededexpectations.It now has 3,000
m€mberc. As many as 2,000 people come on a given
day.

For years World Glm had few seriouscompetitors.In


the city, permits are required io open a glm and thus are
an obsiacle to potential competitors. And the cost of
opening a 35,000-square-footfacility is an additional
barrier to entry,

Qrr€stions :

{a) Was World Gym's pricing plan appropriate in a


market where there was a relatively low supply ol

(b) Over t'me World cym's membersh,p exceeded


expectatjons, Demand was strong and constant.
What impact might this situation have on the
company'spricing strategy? On product strategy?
(c) Suppose the company learns that two new
competitors plan to open liiness faciliiies within a
mile of Wodd Gym. How migbi the increasedsupply
of servicesaflect World cym's pncing policy ?

Ms€s@ 1,000
MANAGEMENT PROGRAMME
Term-End Examination
December.2OO5

MS-66G) : MARKETING
RESEARCH

Tlme: 3 hours MaximumMarks, 100


(Weishtage70o/o)

Note t This poper consisis oJ two Sections A and B.


Attempt on9 ,hrce questions lrom Section A.
Secfion B is compulsory. All questions carrg
equal marks.

SECTION A

f. (a) What are the dilferent data scales? Give examples


of each of them. How do they aff€ct ma*eiing
researchprocess?

{b) Whai is Principal Component Analysis ? What is


the m€aning of rotation in Factor Analysis ?

MS-66@ P.T.O.
2. . (a) Which research design strdll you r€commend for
profiling the potential customers of a fast moving
consumer good ? What would be the k€y steps for
such a research ?
(b) Wbar ,s patjrjpdtion observalionmerhod ol dara
collection ? What are the , special precautions
requiredfor conductingsuch res€arches?

3. (a) What are the special considerationsrequired lor


conduating a'readership surv€y as compared to any
other product'susershipsurvey?
.
(b) What are the ditferent methods available for
conducting consumer motivation research ? Describe
any one ol tnem.

4. (a) What are the main secondarydata sourcesavailable


io lndian marketing researchers ? Describe th€ir
main limiiations.
(b) List the important "exieris" that you may like to use
in marketing research€sin India. For each ol the
€xpert categories id€ntified, give some of the product
categories that they would be specially surtable fof.
Give reasons.

5. Write short notes on any thtee qf the lollowing ,


(a) ProjectiveTechniques

{b) StratifiedSampling
(c) Multi Dim€nsionalScaling
(d, Indiancensusdata- a soLtrcefor markeilngr€search
(e) Consumerpanels

MS-66@
SECTION B

6 . Uttam Vahan Ltd. (UVL) is a major car manufacturing


company ol India. UVL has decidedto understandIndian
car distom€$ afreshin responseto increasingcomp€tition
They believe that the car purchase is not so much al{€cted
b9 the socio-economicand demographiccharacteristics of
pot€ntial customersas by their lifestyles.Therefore, they
have designed following questionnaireto captur€ the
profilesof potentialcustomers.

Q.1. Do you int€nd to buy a car within next six months'/

Q.2. Which brand and model of car do you intend io


buy?-

Q.3. Why do you intendto buy that car ?


Q.4. For what purpose do you intend to use the new
car ?
Q-5. Besides yourself $ho else in your {amib ate likely 10

Q.6. Your monthly income :

Q.7. Your occupation


,

Q.8. Your education:


Q.9. Do you oum : A car ?

A credit card ?

A mobile phone ?
A personal computer ?

MS,66@ P.l.o.
Q.10. No\x I shall read a few statemenis. Please indicate
v.rhether you "Strongly Agree", "Somewhat Agee",
"Neither AgreeNor Disagree","SomewhatDisagree"
or Strongly Disagree" with each oI them.
(a) Cars are the main reasons of pollution in the
counfy.
(b) Women should not be allowed to drive cars.
(c) Cars promote consumeristculture.
(d) Cars deny use of regular exercising.
(e) Cars are homes on ihe roads-
(0 Cars are luxuries that our country con ill afford.
(g) I enjoy long car drives with my friends.
(h) Govemment has taxed cars excesslvery.
(i) My car usageis very fixed and regular.
t) My car sp€aksabout me.
(k) The speed limit fixed for cars on Indian roads are
too cons€natlve,
0) My friends are e ,ious of my car.
(m) I am never afraid of accidentswhile driving.
{n) Cars bring ihe family closer.
(o) On today's congested roads it is belter to $alk
than drive.
{p) Your car createsyour lirst impression.
(q) I understandthe car rnachinery completely.
(r) CarsgreatlyfacihldLe socialisatlon.
(s) I love adventurousjo$neys.
(t) Cars are my fiIst love in li{e.
Q.11.Yourname:
Q.12.Your address
:

Ms-66@
Assignment i
(a) Comment on the questionnaireused.

F (b) Outline an interview guide lor th€ above


questionnalr€.
(c) Outline an analysis p{an Jor the above questionnaite
i- based study.

MS-66@ 't,000
MANAGEMENT PROGBAIU|IIE
Term-End Examinatlon
December,20O5

MS-611@ : RURALMARKETING

Time : 3 hours MoximumMarks, 100


(Weishtase7@/0)

No,€ r This paper consists of two Sections A and B


Attempi ony three questions Jtort Section A
Section B is compulsory. All questions corry
equal marks.

SECTION A

1. Brie{Ly discuss the rural enviro$ment ir} terms o{ the


infractructure available for markeiing. How does this
environment affect the marketing decisions and
opportunities ? Explain with refer€nce to

(a) N4arkeiingol bankrngservices

(bl Marketins ol consumer n€cessities iike toilet soap


and detergents

MS611@ P,T.O.
2 . Disc'-rssthe major dilferencesbei..rcen urban and rural
market res€arch. You have been ask€d to assessth€
market acc€ptancefor an €conomyIangeof personalcare
products in villages ol North lndia. What sourc€s of
secondarydata :and primary data witl you use lor your
markei research?.Brieflyexplain.

3. Dlscrlssthe udriousstrategiesmarketersuse for product


modification. d€cisions for ihe /ural markets, with the
help of suitable examples.As the product managerof
an €stirblishedconsumer goods company, hc!, woujd
you handle the menaceof spuriousbrands in ihe rural
markets?

4. What are th€ impoltant media \rehiclesspeciallg srnted for


adr€dising a.ld prcmotion in th€ rual .rnarkeis ? Having
identified the rural market po6ntial, gour company is ke€n
' ' io promote its
economy range of cooking gas stoves ln the
rural market. What is ihe media lnix you would suggest to
the company and why ?

5. Wdte short notes on any three of ihe following :


(a) Roie of retailers in rulal dist/ibution
{b) $gnificance of periodic markeis like Haats and
Melas
{c] Behavioural aspects jn rural disirjbution
ld) E chaupal as a disiributlon iniiiaiive
(e) Physical distrlbution decisions lor the rural market

MS,611@
SECTION B

6- Bead the case given below and an$rer th€ quartions given
at the end of the cas€.
Requitmenl oI Insurarce Agents by Orn Xotak Life
lnsurance
,/
S€nsing a big opportunitg in rurat markets, Om KoFk
Lit€. lnswance set up a separate rural business division
headedbr a senior'vicepresident.The marketingsiraiegy
revolved around generaiing businessusing agent routes,
jnirially in Maharashtra

Press advertisements were ret€ased in both national


and regionaL newspapers io select afld appoint agenis.
Ho\xe\ret, despite rcpeated inseriions the response was
very poor. Thereafter, the company approached a
recrultmentagency,a marketingagencyoperatingin rulal
areas and an agenca with its net'r,/orking sources ln ffal

The fi$t t\ro agencies did not have reach in llral


areas. So, Om Kotak linaliz€d a deal with Rulal Relations,
a Pune-based agency with an excell€nt network of rural
volunteers, to recruit agenis for the company.

Bas€d on its previous experience, R\$al R€lations


emphasized three components oJ their recruitment
strategy : {1) dired personal contact (2) spreading

MS,611@ P.T.O.
informatior, through group discussions(3) motjvatjDg
people to become ljfe insurance agents through
counselling.

The awareness-buildlng stmtegy included sending


letiers to opinions ieaders using the huge Rural Relations
databas€, highlighting the beneliis of becomjng agents for
the insurance company and requesting them to spread the
news locally Lo ldentify suiraDleagenls. Six personswere
kained by Rural Relations ic) contact villagers, particularly
rural youth and bri€I them about the insubnce business.
A $,ellproduc€d leaflet ploviding inJormation on the
cgmpany and career prospects, was dso hand€d oui at
the preilminaly interviews held in nearby tdlukas.

'lhe
lirsl level agenLasse$ment was done based on
the information provided in the appllcalion foyms,
followed by quality of participation in the counselling
session and petlormance in a djrect personal intervi€w.
Out of the 375 candidates who appeared, 110 w€re
shonlistedfor rhe secondround of inteMews.

ln the second round, the company head of ihe rural


life insuhnce busjness conducted th€ interviews, along
wjtb his sales maDager and an HR manager from Rural
Relations. Oirt oI the 68 candidates wbo appeared, the
best 30 were seiected. Thereafter, they r{€re put throwh
a 20-dag lnsurance Regulatory Development Authodty

M S6 1 1 @
stipulated training programme, conducted by Maharashira
Insititute of Technology at Pune.

During the training period, Ruml Relations


coordinators interacted with the candidates on a daily
basi6 to soh,e an9 personal or couGe-related problems. To
sustain motivation levels, informal dinnerc were also
arranged on a weekly basis to encourage candidates to
interact with each other and wifi members of the Rural
Relanonsieam.

These ellorts were successlul in building up the


candidates' self-confidencein their abilities to become
competent rural life insurance agents, as was €videni from
the 100 per c€nt pass rate in the final €xamination.

Questiorrs j
(a) Crjtically evaluate the recruitment strategy followed
by Om Kotak for its insurancesalespeople.
(b) What are the altemative sfrategies ihat you would
have followed to atiract larger numbers of
prosp€ctive applicants from the rural segment ? Give
feasons for your answer,

MSS11@ 1,000
MANAGEMENT PBOGRAMME
Term-End Examination
December,2005

MS-612 O : RETAILMANAGEMENT

Tlme: 3 hours MoximumMarks: 700


(Weishtoge70o/o)

ti) Attempt any three questionsfrcm SectionA


(it) Seclion B is cotnpulsory.
(iil All questionscorry equal marks.

SECTION A
1. (a) Discuss the rele',ance and scope of marketing
research in retailing. Exilain brjefly the probable
areas when MR can be applied in retailing
lb) Define "Category" in retail business. E\plain ihe
importance ol category management as a strategic
tool in retail business.

2. lal Briefly discuss ihe key components ot atmospherics


enticing customer pull towads your storc.
(b) Trace th€ evolution of retailing in tie ltdian
marketing envfonment and comment on its
contribution to the economY.

M5-612o P.T.O.
3. Explain the elements of strategic planning process in the
context of retail busin€ss.Take an example of your choice
to illustrate.

4. (a) Merchandisingis at the core of.evety retail business.


Dscuss.
(b) What role does technologyplay in merchandising?
What precautions are to be taken while plannins to
adopt technologg in retail business?

5. Write short notes on any three of the following r


(aI Role and Responsibilitiesof v€ndor
(b) Types of chain stores

{c) Trends in security


(d) Wheel ol retailing
(e) Positioning of retail store

M5,612@
SECTION B

6. Studvthe casegivenbelowanddrswerrhequestion\given
ar lhe Pndof the cd.e.

Dinesh looks outside his first store outlet at the


Metropolitan Mall in Gurgaon gear Delhi. H€ sees the
two MBA studentsapproachinghim for an interview.The
studentsNeeiu and Smita had mentionedthat they were
preparing a case on r€tail location as part of their MBA
studies.Dinesh bad readily_agreed
lor ihe interview. He
has opened his second waich outlet in the Metropolitan
Mall ln Gurgaon just 15 days back; while he is confident
about his choice oI ihe Metropolitan Mall as an ideal
location for his outlet h€ is not sure if this ouilet will do
as well as his first outlet, about 7 to 8 km away at Sadar
Bazaarmarketin Gurgaon.

Neetu i Thanks a lot for granting the inter",iew. Can


you please tell us atrout your outlet at Sadar market in
Gurgaon ?

Dinesh : I startedretailingin watchesin 1994 under


the name Ram & A!""tar Sons in Sadar market. When I
startedthere were 15 shops sellingwatchesin the Sadar
market, no\,,, there are about 20 io 25. But Ram &
A,"?lar Sons is the largestand besLknown In fa.l we are
the largest watch outlet in the whole state of Haryana.

M5612@ P.T.O
Smifo ; You get customers from all over Gurgaon ?

Dinesh : Ybu wolrld be surprised to know that my


customers come from as far as 50 - 60 kms south of
Gurgaon from neighbouring iowns like Rewari, Pataudi,
and Bhiwadi. You may know that Sadar Bazaar in
Gurgaon is the destination market lor people fr<im this
large region. I ha\€ custbmets hom all sections ol tbe
populalion from small children to old citizens belonging to
all income categori€s. .l keep a large variety of branded
watches and am a franchisee of Time Zone. I have
expanded my ouret jn Sadar and now jt is spread over
t\ro floors. I plan to add t'ro more floors to the Sadar
Bazaar outlet,

Neetu ; Why did you plan to open an outlet in


Meiropolltan Mall ?

Dinesh : In Sadar market there is too much


congestion now; car owners find it difficult to find parking
space.They now prefer to shop in the malls where there
is abundant parkins facility.
Besides, th€s€ newly
upcoming malls provide a much more pleasant shopping
experience, I expect suc}Jcustomers wolrld ncNJprefer to
shop in tlre World of Watches - the name I have chosen
tor this n€w ouilet.

M5-612@ 4
Smita i Do you expect your customers to shift thei{
purchases from Ram & Avatar Sons in Sadar market to
the Wotld oJ Watches oullet in the Metropolitan Mall ?

Dinesh : I expect a rery small perc€ntage of my


custoro€rs io shift their purchases from Ram & Avatar
Sons to World of Watches. As I mentioned many of my
customers com€ from far off places to the south of
Gurgaon ftom small towr6. The Sadar market is much
more accessible to these people since the railway station
and bus teminus are nearby. I don't expect more than
15 - 20% o{ m! customeis at Ram & Avatar Sons to
shift their purchasesto these malls and shop at World of

Neelu i Then why have you set up this new outlet in


ihe Metropolitan Mall ?

Dinesh j I expefl most ol rhe cuslomersof World of


Watches io come lrom coloniesin south and west Delhi.
I expeci that 80% of my customerswill be from Delhi. I
do not g€t customers from Delhi at my Sadar markei
outlet. So you can see that I am targeting a totally
different segmento, customers.

Smito i Why do so many people come io your shop


from colonies in Delhi some of which are more than

Nrs-612@ P.r.o.
20 km away. After all there are many watch retail outiets
in Delhi.

Din€sh i They do not come to my shop alone. They


come to these malls from Delhi for recr€ation and
shopping. You can see that these malls provide a very
pleasant air-conditioned shopping environment. The null
also has abundani parking facilities and is convenient for
people owning cars. Becaus€ ol that they attract people
from Delhi. B€sidesthat many people from Delhi work in
Gtfgaon; they also pref€r to do ti€ir shopping in th€se
malls.

Neet'u ; What else wotrld be diff€reni about your


customers in Sadar market and here ?

Dinesh : My customers in Sadar market aye from a


cross-section of the socieiy, most of them use public
transportation. I a\pect my customers at the Metropoltan
Mall to be from upper middle class and middle-class
income groups. The customershere would generallybe
car owners. Also I expect a hrye nutnbet ol nry
customers here to be younger in age.

Smita j Why do you ej<pectyounger crov.,d to cotn€


to this shop ?

M5-612@
Dinesh j This mall attractsyoungstersbecauseof the
presence of a large number ol eating joint, pubs, and
cineplexes.There are 4 - 5 pubs near my shop.

Neeru : Did you do any sunreybefore deciding on


this location ?

Dinesh i I am in businessfor long. I understandthe


needs of my customersand, hence, I could identify the
customerswho would prefer to shop here. I met a lot of
shop own€rs in. the mall and also had discussionswith my
customers-I estimatedthe daily walk-ins to be around
6000 to 8000 going up fudher on week-ends.As ihis
Mall becomespopular, I expect to do good business.

Neet'u i You have chosen the first floor location


rather than locating on the ground floor of the mall
where the customer traffic is higher. Why 7

Dinesh j The rentalson the ground floor are armosr


double; I don't expect to do enough business to recover
the high rentals on the ground floor. Besides, as you
might have seen, the shops on the lirst llooi are smaller
and more in number. Sinca there are larger number of
shops on the first floor, a broader range of merchandise
would be available. That will attract a lot of customers to
the first floor. Besides, ihere are many pubs on the first
Iloor which would atrralt youngsters.

M5-612@ P.T.O
Neetu j How many customers do you expect at
World of Watches esery da9 ?

Din€sh I I expect about 400 to 600 \^ralkins e\Jery


dag. Il the conversionratio is about 8-10% I will be
happy.

Smita i Are these figures based on market


reseafch?

Dinesh r I did not do any formal marke4 research;


ihese tiguies are based on my discussionswith other shop
owners and my long experienc€ in the fieid.

Smita j Is there any difference in your business


strategg between here and your Sadar market outlet ?

Dinesh : I keep a wide rang€ ot branded watch€s and


provide repair facilig at both these outlets. However, at
Sadar I keep more of lower pric€d bran& like Sonata.
H€re, I plan to keep more of high-priced branded
watches. I sell Titan, Timex, Casio, and Citizen brands of
watchesin Sadar. Here, I am also planning to introduce
premium brands like Pierre Cardin and EspiriL. The
average value of watch I sell in Sadar is priced around
Rs. 1,000. Here I exp€ct the averageprice of a watch
sold to be around Rs. 2,000 to Rs. 2,500.

MS$12@
Neetuand Smitaj Thankyou v€rymuchsir; thishas
beena greatleamingopportunityfor both of us.

As Neetu and Smita driv€ back to their collegehostel


theg discussabout their interview.

Smitd r Dr|esh is certainlyan intelligentbusin€ssman


but I wonderif he has takenthe right decisionby opening
his secondoutlet in the MetropoliianMall. I also wonder
if Dinesh has made a right decisionby locating on the
first floor.

Neetu : T11e Mehopolitan Mall attmcts a lot of


footfallsbut I wonder lf the peopl€visiting this mall would
purchas€watchesfrom here. If watchesare purchasedas
_products,
impulse I suppose the large walk in crowd
would be attlacted- I would like to discusswith orr
professor if watches are convenience, impuls€, or
shopping products.

Questiors :
(a) Has Dinesh done adequatemark€t researchbefore
decidingon the locationof his secondouilet? What
elsecorlld he have done ?
(b) Should ihe watch outlet have been located on the
ground aloor instead ol first floor - whai
. datalnformation is needed belore taking this
decision?

Ms,612G) P.T.O.
(c) What are/the advantages and disadvantages oI a
location in a mall like the Metropolitan MaI
compared io a location in a central business district
like Sadar Bazaar ?

M5-6120 10 1,000
MANAGEMENT PROGRAMME
Term-End Examinatlon
December,2OO5

MS-6S@: MANAGEMENTOF MARKEflNG


COMMUNICATIONAND ADVERTISING
Time : 3 houts Matimum Marks : i00
(Weightoge 7@/ol

Note :
(i) Attempt any thtee qtlesfionslrcm SectionA.
(ii) S€ction B i5 compulsory,

{iii, All questions carry equol fiorks.

SECTION A

l. Why wor.rlda manufacturerof smallhand tools see trade


promotionsas an effectirerlay to increasehis sales? What
type oI salespromotionschemes
\r,ouldyou recommendfor
suchan otganizal/ofi?

Ms68(D P.T.O.
2. ln 2005 "Exoiica" a corporate gift company mailed
catalogs devoted to providing corpo&tions wiih a s€lection
ol Diwali gifts to be used as employee inc€ntives. The
catalog offered bulk quanuti€s of merchandise embroidered
,rrith corporate togos ai \,arying discounts, depending on
the size of the order. Sucha mailingwould only be sent to
those companies designated as qualified prospects. How
does the prospecting activity o{ the cataiog in this case
dif{er ftom that used lor selling to consumer market ?

3. (a) Discr.lss th€ advantages and disadvantages oi


Television as an advertising medium.

lb) ln what ways could a direct marketer use teievision


to promote direct sales ? Brielly describe each
method.

4. What factorsurould!,ou considerfor mediaselection,while


yout answer
d€v€lopiagan advertisingcanrpaign? lllu.strate
eith suitable€xamples.

l\,4s
68@
SECTION B

5. Carefullyread the gr€n.ase on IndidnAir Force {lAR dnd


answer the questions given ai the €nd of the cas€ i

The Indian Air Force {lAD doesn'i pay, it's not wofth
risking !,our lif€ liying MiGs and its too difficuit getting in
nnway - this is what youngsters thirll ot a career In the
lAF. An IAF suwey conducted by ORG Matg asked
3,000 students b€h^/een 18 and 20 u,hat they thought of
a job in what was once considered the most glamorous
d€fence wing. Theg were asked to till up a questionnaire
and list iheii prefgrenc€s in order of priority. Education
topped th€ pre{erence list, Iollou,ed by an opporiunity to'
do research work. A luxurious lil€siyle and fat pay
package came in The surtey revealed that
thid.
youngslers were scared ol {lying MiGs - the IAF hasn t
'flging
been able to shake olf the coffiD' tag. Students
thtnk the IAF is a poor pagmader and that promotions
happen at a snail's pace. For ihe lAF, whicb i3 shott of
45D ol|icets, the $rrvey's lindings are damning. The Air
Force has now declded to launch B massite awareness
'misconceptions among the
c€mpaign io remove these
youth. "Our publicity ceils will be activated so thai
youngsters get to jnferact with officers. We are exploring
'
ihe idea oI a car rall, to popularlze the IAF, Eaid Wjng
Commander R.M. Danak, Joint Director Personnel
Of{icers.

MS-68@ P.T.O.
la) What marketingissue(s)has IAF to handle?
{b} Who are the Target ALdiencesand what is their
minds€t ?
Do you ihink that the car rally will help in achieving
the communication objeciives ?
lf you have to handle this iask, what communicaiton
sbategy woLrld gou recommend to IAF ?

MS€8o 1,000
l-Mrqdl
MANAGEMENT PROGRAMME
Term-End Examination
December, 2OO5

MS-91@: Otdr STRATEGIC


MANAGEMENT
Revised: ADVANCEDSTRATEGIC
MANAGEMENT

Time : 3 hours MaxiinumMorks: 100


(Weighnge 70Vo)

Note : There are two Sections A ond B. Section A hos tvro


sefs. Sef .l ls meont Jor the students lDho houe
reglsteredJor MS-91 : StrategicManogementprior
to July 2005 i.e. upto January 2005. Set Il is meont
for the students who haue registeredfor MS-91 :
Adnnced Strotegic Manogementhom July 2005
onwards. Attempt ony three questions frcm
Section A. All questions carrg 20 morks each.
Secfion B is conpursory Jor oll, ond corries
40 morks.

Ms-91o P.T.O.
SECTION A
S€t t (OId)
Strategic Managem€nt

t . Organizationale nronment plays a major role in the


decision-makingof the firm. Discuss in bdef, ihe
organizationalenvironmentwith respectto the stakeholdeF
and their relationshipwith the firm.

2. Explainthe conceptof selflessness


with the rmi\€rsalinner
structureof leaders.

3. Explaintechnologyforecastingprccessand discussvarious
forecastingtechniques.

4. Explalnthe conceptof SocialProcessAudit. Briefly d:scuss


bnow€s Modelas an apprcachto socialr@ort.

Write shod noteson any tfrree of the following ,

ta) Environmentalanalysrs
(b) t€vels of strateg!'

(c) Evaluationof shategy


(d) Leadershipstyles

(e) Holding companies

Ms-s1o
SECTION A
Sa tr gwised)

Advanced Strategic Management

t . Discussthe natureandscopeof corporatemanagementand


its role in non-business
organisation,givingexamples-

2. 'The different market structureshave different viewpoints


with respect to compelition.' Explatnihe statementwith
fespectto the marketstructuresand slrstainable
comp€tifive
ad\Entage.

3 . What are the diller€nt stepsinvolvedin developingR & D


sitategy ? Dsd$s with suitableexamples.

4, Writ€ briel notegon i

(a) KlmaffnangalarnBirla CornmitteeRepori

{b) },Lara!,ana
Murthy Corynjttee
(c) NareshChandraCommtttee

t. Discuss tbe isslres and challenges in Knowledg€


Managementin the presentday competiti!€endrorunent.

Ms-e1@ P.T.O.
SECTION B

6, Readthe lollowing casecarelullyand answerthe questions


that follow :
ASIAN PATNTS0NDIA) UltrrED

The siege is over, and the time has come tor the
leader to sallg forth into greener pastur€9.Even as tbe
paints industry is em€rglngtrom the sh;dow ol recesgion,
ile Rs. 560 crore Asian Painis (lndia)Limited (APIL), is
mixing ne!/l/shadesto emergewith winning colours.

SaysmanagingdirectorAtul Chokey : "With proper


plannjng and a comptehensiveapproach to ilsues, \r€
intend to keep pace with the grovrth ol the industry".

APIL is actually targeting a groMh tate that is


highef than the 9 to 10 per cent that the industry has
been averagingrecendy.ln the year to March 1994,
the company notched up a grosssales iumoverof
Rs. 559 96 crore (net sales: Rs. 401 96 crore), a gto$dh
o{ 10.8 per cenl over th€ breviousyear. Net proljt also
registereda healtly growthof 31 5 perc€ntto Rs. 25 61
crore. The resultshave tidied up the company'sbalance
sheet,which had begun to look a bit ragged.

APIL's approach js multipronged: expansionof its


product ra$ge and introduction of lalue added, niche
productsin the industrialpaintsareai Iine extensions
ol

Ms-e1@
existing products to target lower income market segments
both in rulal and urban areas; expansionsoI production
capacityand continuousmodernisationto keep pace with
the growing demandiand diversificationinto the unrelated
but synergisticar€a ol ceramics.

AII these strategies are part of what ihe compang s


top managementterms "harnessingour full potential" ,or
the challengesthat lie ahead. They are also aimed at
r€taining leadership in a recession-fr€e industry over the

APIL is the leader in the entir€ industry, comprising


both organised as wetl as unorganised players, with a
market share of aboul 19 per cenL. The company is
confident of the lact that its share of indusiry sales is
twice as much as that of its near€st competitor, Goodlass
Nerolac.APIL also dwarfs the oihers in size, its net sales
nearb twice that of CoodlassNerolac,well over twice that
of third-placedBerger Paints, and nearlg lour times that
of fourth-placedJenson and Nicholson(seeExhibit-I).

Ii is only wary of the expandins unorganisedsector


which seems to be eaiing up the share of firms in the
organis€d secior. Nevertheless,given the multiplicity of
shades it is capable o1, APIL reckons it can look forward
fo a compound grov,th in its market share.

Ms-s1@ P.T.O.
Exhibit I
How Th€!,rCompare
(Figuresin Rs. croreJor 1993 - 94)

Net
l\er profit,/sales
Company Net sale
pro,rr pkl

Asian Paints 401 96 25 62 636


GoodlassNerolac 20588 805 3.91
Berger PajDls 17495 3.24 1.85
Jenson& Nicholson 110.33 r.97 1.72
Ganrare Paints' 106 257 233
ShalimarPaints" 70259 160
Bombay Paints" 37 81 0.03 0.08
'18 monthsto September
1993

"12 monthsto March1993

But though the good iimes are back, the company is


not content to sit back and relax. The last three years,
during which the paints industry went through a ttough,
saw APIL taking a beating {though it remained the market
leader all through), with its paints division showing a
negative grov./ih ol 3 5 per cent ln terms oI volume.

Ms-e1(t
Wiih the rupee having been progressivelydevalued
during the years 1989-92, and with high rates of
inllation also rarnpantover this period, excisedufies and
othe/ l€viestoo a\ated upward pressureon paint prices,
and thjs served to depress demand. An additional
complicatjon, reinforcing this trend. was created by th€
ditteret\ce in the selling prices of paints made by the
organisedand unorganlsedsectoys,

The first signs of recovery came with ihe Unjon


Budget o! 7993 which cut eKciseand custom duties,
Exciseduties were reducedto 30 per cent and customs
dufies were cut from 85 to 65 per cent- This provideda
tespite to $e industw bg facilitating a rollihg back ol
prices, and it beganto grow at about 2 per cent a Vear.
In spite of intermittent soclal disturbanc€sin 1993, ihe
industry graduallgrespohdedand so did fhe derDandfor
its prociucts. Simultaneously,the automobile iridustry,
which is a major us€r industry for paints, also began to
emergefrom the two-liearrecession,

A gradualrevivalof the jbdustrybrought atonga new


thrcat tor the seveDrDajor players lrom the organised
segment. Uneven prices during the recessionyears had
the unorganisedcoDpetitors grabbing at a sigbificant
chunk of the marhet-

MS.e1@ P.T.0.
Budget concessions brought telief to the orsanised
s€ctor, bul its constituents also found themselveg havillg
to compete with an unorganised sector that had groLento
become a significant thr€ai, €een as the Frospect of
competiiion lrom imports began to wofiy ihe otganised

APIL'S largest new venture will be a div€rsification


into ceramlcs, though lhe project is still at the plannlng
stage. The decbion to enter a new field is fr.relledby the
managem€nt's perception that the ceramics industry has
tremelrdous potentiat ior groqnh.

Even though the company has no expedence in the


production and technologg aspecis oI ceramic tiles
manufacture, it has opled for ceramics becaus€ the
mafketing will involve utilisation ot its existing distribution
nef'r0ork for paints. The rationale is that since painis and
ceramics are both buildins haterials, APIL'S exisiing
custom4r base (which can serve as a ready-Dade mark€tl
will be largeted for its ceramics products.

"With our extensive distribLtiion nen^rork and gtocking


points, we can reach elen the remote matkeis. So
marketing ceramicn is not likely to be a ptoblem," says
Choksey. The plan is to penetrate ihe market as qujcklt
as possible, and grab a substantial cbunk of indus'try sal€s-
The compang wilt initialb siart wiih ceramic tiles, but

MS-s1()
ihese is no plan to resirict itself to any specific market
segment,

The project involves a Rs. 70 crorc initial jnveslment


in th€ first phase,which involvesinstallationof a capacity
ot 23,000 tonnes per year. Thjs wjll b€ tollowed ir a
couple oI y€ars by the second phase, which will see an
jnoease \D the capaciiy to 50,000 tonnes-

The new project is scheduled lor completion by the


end of 1996, and it will. in aU probability,be located in
Gujarat. This is because any location in thai state will
have the advantage of proximity to the raw material
supplyjng areas in Guja6t and Rajasthan. APIL is
currently negotiating wiih foreign collaborators for the
technology, which will have to be imported. The
technology Lvrll also hav? to be adapted Lo Indian

While putting a few eggs in a new basket to ensure


that fluctuating fortunes in the paint industry do not have
the elfect of hurting the company's bottomline yet again,
APIL is not ignoring its bread-and-butter busln€ss - that
of paints. Over the past yeai, a variety of new brands
bave been added to its produci range. The company has
made an attempt to extend its marketing and distribution
beyond the country's major towns, to which its aciivities
were hitherto con{ined.

rvls-9| @ P.T.O.
'Utsav', an economicallypriced brand, was launch€d
last gear and is targetedat small householdswith limited
budgets.This prcject concentmtedmainly on consumers
in Tamil Nadu, Maharashfa and Gujaral, thus widening
the accessibilityof lts productsto all consumerlevels.

General Manager Mr. P.M. Mr-rrthysays that "the


degreeol penetrationconcentrateson how economicalit
is to do buslness."He saysthat though lhis new product
has performedfar,ourably,lt has not contributedmuch to
the prolits of the year. "Of courge,it promisesto be a
very good and attractives€gm€ntfor future business,"he
adds, when asked about its futur€ groMh and profit
potential.

Other new productsalso includepowder paints to be


us€d for both auto and non-auto appliances.Th€re are
othel products fike wood linishings (Touch-wood)that
iakes care of refinishingson turnilure.

To sirengthen its industrial prcduct base, APIL has


collaboratedwith PPG lndustries,an American{i!m, and
thus enjoys the use of cathodeelecbo deposiiion primer
(CED).The companvhas coocLuded a tie-up with Nippon
Palnts for original equipment paint products and with
Sigma Coatings of Holland for conosion coatings-The
technologythat has been broughthome as a resultoi

MS-e1@ 10
these vebtures is modi{ied at the company's plant at
Bhandup,so as io make it suitabletor the Indian climate.

With a tretter product tange on olfer now, APIL js


,ust wai$ng for a grcater awarenessof industrlal paint
appiications to develop iD the Indian markei; tlre
presumptionis thai th? d€mandfor this patticularproduct
js still htent. For its decorativepairls, the company has
gone in for ditlerentialprlcing to €ncourageall segDenis
ot the market.

fhe companyrs intenton a conlinuous


modemisation
and upgradationof lts technologyand lts assets,so as to
keep in tune wi{h the changlng requirementsof the
mar*€tplace.ln addition, ii is also woiking on plans to
increaseproductioncapacityowr the he)d tew years.

Besidesihe activity on ths domesticftont, APIL is


lncreasingits oveEeaspresenceas well. One o{ the ,ert
lndian companies with overseas subsidiaries in the
South-Pacific region, APIL is now settFg up a new
subsidiaryin Ausfualia.lts existing ventures abload too
have reported healthy resu)is : Asian Patnts (Sor.rth
Pacific)has registereda 12 per cent grov/th, Aslan Paints
{Tonga) grew at a rat€ of tive per cent, Asian Paints
(Solomon Islands)at over 10 per ceni and Asian Paitts
{Nepal)at over 18 per c€ht.

Ms-e1@ 11 P.T.O.
With a new s\rbsidiaivat Vanuatu(New Hebddes)and
a joint venhrre unit in Townsville(Auskalial, APIL has
established at least a fooihold in the intematonal
markeis-

When asked about the ihreats facing the company,


Choksey chucldeg and says h€ pr€{ers to calt them
challenges. "We need to meel the dema^ds of this
g{oqing organisation- ol our workforce,our i€chnology
and ouf ass€ts.A majorpoint to be tackledis to be able
to meet the qrowing d4mand for our product and to
create a greatef awarenessfor orrf newer productg," he
says_

Over the first lew months of the qilreni financial


year! salesvolume has been growing at a rai€ of 14 per
cent. well abovelhe industryaverage.With the recession
ti{mly behind jt and goiernment leviesno longer inflating
its pices, lhe paint industryseemsto be on an uptr€nd.
Brrt lhe APIL managementhas its wolk cui out for it i it
will not mer€19have to gear up to m€et the burgeoDing
demand,but will also hav€ to worl hard at retainingand
then increasingits mark€t shar€.

Quesliorg :
ia) Whatcorporategoalhasthe comparyadoptedfor the
next few years and wiih what strategiesdoes the
companypropos€to realisethe abo!€ goal ?

Ms-e1@ 12
Whatthreatsis th€ companyfacjngor/and rnight{ace
in futlre ? What has it done anvor what could it
further do to safeguarditself lrom thteat(s)?
(c, Evaluat€the nelv strategiesof Asian Painh (lndia)
Limited.parlicdarlyirs proposed
forayinlo ceramics.

mat actior dans has the company propos€d to


strengthenits productbase?

ClassiJyall the strategicplans or propos€dskat€gic


actionsof the corhpanytor achietrnggro[th against
suitableheadings,e.g., Oiversilication,Joint Vettlires,

MS-slo I,O00
MANAGEMENT PROGRAMME
Term-End Examination
December. 2OO5

MS-92€) : MANAGEMENT OF PUBLTC


ENTERPRISES
Time : 3 hours Maximum Marks : 100

There orc two SectionsA and B.

Section A hos ttto sets. get I is meont Jor the


students who hole registeredfor MS92 pnor to
July, 2004 i.e. upto Januory, 2004.

Set 1l is meant Jor the students who houe


resisteredJor MS 92 lrom July, 2004 onwards.

Attempt dny three questionsIrch SectionA.

S".rion B is computsory Jat au

All questions cafty equal marks.

Ms-s2@ P.T.O
SECTION A
Set I

{Pre-Revised)

l. Whai are the foundations on which Government- PE


relationshipsare based ? Discussthe differeni ways in
whrch Covemment inter".l. wlh Pts

2. "The governing boards of PFJ in India occupy a very


significant position and enjoy far wider responsibilities than
ihe boardsof commercialcompanies."Discus,s.

3. Explain the concept ol employee participationand how


dop.'i nork rhroughownershipin lnd'd.

4 . Discuss the causes which can be attributed to


organizationald€clineof the PEs. Illustratewith examples.

Explain ihe concept of a holding company and discussthe


evolutionof holdingcompani€sin PEs.

MSe2@
Set II
(R€vi3ed)

l. Djscuss ihe impact of economic refoims on PEs since


1991.

2. 'Legislaiive
contrcl beco.nesone ol the most important
and elleciive methods of enforcing accountabiiig.
especiallgin the Indian confext.' Elaborateand critically
analyzethe statement.

3. Briefly discuss the pedormance evaluation of the Siat€


Electricv Boards.

4. Discuss ih€ variolls methods of disinveslment and ihe


qrrdregrc
rs5\resInvolvedi"l di,invernenr proce"s.

5. Wrjte short notes on i

{a) Restructuring

{b) Leases
(c) Joint-ventures
(d) Corporate Covemance in PEs

MS-92 o P-T.O
SECTION B

Read ttl€ following passages


and answerthe queshonsthat
follow i

BharaX Bhari Udyog Mgam Ltd,

li is holding company comprisjng seven subsidiaries


in the eastem reglon includlng wel) known names
like Burn StandardCo. Ltd., Jessop& Co. Ltd.. and
Braithwate& Co. Lid. Its 1990 - 91 salesturnov€r
was Rs. 410 cror€s wiih a loss {or the year of
Rs. 10 90 nores

Th€ company'sobjectjveas given in its 1991 - 92


MOU inter oliq 'is io achi€ve a high degree of
customer satisfaction through timely supply of quali\'
ptodrcts and s€rvjces". This was to be m€asuredwith
refercnce ta delivery/erc.iion commjtnlents of varjous
products and services on a five-point scale with a
weight of 10%.

Do you think that the company's objectire is


appropriatein the pre$entcont€xt ? Commeni.

(b) Andreu Yulc & Co. Ltd.

This old Brliish ManagingAgencyhouseb€camea


govemm€nt company in 1979. lt is a multi-unit,
muliiproduci conglomerate with five subsidlariesand
a gross turnov€ro{ Rs. 325 qores in 1990 - 91.

Ms-e2@
The company's MOU attached 57o weight to
customer satisfactionm€asured with telerence lo
liquidated damages and fr€e rectification and
replacementcolt as a perceniageof turnover. The
companys aim.was to reduce it from .40%oin
1990-91 to .36Voin 1991-92. The company
wanted its marketing efforis to be judg€d with
relerenceto (i) total orders booked during the y€ar,
(ii) selling and marketing cost, and (iii) number ol
productslaunched,with an overallweight of 8%.

Comment on the marketingefforis of the company


keepingIn mrndrhe custom"rsperspective.

MS-s2
@ 1,000
MANAGEMENTPROGRAITIITE
Term-End Examination
December.2OO5

MS.93@: MANAGEMENTOF NEW AND


SMALL ENTERPRISES

Time : 3 hours Moximufi Morks:100


{Weishtase700/0)

Nore, This pqper consisis oJ two Sections A ond B.


Alemp! anv thtce questiohs J,oh Sedbn A.
SectionB is compulsory- AII q\.testions
coffy equdl

SECNON A

1. What is the scope and types of financirg suppotl provided


Lmder ih€ current policy for the SSI sector ? Explain in
d€taii,

2, ld What are the criteria that entrepreneursuse in


' e',bluating alternative produclion processes and
equipm?ntsple(riont

MS-s3@ P.T.O.
(b) A group o{ rutal agriculturistr is planning to set up a
{ruit preservation unii io make jams and jelli€s out of
the locally grown produce. What is the process 9ou
would advie them to follow to settle on ihei! tocation
decision?

What are the components of project cost that must be


refl€cted in a tusiness plan ? Taking an er"ample ol your
chojce,discussthe cogt components.

4. (a) E\plain the impoftant iesues in product desisn


selection {or a small entrepreneur

(b) Briefly describethe process of lroiotype development


and iis implicationsior productmanagement.

5. (a) What are ihe alternative choices regaiding


management control that can be consider€d by a
familv enlerprl5e?

{b) How shodd issues ol strccession platning and


continuity be handled in family enteryris€ ? L\plain by
taking exarnples.

Ms-e3G)
SECTION B
,
6. Read the case given below and answer the questions given
at the end of th€ case :

Tasty Foods Limited

Tasty Foods Ltd. is a small enterprise €ngaged in the


manufacture of dilf€rent food products. Its R and D
activities have recently col4e out with a new soyabean
snack. The main strength of this product is the highly
nutritious value of protein-rich soyabeans.

While the R and D DePartm€nt is happy to have


developed a highly nutritious snack especially useful fot
children, the company managementwants to be cedain
that the new snack will hav€ a good market in the
country. In order to ensure that the new product is
successfully launched in the market, the Managing
Direcior (MD) is thinking of hirjng the professional
seruices of a Marketing Research Agency (MRA) ln
consullalionwith lhe MRA, *re MD decid€d to convene
a meeting of Marketing Manager (MM), Production
Manager iPM) and the MRA, to be represented bv its
chief executive.Th€ me€ting was held in MD's chamber
and il tumed oui Lo be a prolongedsession

Ms-e3o P.T.O.
A few excerpts from th€ discussionsheld in thai
meeting afe given below :

MD , I feel certain that our new soyabean snack is


going to be very 5uccessfulin the market. The
project is unique in the sense that no soyabean
based snack is pr€sently available in the mark€t.
In view of this, test marketing of this product
seems to me rather Lrnnecessary,We can do
without it. Moreover, it will save a good deal oI
time as the new product can be introducedin
the market almost immediately.

PM : I fully support the vi€ws expressed by our MD.


Let me emphasisethat our R and D Department
has taken considerable pains in developing this
new product. It has come through a long
. succession of concept and product tests. Test
marketing wodd obviously delay the process of
marketingLheproduct.

MM : Since our new product is going to be our major


achievement, it is desirable to have detailed
information about it. It will b€ to our ad',antage
if we know our consumer target and their major
characlenstics.

MRA , I know some cases wherc a new product was


'strong' product
regarded as a by the R and D
and the Production Department. But when it
was commerciallylaunch€d,it tutned out to be

Ms-e3@
a failure. The concerned companies learnt a
bitt€r lesson. I am of the opinion that the
product in questionmust be test markeied to
avoid any such disappointment at a later
srag€.

MD: Suppose we decide io go in {or test matketing,


what are we going to get from such an
exercise ? w€ musi be clear as to what test
marketingis expectedto do for us.

MRA, The MD has raised a basi. que-stion. The


company must b€ clear about the objectives o{
test markeilng. Are you looking at test
marketing to decide whether ihe new product
should be launched naiionally or not ?
Alternaiively, are you looking at it as a rehearsal
before an extensive laurch ? This means lhat
9ou are intetested in knowing wbat is likely to
go wrong and how this can be
prevenledAmproved.

MM. I think both these objectivesmentioned by MRA


are yele@nt in our case, ln fact, the restlts of
test marketing r,.'ilt indicate whether we sbould
launch ihe product nationally or drcp it
altog€ther. I think it is advisable to decide the
nature of information to be collected so that iest
marketingcan be reallyhelptul to us.

MS,e3o P.T.O.
MD r l-et us then have test marketing. But before it is
undertaken, we must specify our data
requirement. I suggestthat MRA and MM should
meet and discussthis fully.

Qdes,ions i

(a) What advice would you give to the management in


terms of the information to be elicit€d through iest
rtarketing ?

(b) What other mechanism ot makrng a markel


assessmentcan be used L'y th€ enterpdse in cas€ the
test marketing is not done ? Suggest.

Ms.e3@ 1,000
MANAGEMEM PROGBAMME
Term-End Examination
December. 20O5

MS-94 @ : TECHNOLOGY
MANAGEMENT
Time : 3 hours Mdximum Morks: 100
7@/o)
lUVetghtoge
Note t Attempt atl questiotls.All questionscarry equal

1. (a) "R & D provides a window to de,le:lopment at:f


enables tbe Dation or a fi'tu to ev)vate exte:r,al
developments and react morc quicldy to th€m."
l ustrate by indicating alternative objectives in th€
area oI en€rgy and matedals technology.
(b) Wlrat are some of the commitments and assurances
that a technology policy is sr.lpposedto commit to
the government ?

OR
{a) "A prerequisite for effective utilizauon of R & D
tesources is the development of technalogy
infrastnrciure within the country." Giv€ some
.
examples of technology infrasiructure ihat are
necessary for developing technology capabiltties.

Ms-s4@ P.T.O.
(b) Discussthe Freeman'scaiegorisalionof technological
changewjih examples.

2, Explain the Delphi t€chnique.State its advaniagesand


disadvantages.
OR
what are the objectivesol technologysearchstrat€gy?
Briefly explain varioug categoriesof palments lor the
technology.

3. Define technology absorption. What conslralnts m


' tech$olosy absorption
have teen reported as far as lndian
experience is concern€d ?

OR
Describe the various siages in Technology Assessmentand
brie{ly explain each stage.

4. "An enierprise should b€ viewed as a Human Perfolmance


System." What wor-rldbe the major components of such
hlman performance sgstem iq the cont€xt of t€.hnologV
management ? Expiain.

OR

What is Technology lnformation ? Discuss the role of


Technology In{ormatio$ sygtem. Explain the contents o{
Technology Inlorrnation with suitable examples.

MS-s4
@
5- Explainany four of the following,
(a) Impodant inputs Jor enterprise ievel technology
srraregy
(bl General guidelineslo be lollowed lor iransler and
absorptionol importedtechnologEs
(c) lndian indusiryand industrialresearch
(d) S & T manpowerin India
{e) VentureCapital I Role and its groqft,hin India
(0 Technologyand Socio-economicplanning

MS-e4
@ 1,000
MANAGEMENT PROGBAMME
Term-End Examinati,on
December, 2005

MS-95O : RESEARCH
METHODOLOGY
FOR MANAGEMENT DECISIONS

Time: 3 hours MoximumMorks: 700


Meightose 70o/o)

tj) This poper contoins iroo sections, Section A and


B. Section A cornprising 60 morks contoins Jioe
questions. Attempt on| folrr questionslrom this
Secrion- Secfion B is cohpursory and carries
40 marks.

(il Stotisticol tobles will be pro\ided, on demond.

(rti) Use oJ personol non-programmoblecalculators Is

MS-s5@ P.T.O.
SECTTON A

I, A local supermarket has €xperienced a decline in unit sal€s


and littl€ change in rupee value sales. Proliis have almosi
vanished. The chiet a€cutive in s€arching for ways to
revitalize the operation, was advised to increase the
number of hours tie market is open for business. He
comes to you for advice 'n structuring a fesearch problem
that will provid€ relevant informaiion for decision making,
Define fle researchproblem taking care lo :

(a) state the relevant question.

(b) enumerate the alternative answers.

(c) clearly define ttr€ units ol analysis and charactedstics


of interest,

2. "A sample may be large yet wofhless because it is noi


randomi or it may be random but unreliable because it i5
smdl." Comment upon the above statement and explain
th€ importance ol sampling in daily life. t5

3- What do you understand by factor analysis ? Mention th€


purpose and uses of lactor analysis.

4 . What are the seven elements of communication, which are


relevant lor making a presentation ? Dscuss.

MS-s5
@
5. Write short notes on any ahlee o{ the following : 15

(a) Objectivecharacteristicsandlnlenedcharaciedstica

(b) &liijng of primary data

(c) Operating and Strategic decisions

(d) Ordinal Scale

{€) Audio-visual aids in presentation of reports

Ms-e5
0 P.T.O.
SECTIONB

6. Describe the semantic dillercntial scale. Use the semantic


differential t€chnique to dev€lop the profile of ihree
television manufacturing companies in India.

7. A random sample of 30 stud€nts obtained the lollowing


marks in a class test : 20
Tesl the hypothesis that their median score is more
than50.
58 55 25 32 26 85 44 80
33 ?2 10 42 15 46 64 39
38 30 36 65 72 46 54 36
89 94 25 74 66 29

MS-s5@ 1,000
MANAGEMENT PROGRAMME
Term-End Examination
December.2O05
II
MS-96@ : TOTALOUAI-ITYMANAGEMENT
I
I MoximumMorks: 7OO
I Time: 3 hours
(Weishtose
70/ol
I
t '
I
I (i) There ore hoo Secfions : Section A ond Section B
I '

(ii) Attempt ony three questions from Section A,


i uhich caffy 20 marks each.
(iii) Section B is compulsory ond cafties 40 marks

SECTION A

1. (a) "lnnumerablebusin€ssorganizations,wheihet in the


East or the West, have realized ihe role and benefits
of quality." Discuss some of the important factors
that have caused this realization
(b) According to Crosby, what are the basic tenets oI his
statement that quality is an unending cycle ?

2. . (a) Discussthe role and functionsoI top managemeniin


quality improvement.

MS,e6o P.T,O.
{b) How are quality and productivity r€lated io each
other ? Does technologyhave any role in these ?

3. (a) Discuss the role of TQM coordinatoG. Who are


qualitg champions and wlat is theil imporiance ?
(b) Highlighting the special characteristics that
distinguish leadership in a TQM context, discussthe
six actionsof leaders.

4. (a) What factors motivate a businessorganization to


adopt EnvironmentalManagementSystem (EMS) ?
Discuss.

{b) What are ihe profession;1,/technical abilities that a


quality auditor is suplrosed to possess?

5 . Explainany tour of the following,


(a) Criteria for DemingPrizelAward
(b) ElementsoI SafetyProgramme
(c) StatisticalQualityControl
{d) Characteristicsol QualityCircles
{€) Applicationsof RobustDesign
{0 ChamcteristicsolReengine€ring

M S9 6 @
SECTION B

6. Read the following case cayefullyand answer th€ qu€stions


given at the €nd.

Oberol Group

One hundred per cent hospita'ity.That's the simpie


quality target set by the Rs. 520-qorc Oberoi Cralp [or
each of its 19 hotels, 11 of whjch ar€ managedby the
flagsbip Bs- 220-crore East India Hotels. For, as a sirdy
conductedby the group's top managementand presented
periodically to executives, housemaids, cashiers, stewards,
doormen, chefs, and laundry-boysin ihe hotels revenls,
the cost of even 99.99 per cent quality is ioo high. For
99 per cent qiality means t

. Tbat every three days, one guest checks olrt without


paying his bills.

r That every day, 20 orders ol {ried eggs have human


hair in lhem-
r That every day, 15 tables in the restaurants have
soiledlinen.
r Tbat every day, 250 ptates are broken.
r That every day, the drinks of 40 guestsget mrxed
up.
r That every day, 20 guests do noi get back the right
lauodry.

Mse6@ P.T.O.
Shocked by th€se numbers, the hotel chain is
iuriously adopting the fomal quality initiative )aunched in
1993 by group managing director Priihvi Raj Singh
"Biki" Oberoi, 65, son of the chain's founder, Mohan
Singh Oberoi. Actually, Oberoi Jr. is m€rely
institutionalisingthe quali9 standads set by his lather,
who personally auihored the bulky manuals that detail
every step the hotel staff follow, ranging from how guests
shodd be received to how the rose should be placed on
a btealdast 1ray.

In th€ past, this cr€ated exact:ng guidelinesfor ihe


hot?l chain - such as the one which ltipulates that every
pillow in every hotel must be sluffed with exactly 1.8 kg
of impofed down. Or that all upholstery and fumishings
must be Scotchgarded. Or thai th€ soap to be placed in
balhrooms must be selecied alier dgorous examinatiotl,
involving the testing ol 15 {ormulations in laboratories
and the conducl o( {oqgs gyo\jp inler,'iews among

So, quality has never been an alien concept in the


Oberoi Group. What Oberol is now doing, though, is to
standardise quality, with th€ aid ol professional consulting
expertise, in ihe Jorm of systems raiher than individual
initiative, "l won't deny that the quality initiative is a
/esult of increasing competiiion," he confesses. "Bul it is

MS-s6@
also a fact that until recently,we did not have accessto
the expertiseof .professionalagencies.

HOW OBEROI IDENTIFIES CUSTOMER NEEDS


IDENTIFY THE OI,"TPUT

ls output staied in noun/verb format ?

ls it a tangible output ?

Is there a customer who


expects this output ?

Is this? new output ? Or do you produce


it on an on-going basis?

What ov€tallresultis this ouiput


workingtowards?

Is there a need for problem solving


at fhis stage ?

MS-e6@ P,T.O
IDENNFYTHE CUSTOMER

Haveyou namedthe prirnarycustomer?

Is this the person {or functional group)


who wo/ks on the outputnext ?

Doesth€ customerknowhe,/sheis the


customerfor thjs output ?

Are there others who make


use of this ollipui ?

Who is the end-lrser?

Is there a needfor probl€m,solving


at this stage2

M Se 6 o
ls there a need for problem'soNing
at this stage?

Will thesespecificationsresultin an output


thai meetscustorDetteeds ?

Are sp€cilications achievable with


the existing work pioc€ss ?

Translatecustomeis needsinto
neasurablespecifications

What requirementsdo VouexPect


your customercto have ?

Wbat changesmustyou make


to.the requtrements?

Which requremenfsdemandanalysis
oI yow work process?

Do you n€edto discussanything


with the cusiomer?

ls th€r€ a need for ptoblem-solving


at this siage ?

Ms-e6
o
Thai's why Oberoi has commissionedtle consultancy
company! McKinsey and Co., to examne ancl
recommend chang€s in all its operatior.E- including
quality standards.Scanningevery practice in the hoiel -
from the most guest-efficieni way of folding the drawsheet
in a bed to the attiiudinalchang€srequiredin employ€es
- McKinseyis implementinga charter for change
over a
six month period.

Also helping the group in iis quality quest is


D€lhi-tlased consultancy group, the Centre for GroMh
Alienatives (CGA), which has created a deiailed
flow-chad of the chain's transactionswith customers.
Points out David Wilson, 37, general manager, Hyatt
Regency, Delhi : "The Oberoi chain has al,;vayshad a
reputaiion for quality. Their biggest strength is a
consistentapproach to it."

Simultaneously,Oberoi has also launcheda training


programme to coach workers .in the use of ihe new
hardware, rotating the programm€ through different
hotels in the chain. Because of the levels of quality thai
the chain has already achieved through its founder's
insistenc€ on standards, the Oberoi Group's quality
practices today focus on providing the extra, unexpected
touch in order to get a competitive edge, rather than the
routine accommodation of demands that every guesl
makes.

MS-e6@
For instance, when a mistak€ is committed and a
guest complains, the hotel's employeesmust not only
apologise and rectify the enor, but also report to the
guest the precautionary steps being taken ro prevent rts
recurrenc€.This also makesit mandatoryfor the problem
to be traced to.lts root cause. Just what, then, are the
steps that lie at the heart ol the Oberoi quality

r Set benchmark lor ev€ry action that relates to serving

r Break down all activiiies into small, definable

r Measure each of them in mathematicai rerms so that


action standardscan be h-ackedand monitored to ascertain
consi(ency of quality.
r Set up the infrastructure and systemsto ensure that each
transaction is accomplished without error.

. Use internal and €xternal measurementsto monfor their


€fficacy.

Take a look at how this algorithm b'dnslated itself into


practice. Using guest satisfactionindic€s - which are
used for every functionr ranging from food and beverages
dnd housekeepingLo guest reldrionsand €ngineering-
which are measuresof guest satisfactionlevels and the
tim€ taken to accomplisha function, the Oberoi chain

MS-s6
o P.T.O.
pegged iie acceptable lime-limit for a check-in at six
minutes at peak hour.

ln other words, the time that must elapse bet"veen a


guest stepping into the lobby of the hotel, being met bv
the butler, having his check-in tormaliii€s completed,
being escorted to his rcom, having his L,aggagedehvered,
and the telephone armed should not exceed six minutes
if a guest is to fe€l good.

To ensure this, the process was analysed to


determine peak check-in hours, which, in turn, were used
to determine the staft strength required at diiferent times
of the day to meei the six-minuie deadline And to
support the system with infraslructure, the chajn has
-
equipped iis senior executiv€s and butlers with pagers
and is in the proc€ss oi making them al'tilable to all staff
- to ensure smooth information flows.

Using the same svstem, th€ Oberoi chain address€s


customer complaints and the mislakes thar provoked
them n6t to assign blame, but to probe tl€ flaws in
transactions and rectify them. So, when a guest
complarned about not b€ing served his favourite dinner'
y\aetg l/jeo Wan Kai - chicken green curry - the
problem was tracked down, through an analysis of every
st€p. to a uong purchasedeci.ion

MS-e6@ 10
Apparently, tbe coconut mjlk used rn this dis}I is
purchased Irom the Lakshadweep Islanos ro ensurc
consistency in quality. But becaus€ delays in dejivery had
nol been ac(ounled for. supplies of coconut milk
p€reniaily dried up. No sooner was the systern defect
detected than the lead iimes oI acquiring supplies was
changed from s€ven to 15 days - even at the risk of
over-stocking.

One crucial concept that drives this mapped-out


process is that of the intemal customer_ Every step is
delin€d as a transactionbetwee n an internal customer
and an inlemdl supDlier. >etlrng up d chain which,
ultimat€iy, cuiminates in the exiernal customer_ Or, the
guest,

ExplainsSanjiv Mathotra,33, generalmanagerj The


Oberoi, Delhi | "lntemal customerseNice has crystallised
and become targible dudng the last tito years.,,Tbis
enablesevery employeeto focus o; his or her immediate
customer, rather than the end,user alone, so that the
finer details are not missedin the attempt to get the big
picture right.

Adds RajeshKapoor, 39, managingdirector, CGA :


"The focus js on meeting customer needs through an

Mse6@ i1 P . TO .
internal backward int€gration process so thai each link in
the chain is strenglhen€d and existing conflicts are
resolved." Quality-retated problems are solved by
re-examining each transaction and intensitying the
clLstomer-client re]ationsiip in each case to ensure that
the step receivesextra attention.

THE QUAIJTY TRIANGLE

PROCESSIMPROVEMENT

TEAMWORK
LEADERSHIP

This is possible because the chain makes it a point


to provide the inflastructure lor qualiiy, ensuring that
equipmentcannot be blamed For instance,if the distaice
be\.veen ihe lip of ihe tandoor and the lnside ls not the
prescribed 12 inches, the Ob€roi chain pfefers to write

MS-s6o 12
off ihe cost of making ihe tandoor - Rs. 4,000 - than
risk a badly-cooked chicken drumstick. Nevertheless, il
somethjtg goes wrong, it can on19tte becausene sysrem
isn t delivering.

- With these crucidl componentsof the qualiiy process


in place, the Oberoi chain also empowers its employees
to inno,"ate as much as necessary in order to delight
customers. A tyavel clerk had the power - and was
rewarded for his initiative - to charter a pri!?te plane lor
an American gueit to fly from Delhi io AgIa despite
knowing that his cornpany stood to lose Rs. 20,000 in
case the guest decidedto call off his trip.

Since such empowerment makes grounding


mandatotg, employees are educated constantlv using a
training cycle of technical sl.jlls. To ensure conformiiy and
cam,, out measurements ol quality practices, intemal ,
audits are perJormed wjtb unbending rigour to ensure that
the grcup s complex manualsand checklistsare followed.

Then. they are validatedby extemal rneasurements


of results by gauging customer satisfaction through
customer comment cards, markgt research, and mystery

MS-e6@ 13 P,T,Q.
shoppers.Says Oberoi : "Our quest for perfection is not
yet over." And it won't be so as iong as the 0 01 per
cent gap b€iwe€n the Oberoi Group's quality targetsand
ils servicesremains,

THE SERVICEIMPROVEMENTMODEL

I\leasuremenl
& Feedback
Loops

ManpowDr
Measuremenls

Questions ;
(a) Describe the principal methods of the Quality
programme at Oberoi.

(b) What 'Best Practices' at Oberoi can you discern


from the case ?

(c) What do internol customers and internal


suppliers n'lean and who are such people at the
Oberoi ?

MSs6@
(d) In the context ol a service industry like hoteliering,
examine/€xplain th€ lollowing siatements and give at
least ffuee examples/rcasoDs in support of each

(i) "ln today's competitivescenario,mamrenance


of quality is not enough, you hav€ to improve
quality."

(ii) "Everybody in the company ha! to be quality


"
conscious,
(iii) "lf you want to be ahead of your competilors,
your qualityhas to be monitoredall the time."

(iv) "The only way io achieve quality is throush


training the people."

MS96@ 15 1,000
MANAGEMENT PROGBAMME
Term-End Examination
December,2OO5

MS-97@ : IIVTERNATIONAL
BUSINESS

Time : 3 hourc Maximum Morks: 100


Nvelshtose70Yo)

Note :

(i) are two sections: SectionA ond SectionB.


,There
(ii) Attempt ahy three questionsJrom SectionA which
coirg 20 rr,orkseoch.
(iii) Section B is computsory and conies 40 morks

SECTION A

l. (a) Briefly discuss the nature, scope and role o{


International Business.

{b) Explain t€ontief's theory. Why is it also known as


"Lpontief's Paradox"?

Ms-e7
@ P.T,O,
Explain the pattern of evolution of MNES in terms ol :
(i) Tgpesi (iiJ Structurei (iii) Degree of divetsificationt and
(iv) Characterisiics.

la) What co d be ihe ditferent approaches 10 conirol in


MNES ? What could be some of the negaiive ejfects i{
communication network within a corporation is tightly
drawn Lrp ?

(b) Discussthe meritsand demeritsoI multipleindicators


of performanceover single jndicator as a basis lor
performanc€evaluation.

4- (a) Discuss how socio-economic and cultural variables


influence managementthinking and practices in a
counlry In rhe context ol MNCS.

(b) Whai considerations, in general,wouldguidean MNC


in ils decision to locate produciion facilities ? Why has
ofj-shore manufacturing gained considerableground in
receni yeals ?

5 . E\plain any ,ftree of the following I

(a) Strategic ailiances

(b) Transfer pricing in international opeBtions

(c) Factors inJluencing Joreign investment decisions

(d) Flexible Manufacturing Systems

\e) Arb;tration of disputes

Ms-s7
o
SECTION B

6 . What factors,you think, havecontribuiedto me emergenc€


ot sizeableBPO service industry in tndia ? What challenges
do you thjnk the jhdustry could lace in futr/e ?

Read the following caselet and present a diagnosis o{ the


problem.Also comm€nton the statement: ,,Transplantation.
of management policiea and practic€s oI one colrmry nto
the socio economic milieu of another country may be fraught
with dangefs."

HMSI

They were so calm, so relaxed, so unhurried. They


werc at last doing somethtng for which their |raining had
equipped them. They were perfect examples of the adage
that practicemakes perfect.They were beaiing the young
men on the ground with their long stick. They were
drawing bloodi and iI some unlortunate victim dared raise
his head, he was rapidly disabled. They were the
ilfuslious Haryana police. 'fhe venue was Gurglaon, the
date 28 July, th€ occasion was a public demonstration of
the workers who were trying to disrupt ptoduction in
Honda Moiors and ScootersIndia (HMSI).

MS-e7
o P.T.O.
The Haryana police were not the only ones who
li\,€d up to their stereotyp€. HMSI was the first. There are
many stories of the laik of love betu/€en the Japanese
and trade unionjsts. Many joint ventures that wer€ staded
wilh Japanesecompani€sin the 1980s failed,Some were
tal<en over by their Japanes€ partners who inheited their
labour troubles. The Japanese became nototious lor
messing up labour relations- Admitiedly, workers becom€
restless when {actorks fail and when employ€rs cannot
meet their expectations. Failing factones are not the
easiest to manage. Whether for bett€r or lor worse,
however, the Japanese acquired a reputation lor lack of
flexibilitvand imagination.

But lhe slrange lhing is thal HMSI is not d taittng


factory. It is hugely successfr.rland immensely profitable.
Its scooterc are obscenelg populari HMSI cannot meet
demand for ihem. Customers prize their scootets And
thai is whv it was targeted by professional trade unionists
They saw a ripe f)lit r€ady io be pluched, a fat goat
ready {or sacrifice.So they regi\tereda trade union tasl
December, and ask€d for recognition. HMSI relused to
recognigethem. That is how trouble stafted.

Ms-e7@
This is just the beginning. Trade unionism is part of
the official r€ligion further east ln Calcutta, trade unions
claim not only the yighi to stop production when they feel
like; they claim the right to stop the cit9. A Calcuttan
ne\€r knorvs when his city will be btought to a halt by a
screaming column ol trade unionists Companies have
mo\r€d out of Calcutta in hordes because th€it directors
do not like riotous trad€ unionists shouting jusi outside
board rooms. There is a culture o{ noisy tgitation that the
r€st of India could do without.

Most ol all Gurgaon. For it is the only place outside


the south where ihe new industries - information
technologg and business process outsourcing - have
taken rcot. Th€y are mosi at home is Bangalore, Madras.
Bombay and Poona; somehow they Iind north of the
Vindhyas inhospitable. The only exception was Gurgaon,
whose new industry exports are close to Rs 200 billion
a gear' These are industries that wod< round the clock;
thev cannot countenance a single day's closure Southem
cities have adapted ihemselves to their demand ior
reliabiliiy; Gurgaon is the only norihem city tllat could
match them.

MS-e7@ P.T.O.
It could do so because,like the south, Gurgaon was
fr€e of trade unjonism - of bandhs, gh€raos,
demonstrations.shouting and intimidaiion. lf it is now
infected,it will in th€ long run lose out to ihe south *
as Calcuttadid lor two decadesdespiteoffering the most
attractiveinceniivesto the IT industry.But b€fore it does,
it will do consjderableharm ro the great lTiriven boom
the country is witnessing.For the Japaneseamoassador
was righi - the Japanesewill ask, is India safe trom
crippling agitations,or will they be beter off in China ?
And they will not be the only ones to ash. Alt forcign
investorswill ask the same questions.

And rhev will nol b€ pacitied by ihe loreisn


ministry's boast that India's democratic instlutions ancl
Iegal systemwill do their work. Th€y have seen these at
work in the opposition walk-out of parliament and ihe
scuflle of ihe police and Veer Mati, whose brother has
disappeared.Great theatrei but are these the sights of a
country ready for a leap forward, a country welcoming
foreign investment ?

The Left padies s€e a gr€at chance in the Central


goverqmenl5 dependenceon them - an opponunity to
spread their wings acrossthe country. The instrumeni of

Ms-e7@
their aggression is precisely the centrally organised,
externally financed, confrontative trade unionism lhai
pervades West Bengal and has now come to Gurgaon. It
may seruethe Lett's inierest,bui it is not in the interest
ol the country, which is in the middle of the first
suslainedboom sirce the one that the reforms unleashed
in ih€ early 1990s. We have often wonderedwhat would
stop this wonderfulboomi now we knolv th€ answer.The
I-eft cor.rld stop the boom, helped by a pusillanimous
government. Here is a problem that the Pnme Minister
needs to taclde quickly with all the poliiical skills at his

Ms-e7
o 1,000

You might also like